Cambridge International Examinations Cambridge International Advanced Subsidiary and Advanced Level

CHEMISTRY 9701/12

Paper 1 Multiple Choice February/March 2016

1 hour

Additional Materials: Multiple Choice Answer Sheet *4624495401 Soft clean eraser Soft pencil (type B or HB is recommended) Data Booklet

READ THESE INSTRUCTIONS FIRST

Write in soft pencil. Do not use staples, paper clips, glue or correction fluid. *

Write your name, Centre number and candidate number on the Answer Sheet in the spaces provided unless this has been done for you. DO NOT WRITE IN ANY BARCODES.

There are forty questions on this paper. Answer all questions. For each question there are four possible answers A, B, C and D. Choose the one you consider correct and record your choice in soft pencil on the separate Answer Sheet.

Read the instructions on the Answer Sheet very carefully.

Each correct answer will score one mark. A mark will not be deducted for a wrong answer. Any rough working should be done in this booklet. Electronic calculators may be used.

This document consists of 16 printed pages.

IB16 03_9701_12/4RP © UCLES 2016 [Turn over

2

Section A

For each question there are four possible answers, A, B, C, and D. Choose the one you consider to be correct.

Use of the Data Booklet may be appropriate for some questions.

1 Which compound contains two different elements with identical oxidation states?

A HCl O B Mg(OH)2 C Na2SO4 D NH4Cl

2 For the element sulfur, which pair of ionisation energies has the largest difference between them?

A third and fourth ionisation energies B fourth and fifth ionisation energies C fifth and sixth ionisation energies D sixth and seventh ionisation energies

3 Which has both more electrons than protons and more protons than neutrons? 1 2 16 [H = 1H; D = 1 H; O = 8 O]

– + – – A D B H3O C OD D OH

4 Which species contains the smallest number of electrons?

A B3+ B Be2+ C H– D He+

© UCLES 2016 9701/12/F/M/16 3

5 Nitric oxide, NO, and bromine vapour react together according to the following equation.

o –1 2NO(g) + Br2(g) → 2NOBr(g) ΔH = –23 kJ mol

–1 The reaction has an activation energy of +5.4 kJ mol .

What is the correct reaction pathway diagram for this reaction?

A B

E A

enthalpy E enthalpy ΔH o / kJ mol–1 ΔH o A / kJ mol–1

extent of reaction extent of reaction

C D

E E A A

enthalpy enthalpy / kJ mol–1 ΔH o / kJ mol–1 ΔH o

extent of reaction extent of reaction

6 Which series shows molecules in order of increasing bond angle?

A CH4 → BF3 → NH3

B H2O → CO2 → BF3

C NH3 → CH4 → CO2

D NH3 → CH4 → H2O

7 What is the volume of steam produced when 1.00 g of ice is heated to 323 °C at a pressure of 101 kPa?

3 3 3 3 A 0.27 dm B 1.3 dm C 2.7 dm D 48 dm

© UCLES 2016 9701/12/F/M/16 [Turn over 4

8 Solid carbon dioxide, CO2, is similar to solid iodine, I2, in its structure and properties. Carbon is in Group 14. Silica, SiO2, is a Group 14 compound.

Which statement about solid CO2 and solid SiO2 is correct?

A Both solids exist in a lattice structure. B Both solids have a simple molecular structure. C Both solids have joined by single covalent bonds. D Both solids change spontaneously to gas at s.t.p.

9 An article in a science magazine contains the following statement.

‘It is lighter than a feather, stronger than steel, yet incredibly flexible and more conductive than copper.’

Which form of carbon is being described?

A buckminsterfullerene B diamond C graphene D graphite

10 Which equation has an enthalpy change of reaction which corresponds to the standard enthalpy change of atomisation of chlorine?

1 A C (g) → C (g) 2 l 2 l

1 B C (l) → C (g) 2 l 2 l

C Cl 2(g) → 2Cl (g)

D Cl 2(l) → 2Cl (g)

11 In an experiment, 2.00 mol of and 3.00 mol of iodine were heated together in a sealed container and allowed to reach equilibrium at a fixed temperature. The container had a fixed 3 volume of 1.00 dm . At equilibrium, there were 2.40 mol of iodine present in the mixture.

H2(g) + I2(g) 2HI(g)

What is the value of the equilibrium constant, Kc?

A 0.107 B 0.357 C 0.429 D 2.33

© UCLES 2016 9701/12/F/M/16 5

12 Consecutive elements X, Y and Z are in Period 3 of the Periodic Table. Element Y has the highest first ionisation energy and the lowest melting point of these three elements.

What are the identities of X, Y and Z?

A sodium, magnesium, aluminium B magnesium, aluminium, silicon C aluminium, silicon, phosphorus D silicon, phosphorus, sulfur

13 When dealing with a spillage of metallic sodium it is important that no toxic or flammable products are formed.

Which material should be used if there is a spillage of metallic sodium?

A dilute hydrochloric acid B ethanol C sand D water spray

14 Chlorine gas is widely used to treat contaminated water.

Which species present in water when chlorine gas has been added is responsible for killing bacteria?

– – – A Cl O2 B Cl C HCl D OCl

15 Which row of the table is correct?

shape bonds present ammonium ammonia ammonium molecule ion molecule ion

A pyramidal regular tetrahedral σ σ B pyramidal regular tetrahedral σ π C regular tetrahedral pyramidal σ σ D regular tetrahedral pyramidal π σ

© UCLES 2016 9701/12/F/M/16 [Turn over 6

16 gas and gas have different thermal stabilities. The difference is due to a difference in the energies of some of the covalent bonds that are involved in the decomposition.

Which row identifies the more stable of the two compounds, and gives the correct explanation?

identity of the more explanation for the

thermally stable compound difference in stability

A hydrogen chloride the Cl–Cl bond is stronger than the I–I bond B hydrogen chloride the H–Cl bond is stronger than the H–I bond

C hydrogen iodide the Cl –Cl bond is stronger than the I–I bond D hydrogen iodide the H–Cl bond is stronger than the H–I bond

17 Ammonium sulfate, (NH4)2SO4, and ammonium nitrate, NH4NO3, are used as fertilisers.

These salts have different percentages by mass of nitrogen. They have the same effect as each other on the pH of neutral soil.

Which row is correct?

higher percentage effect on pH of soil of nitrogen by mass

A ammonium nitrate decrease B ammonium nitrate increase C ammonium sulfate decrease D ammonium sulfate increase

2+ 2+ 2– – − – 18 River water in a chalky agricultural area contains Ca , Mg , CO3 , HCO3 , Cl and NO3 . In a water treatment plant, such water is treated by adding a calculated quantity of calcium hydroxide.

What will be precipitated from the river water following the addition of calcium hydroxide?

A CaCl 2 B CaCO3 C Ca(NO3)2 D Mg(NO3)2

© UCLES 2016 9701/12/F/M/16 7

19 After black and white photographic film has been developed, unreacted silver bromide is removed by reaction with sodium thiosulfate.

+ – 3– AgBr + 2Na2S2O3 → 4Na + Br + [Ag(S2O3)2]

What is the function of the thiosulfate ion?

A to make the silver ions soluble B to oxidise the silver ions C to reduce the bromine D to reduce the silver ions

20 People who take statin drugs to control their blood cholesterol may also take ‘coenzyme Q10’.

The diagram shows a simplified structure of one form of this coenzyme.

O

CH3O

CH3O H O n

coenzyme Q 10

Which row describes this structure correctly?

number of π bonds the coenzyme is in one molecule

A an aldehyde n + 2 B an aldehyde n + 4 C a ketone n + 2 D a ketone n + 4

© UCLES 2016 9701/12/F/M/16 [Turn over 8

21 Geraniol and nerol are compounds found in some flower fragrances. They are isomers of each other.

CH3 CH3

C C CH2OH H H2C C H2C C

H CH2OH H3C H2C H3C H2C CC CC

H3C H H3C H

geraniol nerol

Which type of isomerism is shown here?

A chain B geometrical (cis-trans) C optical D positional

22 A section showing two repeat units of an addition polymer is shown.

CH3 CH3 CH3 CH3

C C C C

Cl CH Cl CH 3 3

What is the identity of the monomer that produced this polymer?

A 2-chloro-3-methylbutane B 2-chloro-3-methylbut-2-ene C 2-chloropent-2-ene D 2,4-dichloro-3,3,4,5-tetramethylhexane

© UCLES 2016 9701/12/F/M/16 9

23 But-2-ene-1,4-diol is converted in two steps through an intermediate X into oxobutanedioic acid.

step 1 step 2 X HOCH2CH=CHCH2OH HO2CCOCH2CO2H hot, acidified KMnO4 but-2-ene-1,4-diol oxobutanedioic acid

What could be the reagent for step 1 and what is the intermediate X?

reagent for step 1 X

A cold, acidified KMnO4 HOCH2CH2CH(OH)CH2OH

B hot, acidified K2Cr2O7 HO2CCH=CHCO2H

C steam and concentrated H2SO4 HOCH2CH(OH)CH2CH2OH

D warm, acidified K2Cr2O7 OHCCH(OH)CH2CHO

24 can be added to T to give compound U. Compound U can be hydrolysed to compound V.

step 1 step 2 Br OH

T U V

Four students, W, X, Y and Z, made the following statements.

W All the atoms in a molecule of compound T lie in the same plane. X Compound V contains only one chiral centre. Y Step 1 is an electrophilic addition reaction. Z Step 2 is a nucleophilic substitution reaction.

Which two students are correct?

A W and Y B W and Z C X and Y D Y and Z

© UCLES 2016 9701/12/F/M/16 [Turn over 10

25 Structural isomerism and stereoisomerism should be considered in answering this question.

Compound J is reacted with KOH dissolved in ethanol. Three isomeric with molecular formula C4H8 are formed.

What is J?

A CH3 CH2 CH2 CH2 Br

B CH3 CH CH2 CH3

Br

C CH3 CH CH2 Br

CH3

CH3 D CH3 C Br

CH 3

26 Which row correctly shows a primary, a secondary and a tertiary alcohol?

primary secondary tertiary

CH2OH CH2OH CH2OH

A CH2 CHOH CHOH

CH3 CH3 CH2OH

CH2OH CH3 CH3 B CH3 C H CH3 C OH CH3 C H

CH3 CH3 CH2OH

CH2OH CH2OH CH2OH C CH3 C H CH3 C CH2OH CH3 C CH2OH

H H CH2OH

H CH3 CH3 D CH3 C OH CH3 C OH CH3 C OH

H H CH3

© UCLES 2016 9701/12/F/M/16 11

27 The fragrance compounds of perfumes are often dissolved in solvent Y, which has a molecular formula C7H12O4. It is made by reacting -1,2-diol with ethanoic acid in the presence of an acid catalyst.

H CH3

HO C C OH

H H

propane-1,2-diol

What is the structure of solvent Y?

A B

O H CH3 O O H CH3 O

C O C C C C C C C

H3C H H OCH3 H3C OOH H CH3

C D

O H CH3 O O H CH3 O

C C C OC C O C C O C

H C O CH H C CH 3 H H 3 3 H H 3

28 Which mixture could be used to produce propyl methanoate?

A CH3CH2CO2H and CH3OH

B CH3CH2CH2CH2OH and HCO2H

C CH3CH2CH2OH and HCO2H

D CH3CH2CH2CO2H and CH3OH

29 Which statement about poly(chloroethene) is correct?

A The polymer can be cracked to produce chlorinated alkenes. B The polymer has harmless combustion products. C The polymer is readily biodegradable when buried.

D The repeat unit of the polymer has an Mr of 97.

© UCLES 2016 9701/12/F/M/16 [Turn over 12

30 Which row of the table is correct?

increasing number of carbon atoms

A ethyl methanoate methyl propanoate pentyl pentanoate propyl butanoate B ethyl methanoate methyl propanoate propyl butanoate pentyl pentanoate C methyl propanoate propyl butanoate ethyl methanoate pentyl pentanoate D propyl butanoate ethyl methanoate pentyl pentanoate methyl propanoate

© UCLES 2016 9701/12/F/M/16 13

Section B

For each of the questions in this section, one or more of the three numbered statements 1 to 3 may be correct.

Decide whether each of the statements is or is not correct (you may find it helpful to put a tick against the statements that you consider to be correct).

The responses A to D should be selected on the basis of

A B C D

1, 2 and 3 1 and 2 2 and 3 1 only are only are only are is correct correct correct correct

No other combination of statements is used as a correct response.

31 Which molecules have an overall dipole moment?

1 carbon monoxide, CO

2 , PH3

3 carbon dioxide, CO2

32 The diagram illustrates the enthalpy changes of a set of reactions

ΔH = –134 kJ mol–1 R S

ΔH = +92 kJ mol–1

ΔH = –75 kJ mol–1 T U

Which statements are correct?

–1 1 The enthalpy change for the transformation U → R is + 42 kJ mol .

2 The enthalpy change for the transformation T → S is endothermic.

–1 3 The enthalpy change for the transformation R → T is – 33 kJ mol .

© UCLES 2016 9701/12/F/M/16 [Turn over 14

The responses A to D should be selected on the basis of

A B C D

1, 2 and 3 1 and 2 2 and 3 1 only are only are only are is correct correct correct correct

No other combination of statements is used as a correct response.

33 Which statements about reversible reactions are correct?

1 An increase in concentration of a reactant always increases the concentration of the product. 2 An increase in temperature always increases the rate at which the equilibrium is established. 3 An increase in temperature always increases the concentration of the product at equilibrium.

34 A chemist puts a sample of dilute aqueous hydrochloric acid into beaker 1. She adds a sample of zinc and measures the rate of production of hydrogen gas.

She then puts a different sample of dilute aqueous hydrochloric acid into beaker 2. She adds a different sample of zinc and measures the rate of production of hydrogen gas.

The rate of the reaction in beaker 2 is greater than the rate of the reaction in beaker 1.

Which factors could help to explain this observation?

1 The reaction in beaker 1 has a higher activation energy than the reaction in beaker 2. 2 The zinc in beaker 1 is in larger pieces than the zinc in beaker 2. 3 The acid in beaker 1 is at a lower concentration than the acid in beaker 2.

35 In some rice-growing parts of the world, farmers use a combination of paddy fields and a fish farm. Rice paddy fields are flooded for much of the growing cycle and water running off the fields flows through pens where fish are raised. Nitrogen-based fertilisers are generally very soluble in water.

Which problems could result from farmers applying excess nitrogen-based fertilisers to their paddy fields?

1 decreased fish production in the fish pens 2 decreased levels of oxygen in the water 3 increased growth of algae in the fish pens

© UCLES 2016 9701/12/F/M/16 15

36 A sample containing 0.40 mol of calcium nitrate was decomposed by heating in a roaring Bunsen burner flame until there was no further decomposition.

What are the products of this reaction?

1 0.40 mol of calcium oxide

2 0.40 mol of nitrogen, N2(g)

3 0.40 mol of oxygen, O2(g)

37 Which reagents, when used in an excess, can be used to make sodium lactate, CH3CH(OH)CO2Na, from lactic acid, CH3CH(OH)CO2H?

1 Na

2 NaHCO3 3 NaOH

38 Chlorine atoms in the upper atmosphere cause the breakdown of ozone.

Cl + O3 → O2 + Cl O

Cl O + O → Cl + O2

Which statements are correct when referring to these chlorine atoms?

1 The chlorine atoms act as catalysts. 2 The chlorine atoms are free radicals. 3 The chlorine atoms are formed by heterolytic fission of a covalent bond in chlorofluorocarbons.

39 An unknown organic compound Z reacts with sodium to give a combustible gas as one product. Z does not give a yellow precipitate with alkaline aqueous iodine.

What is a possible identity of Z?

1 ethanoic acid 2 pentan-3-ol 3 propan-1-ol

© UCLES 2016 9701/12/F/M/16 [Turn over 16

The responses A to D should be selected on the basis of

A B C D

1, 2 and 3 1 and 2 2 and 3 1 only are only are only are is correct correct correct correct

No other combination of statements is used as a correct response.

40 Compound X has the molecular formula C3H6O3.

Heating X under reflux with acidified K2Cr2O7 forms HO2CCOCO2H.

Reacting X with NaBH4 forms HOCH2CH(OH)CH2OH.

What is a possible structural formula for X?

1 HOCH2CH2CO2H

2 HOCH2CH(OH)CHO

3 HOCH2COCH2OH

Permission to reproduce items where third-party owned material protected by copyright is included has been sought and cleared where possible. Every reasonable effort has been made by the publisher (UCLES) to trace copyright holders, but if any items requiring clearance have unwittingly been included, the publisher will be pleased to make amends at the earliest possible opportunity.

To avoid the issue of disclosure of answer-related information to candidates, all copyright acknowledgements are reproduced online in the Cambridge International Examinations Copyright Acknowledgements Booklet. This is produced for each series of examinations and is freely available to download at www.cie.org.uk after the live examination series.

Cambridge International Examinations is part of the Cambridge Assessment Group. Cambridge Assessment is the brand name of University of Cambridge Local Examinations Syndicate (UCLES), which is itself a department of the University of Cambridge.

© UCLES 2016 9701/12/F/M/16 Cambridge International Examinations Cambridge International Advanced Subsidiary and Advanced Level *8398966658*

CHEMISTRY 9701/22

Paper 2 AS Level Structured Questions February/March 2016

1 hour 15 minutes Candidates answer on the Question Paper. Additional Materials: Data Booklet

READ THESE INSTRUCTIONS FIRST

Write your Centre number, candidate number and name on all the work you hand in. Write in dark blue or black pen. You may use an HB pencil for any diagrams or graphs. Do not use staples, paper clips, glue or correction fluid. DO NOT WRITE IN ANY BARCODES.

Answer all questions. Electronic calculators may be used. You may lose marks if you do not show your working or if you do not use appropriate units. A Data Booklet is provided.

At the end of the examination, fasten all your work securely together. The number of marks is given in brackets [ ] at the end of each question or part question.

This document consists of 13 printed pages and 3 blank pages.

IB16 03_9701_22/5RP © UCLES 2016 [Turn over 2

Answer all the questions in the spaces provided.

1 This question is about Period 3 elements and their compounds.

(a) Give an explanation for each of the following statements.

(i) The atomic radius decreases across Period 3 (Na to Ar).

......

......

......

...... [2]

(ii) The first ionisation energy of sulfur is lower than that of phosphorus.

......

......

......

...... [2]

(iii) Sodium is a better electrical conductor than phosphorus.

......

......

......

...... [2]

(iv) Magnesium is a better electrical conductor than sodium.

......

...... [1]

© UCLES 2016 9701/22/F/M/16 3

(b) The flow chart below shows a series of reactions.

Mg(s) HNO3(aq) A(aq) + B(g)

reagent X(aq) heat

white precipitate, E(s) MgO(s) + H2O(g) + C(g) + D(g)

(i) Give the formula of each of the compounds A to D.

A ...... B ......

C ...... D ...... [4]

(ii) E reacts with dilute aqueous acid to produce a gas that turns limewater cloudy.

Suggest the identity of reagent X.

...... [1]

[Total: 12]

© UCLES 2016 9701/22/F/M/16 [Turn over 4

2 Spathose is an iron ore that contains iron(II) carbonate, FeCO3. The percentage of iron(II) carbonate in spathose can be determined by titration with acidified potassium dichromate(VI) solution using a suitable indicator.

The ionic equation is shown below.

2– + 2+ 3+ 3+ Cr2O7 (aq) + 14H (aq) + 6Fe (aq) → 2Cr (aq) + 6Fe (aq) + 7H2O(l)

(a) A 5.00 g sample of spathose was reacted with excess concentrated hydrochloric acid and then filtered. The filtrate was made up to 250 cm3 in a volumetric flask with distilled water.

A 25.0 cm3 sample of the standard solution required 27.30 cm3 of 0.0200 mol dm–3 dichromate(VI) solution for complete reaction.

(i) Calculate the amount, in moles, of dichromate(VI) ions used in the titration.

amount = ...... mol [1]

(ii) Use your answer to (i) to calculate the amount, in moles, of Fe2+ present in the 25.0 cm3 sample.

amount = ...... mol [1]

(iii) Use your answer to (ii) to calculate the amount, in moles, of Fe2+ present in the 250 cm3 volumetric flask.

amount = ...... mol [1]

(iv) Use your answer to (iii) to calculate the mass of iron(II) carbonate present in the sample of spathose.

mass = ...... g [2]

(v) Calculate the percentage of iron(II) carbonate in the sample of spathose.

percentage of iron(II) carbonate = ...... % [1]

© UCLES 2016 9701/22/F/M/16 5

(b) Iron ores containing iron(III) compounds can be analysed using a similar method.

A standard solution of an aqueous iron(III) compound is reacted with aqueous tin(II) chloride. Aqueous tin(IV) chloride and aqueous iron(II) chloride are the products of this reaction.

(i) Write an ionic equation for this reaction. Do not include state symbols.

...... [2]

(ii) Any excess tin(II) chloride can be removed by reaction with HgCl 2(aq). A white precipitate

of Hg2Cl 2 is produced.

Complete the equation for this reaction.

...... (.....) + .....HgCl 2(aq) → SnCl 4(.....) + Hg2Cl 2(.....) [2]

[Total: 10]

© UCLES 2016 9701/22/F/M/16 [Turn over 6

3 Over one million tonnes of , HCN, are produced each year using the Andrussow process. The overall equation for the reaction is shown.

1 CH4(g) + NH3(g) + 1 2 O2(g) HCN(g) + 3H2O(g)

(a) (i) Draw a dot-and-cross diagram to represent the bonding in a molecule of ammonia, NH3, and state the shape of the molecule.

shape of molecule ...... [3]

(ii) A molecule of hydrogen cyanide, HCN, is shown. H C N

The bonding between the carbon and nitrogen atoms consists of one sigma (σ) bond and two pi (π) bonds.

Sketch the shape of the sigma bond and one of the pi bonds in the space below. Show clearly the position of the atomic nuclei in each diagram.

sigma (σ) pi (π) [2]

© UCLES 2016 9701/22/F/M/16 7

(b) The reaction exists as a dynamic equilibrium.

(i) Explain what is meant by the term dynamic equilibrium.

......

...... [1]

(ii) State and explain how the amounts of the chemicals present in the equilibrium mixture will change when the pressure is increased.

......

......

...... [2]

(c) The process uses a platinum catalyst, which increases the rate of reaction.

Sketch a Boltzmann distribution on the axes given below and use your diagram to explain how the platinum catalyst increases the rate of the reaction.

number of molecules

molecular energy

......

......

......

...... [3]

© UCLES 2016 9701/22/F/M/16 [Turn over 8

(d) The reaction of hydrogen cyanide with propanone is an important first step in many organic syntheses.

(i) Give the full name of the mechanism of this reaction.

...... [1]

(ii) Complete the diagram to show the mechanism of the reaction of hydrogen cyanide with propanone. Draw the structure of the intermediate and the product of the reaction. Include all relevant charges, partial charges, curly arrows and lone pairs.

O

C

H3C CH3

–CN [5]

[Total: 17]

© UCLES 2016 9701/22/F/M/16 10

4 The following compounds were all found to be components of a sample of petrol.

H CH3

CH3(CH2)2CH3 (CH3)3CCH2CH(CH3)2 H3C C COH

H3C H

G H J

(a) (i) Give the molecular formula of compound G.

...... [1]

(ii) Give the empirical formula of compound H.

...... [1]

(iii) Draw the skeletal formula of compound J.

[1]

(b) Write an equation to represent the complete combustion of compound H.

...... [1]

(c) Fossil fuels are often contaminated with sulfur.

State and explain why supplies of fossil fuels that contain sulfur pose a problem to the environment.

......

......

...... [2]

© UCLES 2016 9701/22/F/M/16 11

(d) The boiling points of compounds G, H and J are shown below.

compound G H J boiling point / °C 0 99 112

Explain the differences in the boiling points of the three compounds.

......

......

......

......

......

......

...... [4]

(e) Compound J can be produced from 2-chloro-3-methylbutane, C5H11Cl.

Give the reagent(s) and conditions for this reaction.

...... [1]

[Total: 11]

© UCLES 2016 9701/22/F/M/16 [Turn over 12

5 Some reactions of compound P, C5H8O, are shown.

compound Q

H2(g) Ni(s) catalyst

O cold, acidified NaBH4 KMnO4(aq) compound R compound S compound P

OH–(aq)

I2(aq)

O compound T + –O

(a) (i) Give the structures for organic compounds Q, R, S and T.

Q

R S

T

[4]

© UCLES 2016 9701/22/F/M/16 13

(ii) Give the systematic name of compound P.

...... [1]

(iii) What would you observe when compound P is reacted with 2,4-dinitrophenylhydrazine (2,4-DNPH)?

...... [1]

(b) Compound U contains a chiral centre and has the same molecular formula as compound P,

C5H8O.

• Compound U readily decolourises a sample of bromine water. • Compound U does not show cis-trans isomerism. • When compound U is heated under reflux in the presence of excess acidified potassium dichromate(VI), the organic product gives the infra-red spectrum shown.

infra-red spectrum of product 100

transmittance / % 50

0 4000 3000 2000 1500 1000 500 wavenumber / cm–1

Use the information given to suggest a structure for compound U. Explain your answer.

[4]

[Total: 10]

© UCLES 2016 9701/22/F/M/16 [Turn over

Cambridge International Examinations Cambridge International Advanced Subsidiary and Advanced Level

CHEMISTRY 9701/12

Paper 1 Multiple Choice February/March 2017

1 hour

Additional Materials: Multiple Choice Answer Sheet *7298237059* Soft clean eraser Soft pencil (type B or HB is recommended) Data Booklet

READ THESE INSTRUCTIONS FIRST

Write in soft pencil. Do not use staples, paper clips, glue or correction fluid. Write your name, Centre number and candidate number on the Answer Sheet in the spaces provided unless this has been done for you. DO NOT WRITE IN ANY BARCODES.

There are forty questions on this paper. Answer all questions. For each question there are four possible answers A, B, C and D. Choose the one you consider correct and record your choice in soft pencil on the separate Answer Sheet.

Read the instructions on the Answer Sheet very carefully.

Each correct answer will score one mark. A mark will not be deducted for a wrong answer. Any rough working should be done in this booklet. Electronic calculators may be used.

This document consists of 14 printed pages and 2 blank pages.

IB17 03_9701_12/4RP © UCLES 2017 [Turn over

2

Section A

For each question there are four possible answers, A, B, C and D. Choose the one you consider to be correct.

Use of the Data Booklet may be appropriate for some questions.

– 1 Which ion has the same electronic configuration as Cl ?

A F– B P+ C Sc3+ D Si4+

2 Compounds J and K each contain 40% carbon by mass.

What could J and K be?

J K

A a hexose, C6H12O6 starch, (C6H10O5)n

B a pentose, C5H10O5 a hexose, C6H12O6

C a pentose, C5H10O5 sucrose, C12H22O11

D starch, (C6H10O5)n sucrose, C12H22O11

3 Two moles of compound P were placed in a sealed container. The container was heated and P was partially decomposed to produce Q and R only. A dynamic equilibrium between P, Q and R was established.  x  At equilibrium x moles of R were present and the total number of moles present was 2 +  .  2  What is the equation for this reversible reaction?

A P 2Q + R B 2P 2Q + R C 2P Q + R D 2P Q + 2R

© UCLES 2017 9701/12/F/M/17 3

4 The boiling points of , , propane and are given.

compound CH4 CH3CH3 CH3CH2CH3 CH3CH2CH2CH3

boiling point / K 112 185 231 273

Which statement explains the increase in boiling point from methane to butane?

A Closer packing of molecules results in stronger van der Waals’ forces. B More covalent bonds are present and therefore more energy is required to break the bonds. C More electrons in the molecules results in stronger van der Waals’ forces. D More hydrogen atoms in the molecules results in stronger hydrogen bonding.

5 The characteristic smell of garlic is due to alliin.

H HONH2 y z x C CSCH2 CH2 C H

H CO2H alliin

What are the approximate bond angles x, y and z in a molecule of alliin?

x y z

A 90o 90o 109o B 120o 109o 90o C 120o 120o 109o D 180o 109o 109o

6 Which gas sample contains the fewest molecules?

3 A 1.00 dm of carbon dioxide at 27 °C and 2.0 kPa

3 B 1.00 dm of hydrogen at 100 °C and 2.0 kPa

3 C 1.00 dm of nitrogen at 300 °C and 4.0 kPa

3 D 1.00 dm of oxygen at 250 °C and 3.0 kPa

© UCLES 2017 9701/12/F/M/17 [Turn over 4

3 –3 7 A student mixed 25.0 cm of 4.00 mol dm hydrochloric acid with an equal volume of –3 4.00 mol dm sodium hydroxide. The initial temperature of both solutions was 15.0 °C. The maximum temperature recorded was 30.0 °C.

Using these results, what is the enthalpy change of neutralisation of hydrochloric acid?

–1 A –62.7 kJ mol

–1 B –31.4 kJ mol

–1 C –15.7 kJ mol

–1 D –3.14 kJ mol

8 HOCl (aq) is the molecule that kills bacteria when chlorine is added to water.

The following reaction produces this molecule.

+ – Cl 2(g) + H2O(I) HOCl (aq) + H (aq) + Cl (aq)

Which statement about this reaction is correct?

A Chlorine is both oxidised and reduced. B Chlorine is oxidised but not reduced. C Hydrogen is both oxidised and reduced. D Hydrogen is oxidised but not reduced.

9 All the reactants and products of an exothermic reaction are gaseous.

Which statement about this reaction is correct?

A The total bond energy of the products is less than the total bond energy of the reactants, and ∆H for the reaction is negative. B The total bond energy of the products is less than the total bond energy of the reactants, and ∆H for the reaction is positive. C The total bond energy of the products is more than the total bond energy of the reactants, and ∆H for the reaction is negative. D The total bond energy of the products is more than the total bond energy of the reactants, and ∆H for the reaction is positive.

© UCLES 2017 9701/12/F/M/17 5

10 Oxidation of ethanedioate ions by acidified manganate(VII) ions is very slow at room temperature.

– 2– + 2+ 2MnO4 + 5C2O4 + 16H → 2Mn + 10CO2 + 8H2O

Mn2+ ions catalyse this reaction.

Which graph shows how the concentration of acidified manganate(VII) ions varies after ethanedioate ions are added?

A B

concentration concentration – – of MnO4 of MnO4

time time

C D

concentration concentration – – of MnO4 of MnO4

time time

11 Ammonia is manufactured from nitrogen and hydrogen using the Haber process.

N2(g) + 3H2(g) 2NH3(g)

What is the expression for Kc for this equilibrium? 2[NH (g)] A 3 [N2(g)] + 3[H2(g)]

2[NH (g)] B 3 [N2(g)] × 3[H2(g)]

2 [NH3(g)] C 3 [N2(g)] + [H2(g)]

2 [NH3(g)] D 3 [N2(g)] × [H2(g)]

© UCLES 2017 9701/12/F/M/17 [Turn over 6

12 The electrical conductivities of two compounds, Y and Z, are shown in the table.

Y Z conductivity of the compound does not good in the liquid state conduct conductivity of the mixture obtained good good by adding the compound to water

What could compounds Y and Z be?

Y Z

A Al 2O3 SiCl 4

B NaF Al 2O3

C NaF SiCl 4

D SiCl 4 Al 2O3

13 Which reagent, when mixed with ammonium sulfate and then heated, liberates ammonia?

A aqueous bromine B dilute hydrochloric acid C limewater

D potassium dichromate(VI) in acidic solution

14 Why does barium react more rapidly with cold water than magnesium does?

A Barium atoms are larger and form ions more easily than magnesium atoms. B Barium floats on the surface of the water but magnesium sinks in the water. C Barium hydroxide is less soluble than magnesium hydroxide. D The sum of the 1st and 2nd ionisation energies of barium is more than that for magnesium.

© UCLES 2017 9701/12/F/M/17 7

15 The magnitude of property X of five elements from the third period of the Periodic Table, P, Q, R, S and T is shown. P, Q, R, S and T have consecutive atomic numbers. The letters do not represent the symbols of the elements.

S

X Q R

P T

atomic number

Which row correctly identifies property X and element R?

property X element R

A electrical conductivity Al B Si C melting point Al D melting point Si

16 Some uses of chlorine and bromine are given.

Which is a use of bromine?

A making bleaches for textiles and the paper industry B making CFCs C making flame retardants and fire extinguishers D making the polymer PVC

17 In an experiment, 0.600 mol of chlorine gas, Cl 2, is reacted with an excess of hot aqueous sodium hydroxide. One of the products is NaCl O3.

Which mass of NaCl O3 is formed?

A 21.3 g B 44.7 g C 63.9 g D 128 g

© UCLES 2017 9701/12/F/M/17 [Turn over 8

18 Z is an anhydrous compound of a Group 2 element. When it is heated, Z undergoes thermal decomposition to produce two different gases. Z has relatively low thermal stability compared to other Group 2 compounds containing the same anion as Z.

What is compound Z?

A barium carbonate B barium nitrate C magnesium carbonate D magnesium nitrate

19 Methylamine, CH3NH2, has very similar chemical properties to ammonia, NH3.

Methylamine reacts with hydrogen chloride to form a white crystalline salt, methylammonium chloride.

+ – CH3NH2 + HCl → CH3NH3 Cl

A sample of methylammonium chloride is heated with aqueous sodium hydroxide.

What are the products?

A ammonia, sodium chloride and water B ammonia, sodium hydrogencarbonate and sodium chloride C methylamine, hydrogen chloride and water D methylamine, sodium chloride and water

20 Structural isomerism and stereoisomerism should be considered when answering this question.

How many isomers with the formula C4H8 have structures that contain a π bond?

A 1 B 2 C 3 D 4

© UCLES 2017 9701/12/F/M/17 9

21 Kerosene is used as an aircraft fuel. Q is one of the molecules in kerosene and has the skeletal formula shown.

Q

Other structural isomers of this molecule are also found in kerosene.

Which structure is a structural isomer of Q?

A B CD

22 Which radical is most likely to form by the homolytic fission of one covalent bond in bromochloromethane, CH2BrCl ?

A •CH2Cl B •CH2Br C •CHBrCl D •CH2BrCl

23 undergoes a variety of reactions.

Which row is correct?

reagent added to propene products include

A Br2(aq) 1-bromopropane

B cold, dilute, acidified KMnO4(aq) propanoic acid C HBr(g) 2-bromopropane

D hot, concentrated, acidified KMnO4(aq) propanoic acid

24 Polymerisation of ethene gives poly(ethene).

How does the bonding between carbon atoms in poly(ethene) compare with that in ethene?

A longer and stronger in poly(ethene) B longer and weaker in poly(ethene) C shorter and stronger in poly(ethene) D shorter and weaker in poly(ethene)

© UCLES 2017 9701/12/F/M/17 [Turn over 10

25 Diols in which both hydroxy groups are bonded to the same carbon can spontaneously eliminate a molecule of water to produce a carbonyl compound.

Which compound, after complete hydrolysis, gives a positive reaction with Tollens’ reagent?

A 1,1-dibromobutane B 1,2-dibromobutane C 1,3-dibromobutane D 2,2-dibromobutane

26 Propan-2-ol undergoes many reactions.

Which row is correct?

reagent added product to propan-2-ol

A acidified KMnO4 CH3CH2CHO

B Cl 2 CH3CHCl CH3

C conc. H2SO4 CH3CHCH2

D methanoic acid HCO2CH2CH2CH3

27 Ethane-1,2-diol has the following structure.

H

HHC O

HHC O

H

Without breaking the C–C bond, there are five possible oxidation products.

What is the total number of aldehyde groups and carboxylic acid groups in these five products?

–CHO –COOH

A 3 3 B 3 4 C 4 3 D 4 4

© UCLES 2017 9701/12/F/M/17 11

28 CH3CH2COCH2CH3 reacts with hydrogen cyanide to form an organic product called a cyanohydrin.

Which statement is correct?

A The cyanohydrin product has one chiral centre. B The cyanohydrin product is formed by electrophilic addition. C The cyanohydrin product is formed via an intermediate which contains a C–OH group. D The formation of the cyanohydrin product requires the use of cyanide ions as a catalyst.

29 Compound X, CH3CH(OH)CH(CHO)CH3, is heated under reflux with an excess of acidified K2Cr2O7 to form compound Y.

Both X and Y are separately warmed with Fehling’s solution and the observations noted.

What are the observations?

A Both X and Y give a red precipitate. B Only X gives a red precipitate. C Only Y gives a red precipitate. D Neither X nor Y gives a red precipitate.

30 Which two compounds can react together to produce an ester?

A B

+ + OH O OH O

C D

HO O O OH + + O OH HO OH

© UCLES 2017 9701/12/F/M/17 [Turn over 12

Section B

For each of the questions in this section, one or more of the three numbered statements 1 to 3 may be correct.

Decide whether each of the statements is or is not correct (you may find it helpful to put a tick against the statements that you consider to be correct).

The responses A to D should be selected on the basis of

A B C D

1, 2 and 3 1 and 2 2 and 3 1 only are only are only are is correct correct correct correct

No other combination of statements is used as a correct response.

31 Nitrogen and phosphorus are both in Group 15 of the Periodic Table. Phosphorus forms a chloride with the formula PCl 5 but nitrogen does not form NCl 5.

Which statements help to explain this?

1 Nitrogen’s outer shell cannot contain more than eight electrons. 2 Nitrogen cannot have an oxidation state of +5. 3 Nitrogen is less electronegative than phosphorus.

32 When a sample of a gas is compressed at constant temperature from 1500 kPa to 6000 kPa, its 3 3 volume changes from 76.0 cm to 20.5 cm .

Which statements are possible explanations for this result?

1 The gas does not behave ideally. 2 The gas partially liquefies. 3 Some of the gas is lost from the container.

33 Which of these statements are always correct?

1 The sum of the oxidation numbers of all the atoms in a compound is zero. 2 The oxidation number of sodium in a salt is positive. 3 The oxidation number of chlorine in a compound is negative.

© UCLES 2017 9701/12/F/M/17 13

34 Sulfur dioxide and oxygen react in the gas phase.

–1 2SO2(g) + O2(g) 2SO3(g) ∆H = –197 kJ mol

Which statements are correct?

1 Increasing the pressure increases the equilibrium yield of SO3.

2 Increasing the temperature lowers the value of the equilibrium constant Kp.

3 The presence of a vanadium(V) oxide catalyst increases the equilibrium yield of SO3.

35 Three test-tubes, X, Y, and Z, each contain a small amount of water.

● A small amount of NaCl is added to test-tube X.

● A small amount of SiCl 4 is added to test-tube Y.

● A small amount of Al Cl 3 is added to test-tube Z.

After a short time, two drops of Universal Indicator solution are added to each test-tube.

Which observations are made?

1 The indicator added to test-tube X stays green. 2 The indicator added to test-tube Y turns red. 3 The indicator added to test-tube Z turns red.

36 Sodium chloride and sodium iodide react with concentrated .

Which statements are correct?

1 Sodium chloride is not oxidised by concentrated sulfuric acid. 2 No colour change is seen when concentrated sulfuric acid is added to sodium chloride. 3 Sodium iodide is oxidised by concentrated sulfuric acid.

37 Which compounds can be obtained from but-2-ene in a single reaction?

1 CH3CH(OH)CH(OH)CH3

2 CH3CH(OH)CH2CH3

3 CH3CO2H

© UCLES 2017 9701/12/F/M/17 [Turn over 14

The responses A to D should be selected on the basis of

A B C D

1, 2 and 3 1 and 2 2 and 3 1 only are only are only are is correct correct correct correct

No other combination of statements is used as a correct response.

38 Which statements help to explain the mechanism of the reaction between 1-chloropropane and ammonia?

1 1-chloropropane has a δ– chlorine that forms hydrogen bonds with a δ+ hydrogen atom in ammonia.

2 1-chloropropane is a polar compound with a δ+ carbon atom. 3 There is a lone pair of electrons on the nitrogen atom in ammonia.

39 Which compounds react with alkaline aqueous iodine to give a pale yellow precipitate of tri-iodomethane?

1 butanone 2 ethanal 3 propan-2-ol

40 Propanoic acid occurs naturally as a result of the bacterial fermentation of milk and is partly responsible for the flavour of Swiss cheese.

O

OH propanoic acid

Which starting materials can be used to produce propanoic acid?

1 CH3CH2CH2OH

2 CH3CH2CHO

3 CH3CH2CN

© UCLES 2017 9701/12/F/M/17 Cambridge International Examinations Cambridge International Advanced Subsidiary and Advanced Level *6447620895*

CHEMISTRY 9701/22

Paper 2 AS Level Structured Questions February/March 2017

1 hour 15 minutes Candidates answer on the Question Paper. Additional Materials: Data Booklet

READ THESE INSTRUCTIONS FIRST

Write your Centre number, candidate number and name on all the work you hand in. Write in dark blue or black pen. You may use an HB pencil for any diagrams or graphs. Do not use staples, paper clips, glue or correction fluid. DO NOT WRITE IN ANY BARCODES.

Answer all questions. Electronic calculators may be used. You may lose marks if you do not show your working or if you do not use appropriate units. A Data Booklet is provided.

At the end of the examination, fasten all your work securely together. The number of marks is given in brackets [ ] at the end of each question or part question.

This document consists of 15 printed pages and 1 blank page.

IB17 03_9701_22/4RP © UCLES 2017 [Turn over 2

Answer all the questions in the spaces provided.

1 (a) The table shows information about some of the elements in the third period.

element Na Mg Al P S Cl atomic radius / nm 0.186 0.160 0.143 0.110 0.104 0.099 radius of most common ion / nm 0.095 0.065 0.050 0.212 0.184 0.181 maximum oxidation number of +1 +7 the element in its compounds

(i) Complete the table to show the maximum oxidation number of each element in its compounds. [1]

(ii) Explain why the atomic radius of elements in the third period decreases from Na to Cl.

......

......

......

......

...... [3]

(iii) The radius of the most common ion of Mg is much smaller than the radius of the most common ion of S.

Identify both ions and explain the difference in their radii.

......

......

...... [2]

© UCLES 2017 9701/22/F/M/17 3

(b) Phosphorus is a non-metal in the third period. It reacts vigorously with excess oxygen but slowly with chlorine.

Some reactions of phosphorus are shown.

excess O2(g) phosphorus A(s) reaction 1

Cl 2(g) reaction 2

water PCl 5(s) B(aq) + HCl (aq)

(i) Write an equation to represent reaction 1, the formation of compound A.

...... [1]

(ii) Give two observations you could make in reaction 2.

1......

2...... [2]

(iii) Name compound B.

...... [1]

© UCLES 2017 9701/22/F/M/17 [Turn over 4

(c) Cerium is a lanthanoid metal that shows similar chemical reactions to some elements in the third period. Most of cerium’s compounds contain Ce3+ or Ce4+ ions.

(i) Cerium shows the same structure and bonding as a typical metal.

Draw a labelled diagram to show the structure and bonding in cerium.

[2]

(ii) Cerium(IV) oxide, CeO2, is a ceramic.

Suggest two physical properties of cerium(IV) oxide.

1......

2...... [2]

© UCLES 2017 9701/22/F/M/17 5

(iii) A naturally occurring sample of cerium contains only four isotopes. Data for three of the isotopes are shown in the table.

isotope 136Ce 138Ce 140Ce 142Ce relative to be 135.907 137.906 139.905 isotopic mass calculated percentage to be 0.185 0.251 88.450 abundance calculated

The Ar of the sample is 140.116.

Use these data to calculate the relative isotopic mass of the fourth isotope in this sample of cerium.

Give your answer to three decimal places.

relative isotopic mass = ...... [3]

[Total: 17]

© UCLES 2017 9701/22/F/M/17 [Turn over 6

2 Hydrogen halides are compounds formed when (Group 17 elements) react with hydrogen. The bond polarity of the hydrogen halides decreases from HF to HI.

Some relevant data are shown in the table.

HF HCl HBr HI boiling point / °C 19 –85 –67 –35 H–X bond energy / kJ mol–1 562 431 366 299

(a) (i) Explain the meaning of the term bond polarity.

......

......

...... [1]

(ii) Suggest why the boiling point of HF is much higher than the boiling points of the other hydrogen halides.

......

......

......

......

...... [2]

(iii) Describe and explain the relative thermal stabilities of the hydrogen halides.

......

......

......

......

......

...... [3]

© UCLES 2017 9701/22/F/M/17 7

(b) The equation for the preparation of hydrogen chloride using concentrated sulfuric acid is shown.

H2SO4 + NaCl NaHSO4 + HCl

(i) Use the Brønsted-Lowry theory of acids and bases to identify the base and its conjugate acid in this reaction. Explain your answer.

Brønsted-Lowry base (base-I) = ......

conjugate acid (acid-II) = ......

......

......

...... [2]

(ii) Explain why the reaction of concentrated sulfuric acid and sodium iodide is not suitable for the preparation of hydrogen iodide.

......

......

......

...... [2]

© UCLES 2017 9701/22/F/M/17 [Turn over 8

(c) Hydrogen chloride undergoes a reversible reaction with oxygen.

4HCl (g) + O2(g) 2Cl 2(g) + 2H2O(g)

The reaction is carried out at 400 °C in the presence of a copper(II) chloride catalyst.

(i) Use the data in the table to calculate the overall enthalpy change of reaction.

enthalpy change of compound formation / kJ mol–1 HCl (g) –92

H2O(g) –242

enthalpy change of reaction = ...... kJ mol–1 [2]

(ii) State the type of catalyst used in this reaction. Explain how a catalyst is able to increase the rate of a chemical reaction.

......

......

......

...... [2]

(iii) The reaction exists in dynamic equilibrium.

The reaction was repeated at 1000 °C and the same pressure.

State and explain the effect on the composition of the equilibrium mixture of the change in temperature.

......

......

......

...... [2]

© UCLES 2017 9701/22/F/M/17 9

(iv) When 1.60 mol of HCl are mixed in a sealed container with 0.500 mol of O2 at 400 °C,

0.600 mol of Cl 2 and 0.600 mol of H2O are formed.

The total pressure inside the container is 1.50 × 105 Pa.

● Calculate the amounts, in mol, of HCl and O2 in the equilibrium mixture.

HCl = ...... mol

O2 = ...... mol

● Calculate the mole fraction of Cl 2 and hence the partial pressure of Cl 2 in the equilibrium mixture.

mole fraction of Cl 2 = ......

p = ...... Pa Cl 2 [3]

© UCLES 2017 9701/22/F/M/17 [Turn over 10

(v) In a separate experiment, an equilibrium reaction mixture was found to contain the four gases at the partial pressures shown in the table.

gas HCl O2 Cl 2 H2O partial pressure / Pa 4.8 × 104 3.0 × 104 3.6 × 104 3.6 × 104

(p )2 × (p )2 Cl 2 H2O Kp = (p )4 × p HCl O2

Use this information and the expression given for Kp to calculate a value for Kp. State the

units of Kp.

Kp = ......

units = ...... [2]

(vi) The reaction is repeated without a catalyst.

State the effect of this on Kp.

...... [1]

[Total: 22]

© UCLES 2017 9701/22/F/M/17 11

3 (a) A series of reactions starting from 1-bromobutane is shown.

HCN and KCN reaction 1 1-bromobutane P CH3(CH2)3CO2H

OH–(aq) reaction 2

polymer but-1-ene Q R

(i) Draw the displayed formula of compound P.

[1]

(ii) Identify the reagent(s) and conditions for reactions 1 and 2.

reaction 1 ......

reaction 2 ...... [2]

(iii) Draw the structure of the repeat unit of polymer Q.

[2]

© UCLES 2017 9701/22/F/M/17 [Turn over 12

(b) Complete the reaction scheme to show the mechanism of the reaction of 1-bromobutane with OH–(aq) to produce R.

Include all necessary charges, dipoles, lone pairs and curly arrows and the structure of R.

CH3CH2CH2

H C Br

H

–OH

[3]

(c) But-1-ene reacts with steam as shown to form a mixture of two structural isomers, S and T.

reflux S U 2– + Cr2O7 / H major product but-1-ene + steam

T

minor product

S can be oxidised with acidified potassium dichromate(VI) to form compound U. S and U both react with alkaline aqueous iodine.

(i) Identify the type of reaction that occurs when but-1-ene reacts with steam.

...... [1]

(ii) State what can be deduced about the structure of S from its reaction with alkaline aqueous iodine.

...... [1]

© UCLES 2017 9701/22/F/M/17 13

(iii) Explain why S is the major product of the reaction of but-1-ene with steam.

......

......

......

......

...... [2]

(iv) Draw the skeletal formulae of S, T and U.

S T

U [3]

(v) Write an equation to represent the oxidation of S to U by acidified potassium dichromate(VI).

You should use [O] to represent the oxidising agent.

...... [1]

© UCLES 2017 9701/22/F/M/17 [Turn over 14

(d) CH3(CH2)3CO2H is a colourless liquid with an unpleasant odour.

It reacts with methanol in the presence of an acid catalyst to produce an organic product V, which has a pleasant fruity smell.

(i) Name V.

...... [1]

(ii) A student analysed CH3(CH2)3CO2H, methanol and V using infra-red spectroscopy. The spectra were returned to the student without labels.

Identify which of the infra-red spectra, X, Y or Z, corresponds to V.

compound CH3(CH2)3CO2H methanol V spectrum

Explain your answer with reference to relevant features of the three spectra in the region above 1500 cm–1.

......

......

......

......

......

...... [4]

© UCLES 2017 9701/22/F/M/17 15

infra-red spectrum X 100

transmittance 50

0 4000 3000 2000 1500 1000 500 wavenumber / cm–1

infra-red spectrum Y 100

transmittance 50

0 4000 3000 2000 1500 1000 500 wavenumber / cm–1

infra-red spectrum Z 100

transmittance 50

0 4000 3000 2000 1500 1000 500 wavenumber / cm–1 [Total: 21]

© UCLES 2017 9701/22/F/M/17 [Turn over

Cambridge International Examinations Cambridge International Advanced Subsidiary and Advanced Level

CHEMISTRY 9701/12

Paper 1 Multiple Choice February/March 2018

1 hour

Additional Materials: Multiple Choice Answer Sheet *8057470051 Soft clean eraser Soft pencil (type B or HB is recommended) Data Booklet

READ THESE INSTRUCTIONS FIRST

Write in soft pencil. Do not use staples, paper clips, glue or correction fluid. * Write your name, Centre number and candidate number on the Answer Sheet in the spaces provided unless this has been done for you. DO NOT WRITE IN ANY BARCODES.

There are forty questions on this paper. Answer all questions. For each question there are four possible answers A, B, C and D. Choose the one you consider correct and record your choice in soft pencil on the separate Answer Sheet.

Read the instructions on the Answer Sheet very carefully.

Each correct answer will score one mark. A mark will not be deducted for a wrong answer. Any rough working should be done in this booklet. Electronic calculators may be used.

This document consists of 12 printed pages.

IB18 03_9701_12/2RP © UCLES 2018 [Turn over

2

Section A

For each question there are four possible answers, A, B, C and D. Choose the one you consider to be correct.

Use of the Data Booklet may be appropriate for some questions.

1 What are the shapes of the molecules of water and boron trifluoride?

H2O BF3

A linear pyramidal B linear trigonal C non-linear pyramidal D non-linear trigonal

2 The electronic configuration of the two outermost shells of an atom is 3s23p63d54s2.

What is this atom?

A manganese B phosphorus C strontium D vanadium

3 Drinking water may contain dissolved calcium hydrogencarbonate, Ca(HCO3)2.

How many electrons are present in a hydrogencarbonate anion?

A 30 B 31 C 32 D 33

4 Which molecule contains a nitrogen atom with sp hybridised orbitals?

A CH3CH2NH2 B HNO3 C HCN D NH3

5 Which mass of solid residue is obtained from the thermal decomposition of 4.10 g of anhydrous calcium nitrate?

A 0.70 g B 1.00 g C 1.40 g D 2.25 g

© UCLES 2018 9701/12/F/M/18 3

6 Sodium hydroxide neutralises acid.

+ – H + OH → H2O

3 + In a 11 000 dm sample of an aqueous solution, the concentration of acid, [H ], is – 3 –3 1.26 × 10 mol dm .

Which mass of solid sodium hydroxide neutralises the acid?

A 0.0214 g B 0.0504 g C 236 g D 554 g

7 The gas laws can be summarised in the ideal gas equation.

pV = nRT

–3 3 0.960 g of oxygen gas is contained in a vessel of volume 7.00 × 10 m at a temperature of 30 °C.

Assume that the gas behaves as an ideal gas.

What is the pressure in the vessel?

A 1.07 kPa B 2.14 kPa C 10.8 kPa D 21.6 kPa

8 Which equation represents the standard enthalpy change of formation of water?

1 A H2(g) + 2 O2(g) → H2O(g)

1 B H2(g) + 2 O2(g) → H2O(l)

C 2H2(g) + O2(g) → 2H2O(g)

D 2H2(g) + O2(g) → 2H2O(l)

9 Hess’ Law and bond energy data can be used to calculate the enthalpy change of a reaction.

Bromoethane, CH3CH2Br, can be made by reacting ethene with hydrogen bromide.

CH2=CH2 + HBr → CH3CH2Br

What is the enthalpy change for this reaction?

–1 A – 674 kJ mol

–1 B – 64 kJ mol

–1 C +186 kJ mol

–1 D +346 kJ mol

© UCLES 2018 9701/12/F/M/18 [Turn over 4

10 Which reaction is not a redox reaction?

A Mg + 2HNO3 → Mg(NO3)2 + H2

B 2Mg(NO3)2 → 2MgO + 4NO2 + O2

C SO2 + NO2 → SO3 + NO

D SO3 + H2O → H2SO4

11 The reaction between sulfur dioxide and oxygen is reversible.

o –1 2SO2 + O2 2SO3 ∆H = –196 kJ mol

Which conditions of pressure and temperature favour the reverse reaction?

pressure temperature

A high high B high low C low high D low low

12 Which statement about the effect of a catalyst on a reversible reaction is correct?

A The activation energy of the forward reaction stays the same. B The composition of the equilibrium mixture stays the same. C The rate of the backward reaction stays the same. D The value of the equilibrium constant changes.

13 Which oxide is insoluble in aqueous sodium hydroxide?

A MgO B Al 2O3 C P4O10 D SO2

14 X, Y and Z are three elements in the third period.

● X reacts with chlorine to give a liquid product. ● Y reacts with chlorine to give a solid product that dissolves in water to give a solution of pH 7. ● Z reacts with chlorine to give a solid product that dissolves in water to give a solution of pH 6.

Which elements are good conductors of electricity?

A X and Y B Y and Z C Y only D Z only

© UCLES 2018 9701/12/F/M/18 5

15 A solution contains both Mg2+(aq) and Sr2+(aq) at the same concentration.

The solution is divided into two equal portions. Aqueous sodium hydroxide is added dropwise to one portion. Dilute sulfuric acid is added dropwise to the other portion.

Which row is correct?

precipitate seen first precipitate seen first

when NaOH(aq) is added when H2SO4(aq) is added

A magnesium hydroxide magnesium sulfate B magnesium hydroxide strontium sulfate C strontium hydroxide magnesium sulfate D strontium hydroxide strontium sulfate

16 The volatility of the Group 17 elements, chlorine, bromine and iodine, decreases down the group.

What is responsible for this?

A bond length in the molecule B bond strength in the halogen molecule C electronegativity of the halogen D number of electrons in the halogen molecule

17 Bromine is extracted from sea-water.

In the final stages of the process two redox reactions take place.

Br2(aq) + SO2(g) + 2H2O(l) → 2HBr(aq) + H2SO4(aq)

2HBr(aq) + Cl 2(g) → Br2(g) + 2HCl (aq)

Which row is correct?

strongest weakest

oxidising agent oxidising agent

A Br2 SO2 Cl 2

B Cl 2 Br2 SO2

C Cl 2 SO2 Br2

D SO2 Br2 Cl 2

© UCLES 2018 9701/12/F/M/18 [Turn over 6

18 When burned, sulfur forms a gaseous product X which can be oxidised to produce a gas Y.

Gas Y reacts with water to produce a product Z.

Which row correctly shows the oxidation states of sulfur in X, Y and Z?

X Y Z

A –2 +4 +4 B –2 +4 +6 C +4 +6 +4 D +4 +6 +6

19 One molecule of ammonia reacts with one molecule of ethyl methanoate, HCO2C2H5, to produce one molecule of methanamide, HCONH2, and only one other molecule, X.

One molecule of methanamide decomposes on heating strongly to produce one molecule of ammonia and only one other molecule, Y.

What could be the identities of X and Y?

X Y

A ethanoic acid carbon monoxide B ethanoic acid hydrogen cyanide C ethanol carbon monoxide D ethanol hydrogen cyanide

20 Which types of stereoisomerism are shown by 2,4-dimethylhex-2-ene?

A both cis-trans isomerism and optical isomerism B cis-trans isomerism only C neither cis-trans isomerism nor optical isomerism D optical isomerism only

© UCLES 2018 9701/12/F/M/18 7

21 An organic ion containing a carbon atom with a negative charge is called a carbanion.

An organic ion containing a carbon atom with a positive charge is called a carbocation.

The reaction between aqueous sodium hydroxide and 1-bromobutane proceeds by an SN2 mechanism.

What is the first step in the mechanism?

A attack by a nucleophile on a carbon atom with a partial positive charge B heterolytic bond fission followed by attack by an electrophile on a carbanion C heterolytic bond fission followed by attack by a nucleophile on a carbocation D homolytic bond fission followed by attack by a nucleophile on a carbocation

22 Compound X can be converted into compound Y in a single step.

OH X O O Y

What could be the identity of X?

A B C D

23 Which compound reacts with 2,4-dinitrophenylhydrazine reagent but does not react with Tollens’ reagent?

A CH3COCO2H

B CH3CH(OH)CHO

C CH3COCHO

D CH3CH(OH)CH3

© UCLES 2018 9701/12/F/M/18 [Turn over 8

24 Structural isomerism and stereoisomerism should be considered when answering this question.

The molecular formula of compound X is C5H12O.

Compound X:

● reacts with alkaline aqueous iodine ● can be dehydrated to form two alkenes only.

What could be the identity of compound X?

A CH3CH2CH(CH3)CH2OH

B CH3CH2CH(OH)CH2CH3

C (CH3)2CHCH(OH)CH3

D CH3CH2CH2CH(OH)CH3

25 Which volume of hydrogen, measured under room conditions, is produced when 0.160 g of methanol reacts with an excess of sodium?

3 3 3 3 A 60 cm B 120 cm C 240 cm D 480 cm

26 Compound X produces a carboxylic acid when heated under reflux with acidified potassium dichromate(VI). Compound X does not react with sodium metal.

What could be the identity of compound X?

A propanal B propanone C propan-1-ol D propan-2-ol

27 A reaction occurs when a sample of 1-chloropropane is heated under reflux with sodium hydroxide dissolved in ethanol.

Which row is correct?

type of reaction name of product

A elimination propan-1-ol B elimination propene C substitution propan-1-ol D substitution propene

© UCLES 2018 9701/12/F/M/18 9

28 Ethanedioic acid has the formula HO2CCO2H.

What is the formula of aluminium ethanedioate?

A Al C2O4 B Al (C2O4)3 C Al 2C2O4 D Al 2(C2O4)3

29 Alcohols, aldehydes and nitriles can each be converted into carboxylic acids.

Which descriptions of their conversions into carboxylic acids are correct?

alcohols aldehydes nitriles

A hydrolysis hydrolysis hydrolysis B hydrolysis hydrolysis oxidation C oxidation oxidation hydrolysis D oxidation oxidation oxidation

30 How many structural isomers with the molecular formula C4H10O give infra-red absorptions both –1 –1 at approximately 1200 cm and at approximately 3400 cm ?

A 2 B 4 C 6 D 7

© UCLES 2018 9701/12/F/M/18 [Turn over 10

Section B

For each of the questions in this section, one or more of the three numbered statements 1 to 3 may be correct.

Decide whether each of the statements is or is not correct (you may find it helpful to put a tick against the statements that you consider to be correct).

The responses A to D should be selected on the basis of

A B C D

1, 2 and 3 1 and 2 2 and 3 1 only are only are only are is correct correct correct correct

No other combination of statements is used as a correct response.

Use of the Data Booklet may be appropriate for some questions.

31 Compound Q contains 40% carbon by mass.

What could Q be?

1 glucose, C6H12O6

2 starch, (C6H10O5)n

3 sucrose, C12H22O11

32 A container is partially filled with hot water, sealed and left to cool.

Which statements are correct?

1 As the temperature decreases, water molecules lose kinetic energy. 2 As the temperature decreases, more water molecules move from vapour to liquid. 3 As the temperature decreases, the vapour pressure of the water decreases.

33 Ammonia and chlorine react together in the gas phase.

8NH3 + 3Cl 2 → N2 + 6NH4Cl

Which statements are correct?

1 Ammonia behaves as a reducing agent. 2 Ammonia behaves as a base. 3 The oxidation number of hydrogen changes.

© UCLES 2018 9701/12/F/M/18 11

34 In which reactions does NH3 behave as a Brønsted-Lowry acid?

– + 1 2NH3 → NH2 + NH4

– 2– + 2 HSO4 + NH3 → SO4 + NH4

+ + 3 Ag + 2NH3 → [Ag(NH3)2]

35 Chlorine reacts with hot aqueous sodium hydroxide.

Which oxidation states does chlorine show in the products of this reaction?

1 –1 2 +3 3 +1

36 In which different forms does nitrogen exist in compounds?

1 bonded by a triple covalent bond 2 as part of a cation 3 in an oxidation state of +5

37 Poly(ethene) and PVC are examples of addition polymers.

Which statements are correct?

1 On combustion, PVC can produce carbon monoxide, carbon dioxide and hydrogen chloride. 2 When poly(ethene) is buried in a landfill site, it will not significantly biodegrade. 3 The empirical formula of an addition polymer is the same as that of the monomer.

38 Organic compound X gives a precipitate when warmed with aqueous silver nitrate. This precipitate dissolves when concentrated aqueous ammonia is added.

What could X be?

1 1-bromopropane 2 2-chlorobutane 3 2-iodo-2-methylpropane

© UCLES 2018 9701/12/F/M/18 [Turn over 12

The responses A to D should be selected on the basis of

A B C D

1, 2 and 3 1 and 2 2 and 3 1 only are only are only are is correct correct correct correct

No other combination of statements is used as a correct response.

39 Propanal reacts with hydrogen cyanide to form 2-hydroxybutanenitrile. A suitable catalyst for this reaction is sodium cyanide.

NaCN

CH3CH2CHO + HCN CH3CH2CH(OH)CN

Which statements about the reaction of propanal with hydrogen cyanide are correct?

1 HCN is a weaker nucleophile than the nucleophile provided by NaCN. 2 The reaction mechanism involves two steps. 3 The product of the reaction has a chiral carbon atom.

40 The structure of lactic acid is shown.

O

H C C 3 CH OH OH

lactic acid

Which esters might form when lactic acid is heated?

1 CH3CH(OH)CO2CH(CH3)CO2H

CH3 CH O O C 2 C O O CH

CH3

3 CH3CH(OH)CO2CH(OH)CH3

To avoid the issue of disclosure of answer-related information to candidates, all copyright acknowledgements are reproduced online in the Cambridge International Examinations Copyright Acknowledgements Booklet. This is produced for each series of examinations and is freely available to download at www.cie.org.uk after the live examination series.

© UCLES 2018 9701/12/F/M/18 Cambridge International Examinations Cambridge International Advanced Subsidiary and Advanced Level *3410012160*

CHEMISTRY 9701/22 Paper 2 AS Level Structured Questions February/March 2018

1 hour 15 minutes Candidates answer on the Question Paper. Additional Materials: Data Booklet

READ THESE INSTRUCTIONS FIRST

Write your Centre number, candidate number and name on all the work you hand in. Write in dark blue or black pen. You may use an HB pencil for any diagrams or graphs. Do not use staples, paper clips, glue or correction fluid. DO NOT WRITE IN ANY BARCODES.

Answer all questions. Electronic calculators may be used. You may lose marks if you do not show your working or if you do not use appropriate units. A Data Booklet is provided.

At the end of the examination, fasten all your work securely together. The number of marks is given in brackets [ ] at the end of each question or part question.

This document consists of 11 printed pages and 1 blank page.

IB18 03_9701_22/3RP © UCLES 2018 [Turn over 2

Answer all the questions in the spaces provided.

1 (a) The graph shows a sketch of the first ionisation energies of six successive elements in the Periodic Table.

The letters are not the symbols of the elements.

first ionisation energy

A B C D E F element

(i) Explain what is meant by the term first ionisation energy.

......

......

......

...... [3]

(ii) Suggest why the first ionisation energy of B is much less than that of A.

......

......

......

......

...... [3]

© UCLES 2018 9701/22/F/M/18 3

(b) P–T are successive elements in Period 3 of the Periodic Table.

The letters are not the symbols of the elements.

On the axes, sketch a graph to show the trend in the atomic radius of the elements P–T.

Explain your answer.

atomic radius of the element

PQR ST element

explanation ......

......

......

......

...... [3]

[Total: 9]

© UCLES 2018 9701/22/F/M/18 [Turn over 4

2 Carbon and silicon are elements in Group 14.

(a) C60 and diamond are allotropes of carbon.

(i) Describe the lattice structure of solid C60.

......

......

......

...... [2]

(ii) C60 sublimes (turns directly from solid to gas) at about 800 K. Diamond also sublimes but only above 3800 K.

Explain why C60 and diamond sublime at such different temperatures.

......

......

......

......

......

......

......

...... [4]

(b) C60 forms with similar chemical properties to those of alkenes. One such

is C60H18.

(i) State what is meant by the term hydrocarbon.

......

...... [1]

© UCLES 2018 9701/22/F/M/18 5

(ii) Describe a test to indicate the presence of double bonds between carbon atoms in C60H18. Give the result of the test.

test ......

......

result ......

...... [2]

3 5 (c) 0.144 g of C60 was placed in a 100 cm container of hydrogen gas at 20 °C and 1.00 × 10 Pa.

The container was heated to make the C60 and hydrogen gas react.

The reaction occurred as shown in the equation.

C60(s) + xH2(g) C60H2x(s)

After the reaction, the container was allowed to cool to 20 °C. The pressure decreased to 4 2.21 × 10 Pa. All of the C60 had reacted.

(i) Name the type of reaction that occurred.

...... [1]

(ii) Calculate the amount, in moles, of C60 that reacted.

amount of C60 = ...... mol [1]

(iii) Calculate the amount, in moles, of hydrogen gas that reacted with the C60.

amount of hydrogen gas = ...... mol [2]

(iv) Use your answers from (ii) and (iii) to deduce the molecular formula of the hydrocarbon,

C60H2x.

If you were unable to calculate the amount of hydrogen gas, assume that 0.00240 mol of hydrogen gas reacted. This is not the correct value for the amount of hydrogen gas that reacted.

molecular formula = ...... [2]

© UCLES 2018 9701/22/F/M/18 [Turn over 6

(d) Silicon shows the same kind of bonding and structure as diamond.

(i) State the type of bonding and structure shown by silicon.

......

......

...... [2]

4– (ii) When silicon reacts with magnesium, Mg2Si forms. Mg2Si is thought to contain the Si ion.

State the full electronic configuration of the Si4– ion.

1s2...... [1]

(iii) Solid Mg2Si reacts with dilute hydrochloric acid to form gaseous SiH4 and a solution of magnesium chloride.

Write an equation to show the reaction of solid Mg2Si with dilute hydrochloric acid.

Include state symbols.

...... [2]

(iv) Predict the shape of the SiH4 molecule.

...... [1]

(v) SiH4 reacts spontaneously with oxygen to produce a white solid and a colourless liquid that turns anhydrous copper(II) sulfate blue. No other products are formed.

Write an equation for the reaction of SiH4 with oxygen.

State symbols are not required.

...... [1]

[Total: 22]

© UCLES 2018 9701/22/F/M/18 7

3 Calcium and its compounds have a large variety of applications.

(a) Calcium metal reacts readily with most acids.

(i) Write an equation for the reaction of calcium with dilute . State symbols are not required.

...... [1]

(ii) When calcium metal is placed in dilute sulfuric acid, it reacts vigorously at first.

After a short time, a crust of calcium sulfate forms on the calcium metal and the reaction stops. Some of the calcium metal and dilute sulfuric acid remain unreacted.

Suggest an explanation for these observations.

......

......

......

...... [2]

(b) Calcium ethanedioate is formed when calcium reacts with ethanedioic acid, (CO2H)2. The compound contains one cation and one anion.

(i) Draw the ‘dot-and-cross’ diagram of the cation present in calcium ethanedioate. Show all electrons.

[1]

(ii) Draw the displayed formula of the anion present in calcium ethanedioate.

[2]

© UCLES 2018 9701/22/F/M/18 [Turn over 8

(c) Calcium chlorate(I), Ca(Cl O) 2, is used as an alternative to sodium chlorate(I), NaCl O, in some household products.

(i) Suggest a use for calcium chlorate(I).

...... [1]

(ii) The chlorate(I) ion is formed when cold aqueous sodium hydroxide reacts with chlorine.

Write an ionic equation for this reaction. State symbols are not required.

...... [1]

(iii) The chlorate(I) ion is unstable and decomposes when heated as shown.

Deduce the oxidation number of chlorine in each species. Complete the boxes.

– – – 3Cl O 2Cl + Cl O3 oxidation number +1 of chlorine: [1]

(iv) In terms of electron transfer, state what happens to the chlorine in the reaction in (iii).

...... [1]

(d) Calcium lactate is used in some medicines. It forms when lactic acid (2-hydroxypropanoic acid) reacts with calcium carbonate.

OH

H3C C H

CO2H lactic acid

(i) Identify the two other products of the reaction of lactic acid with calcium carbonate.

...... [1]

© UCLES 2018 9701/22/F/M/18 9

Two possible methods of making lactic acid are shown.

O OH reaction 1 C H3C CH H3C H reaction 2 CN OH

H3C CH

CO2H OH O lactic acid reaction 3 reaction 4 H3C CH C H3C CO2H CH2OH

(ii) State suitable reagents and conditions for reactions 1 and 3.

reaction reagents and conditions

1

3

[4]

(iii) Name the type of reaction that occurs in reaction 2.

...... [1]

(iv) Reaction 4 uses NaBH4.

Identify the role of NaBH4 in this reaction.

...... [1]

(v) Lactic acid has a chiral centre.

State what is meant by the term chiral centre.

......

......

...... [1]

[Total: 18]

© UCLES 2018 9701/22/F/M/18 [Turn over 10

4 Cyclohexane is a colourless liquid used in industry to produce synthetic fibres.

A reaction scheme involving cyclohexane is shown.

Cl reaction 1 reaction 2

(a) Reaction 1 involves a free radical substitution mechanism.

(i) State the essential condition required for reaction 1 to occur.

...... [1]

(ii) Complete the table to give details of the mechanism in reaction 1.

name of step reaction

l l ...... C 2 2C •

• propagation + Cl •+ ......

Cl •

...... + Cl 2 + Cl •

• termination + Cl • ......

[4]

(b) Name the type of reaction that occurs in reaction 2.

...... [1]

© UCLES 2018 9701/22/F/M/18 11

(c) The product of reaction 2 is cyclohexene.

Cyclohexene can be converted into adipic acid (hexanedioic acid), HO2C(CH2)4CO2H.

(i) Identify the reagents and conditions for the conversion of cyclohexene into adipic acid.

......

...... [2]

(ii) Suggest three main differences between the infra-red spectra of cyclohexene and adipic acid.

In each case, identify the bond responsible and its characteristic absorption range (in wavenumbers).

1 ......

......

......

2 ......

......

......

3 ......

......

...... [3]

[Total: 11]

© UCLES 2018 9701/22/F/M/18 [Turn over

UNIVERSITY OF CAMBRIDGE INTERNATIONAL EXAMINATIONS General Certificate of Education Advanced Subsidiary Level and Advanced Level

CHEMISTRY 9701/11

Paper 1 Multiple Choice May/June 2013

1 hour

Additional Materials: Multiple Choice Answer Sheet *7900784217* Soft clean eraser Soft pencil (type B or HB is recommended) Data Booklet

READ THESE INSTRUCTIONS FIRST

Write in soft pencil. Do not use staples, paper clips, highlighters, glue or correction fluid. Write your name, Centre number and candidate number on the Answer Sheet in the spaces provided unless this has been done for you. DO NOT WRITE IN ANY BARCODES.

There are forty questions on this paper. Answer all questions. For each question there are four possible answers A, B, C and D. Choose the one you consider correct and record your choice in soft pencil on the separate Answer Sheet.

Read the instructions on the Answer Sheet very carefully.

Each correct answer will score one mark. A mark will not be deducted for a wrong answer. Any rough working should be done in this booklet. Electronic calculators may be used.

This document consists of 14 printed pages and 2 blank pages.

IB13 06_9701_11/5RP © UCLES 2013 [Turn over

2

Section A

For each question there are four possible answers, A, B, C, and D. Choose the one you consider to be correct.

1 Solutions containing chlorate(I) ions are used as household bleaches and disinfectants. These solutions decompose on heating as shown.

– – – 3Cl O → Cl O3 + 2Cl

Which oxidation state is shown by chlorine in each of these three ions?

– – – Cl O Cl O3 Cl

A +1 +3 –1 B –1 +3 +1 C +1 +5 –1 D –1 +5 +1

3 3 2 A mixture of 10 cm of methane and 10 cm of ethane was sparked with an excess of oxygen. After cooling to room temperature, the residual gas was passed through aqueous potassium hydroxide.

All gas volumes were measured at the same temperature and pressure.

What volume of gas was absorbed by the alkali?

3 3 3 3 A 15 cm B 20 cm C 30 cm D 40 cm

3 The diagram shows an electrolytic cell for the extraction of aluminium.

graphite anode

l A 2O3 in molten cryolite

graphite lining acting as cathode

molten aluminium

Which statement is correct?

A Aluminium ions are oxidised in this process.

3+ – B Aluminium is liberated at the anode by the reaction Al + 3e → Al. C Cryolite is purified aluminium oxide. D The graphite anode burns away.

© UCLES 2013 9701/11/M/J/13 3

4 Use of the Data Booklet is relevant to this question.

The elements radon (Rn), francium (Fr) and radium (Ra) have proton numbers 86, 87 and 88 respectively.

What is the order of their first ionisation energies?

least most endothermic endothermic

A Fr Ra Rn B Fr Rn Ra C Ra Fr Rn D Rn Ra Fr

5 In which species are the numbers of protons, neutrons and electrons all different?

27 35 − 32 2 − 39 + A 13 Al B 17 Cl C 16 S D 19K

6 An experiment is set up to measure the rate of hydrolysis of ethyl ethanoate.

CH3CO2C2H5 + H2O CH3CO2H + C2H5OH

The hydrolysis is found to be slow in neutral aqueous solution but it proceeds at a measurable rate when the solution is acidified with hydrochloric acid.

What is the function of the hydrochloric acid?

A to dissolve the ethyl ethanoate B to ensure that the reaction reaches equilibrium C to increase the reaction rate by catalytic action D to suppress ionisation of the ethanoic acid formed

© UCLES 2013 9701/11/M/J/13 [Turn over 4

7 The reaction pathway for a reversible reaction is shown below.

30 energy / kJ mol–1 50

extent of reaction

Which statement is correct?

–1 A The activation energy of the reverse reaction is +80 kJ mol .

–1 B The enthalpy change for the forward reaction is +30 kJ mol .

–1 C The enthalpy change for the forward reaction is +50 kJ mol .

–1 D The enthalpy change for the reverse reaction is +30 kJ mol .

8 Why does the rate of a gaseous reaction increase when the pressure is increased at a constant temperature?

A More particles have energy that exceeds the activation energy. B The particles have more space in which to move. C The particles move faster. D There are more frequent collisions between particles.

9 Which would behave the least like an ideal gas at room temperature?

A carbon dioxide B helium C hydrogen D nitrogen

10 The general gas equation can be used to calculate the Mr value of a gas.

For a sample of a gas of mass m g, which expression will give the value of Mr? mpV pVRT mRT pV A M = B M = C M = D M = r RT r m r pV r mRT

© UCLES 2013 9701/11/M/J/13 5

2+ – 2+ 2+ 11 A solution of Sn ions will reduce an acidified solution of MnO4 ions to Mn ions. The Sn ions are oxidised to Sn4+ ions in this reaction.

2+ How many moles of Mn ions are formed when a solution containing 9.5 g of SnCl 2 (Mr: 190) is added to an excess of acidified KMnO4 solution?

A 0.010 B 0.020 C 0.050 D 0.125

12 Use of the Data Booklet is relevant to this question.

This question should be answered using bond enthalpy data. The equation for the complete combustion of methane is given below.

CH4 + 2O2 → CO2 + 2H2O

What is the enthalpy change of combustion of methane?

–1 A –1530 kJ mol

–1 B –1184 kJ mol

–1 C –770 kJ mol

–1 D –688 kJ mol

13 In which row of the table are all statements comparing the compounds of magnesium and barium correct?

solubility of hydroxides solubility of sulfates

solubility of solubility of barium solubility of solubility of barium magnesium hydroxide magnesium sulfate sulfate hydroxide

A higher lower higher lower B higher lower lower higher C lower higher higher lower D lower higher lower higher

14 What happens when iodine solution is added to a solution of sodium bromide?

A A reaction occurs without changes in oxidation state. B Bromide ions are oxidised, iodine atoms are reduced. C Bromide ions are reduced, iodine atoms are oxidised. D No reaction occurs.

© UCLES 2013 9701/11/M/J/13 [Turn over 6

15 Element 85, , is in Group VII. Concentrated sulfuric acid is added to sodium astatide. The mixture of products includes astatine, hydrogen astatide, , and sodium sulfate.

Which product is formed by the oxidation of one of the constituents of sodium astatide?

A astatine B hydrogen astatide C hydrogen sulfide D sodium sulfate

16 Use of the Data Booklet is relevant to this question.

Magnesium nitrate, Mg(NO3)2, will decompose when heated to give a white solid and a mixture of gases. One of the gases released is an oxide of nitrogen, X.

7.4 g of anhydrous magnesium nitrate is heated until no further reaction takes place.

What mass of X is produced?

A 1.5 g B 2.3 g C 3.0 g D 4.6 g

17 Y is a salt of one of the halogens chlorine, bromine, iodine, or astatine (element 85).

The reaction scheme shows a series of reactions using a solution of Y as the starting reagent.

HNO3(aq) an excess of a colourless Y(aq) a precipitate solution AgNO3(aq) dilute NH3(aq)

an excess of

HNO3(aq)

a precipitate

What could Y be?

A sodium chloride B sodium bromide C potassium iodide D potassium astatide

© UCLES 2013 9701/11/M/J/13 7

18 Sulfur trioxide is manufactured from sulfur dioxide and oxygen, using the Contact process.

Which condition affects the value of the equilibrium constant, Kc?

A adjusting the temperature B increasing the pressure

C removing SO3 from the equilibrium mixture D using a catalyst

19 Which reagent, when mixed and heated with ammonium sulfate, liberates ammonia?

A aqueous bromine B dilute hydrochloric acid C limewater

D potassium dichromate(VI) in acidic solution

20 The following compounds are found in the seaweed Asparagopsis taxiformis.

Which compound could show both cis-trans isomerism and optical isomerism?

ABCD l l Br l C H C H O C H Br Br C Br C Br CC Br C I C C Br C C Br C C Cl C C I l Br H H C H Br H

21 Lactic acid (2-hydroxypropanoic acid), CH3CH(OH)CO2H, is found in sour milk.

Which reaction could occur with lactic acid?

A CH3CH(OH)CO2H + CH3OH → CH3CH(OCH3)CO2H + H2O

B CH3CH(OH)CO2H + HCO2H → CH3CH(O2CH)CO2H + H2O

C CH3CH(OH)CO2H + NaHCO3 → CH3CH(ONa)CO2H + H2O + CO2

D CH3CH(OH)CO2H + Cl 2 → CH3CH(Cl )CO2H + HOCl

© UCLES 2013 9701/11/M/J/13 [Turn over 8

22 Bromine reacts with ethene to form 1,2-dibromoethane.

What is the correct description of the organic intermediate in this reaction?

A It has a negative charge. B It is a free radical. C It is a nucleophile. D It is an electrophile.

23 Chloroethane can be used to make sodium propanoate.

chloroethane → Q → sodium propanoate

The intermediate, Q, is hydrolysed with boiling aqueous sodium hydroxide, to give sodium propanoate.

Which reagent would produce the intermediate, Q, from chloroethane?

A concentrated ammonia solution B dilute sulfuric acid C hydrogen cyanide D potassium cyanide

24 Aqueous sodium hydroxide reacts with 1-bromopropane to give propan-1-ol.

How should the first step in the mechanism be described?

A by a curly arrow from a lone pair on the OH– ion to the Cδ+ atom of 1-bromopropane

B by a curly arrow from the Cδ+ atom of 1-bromopropane to the OH– ion C by a curly arrow from the C–Br bond to the C atom D by the homolytic fission of the C–Br bond

25 , C5H12, is reacted with chlorine in the presence of ultraviolet light. A compound R is found in the products. R has molecular formula C5H10Cl 2. Each molecule of R contains one chiral carbon atom.

Which two atoms of the pentane chain could be bonded to chlorine atoms in this isomer?

A 1 and 3 B 1 and 5 C 2 and 3 D 2 and 4

© UCLES 2013 9701/11/M/J/13 9

26 Use of the Data Booklet is relevant to this question.

2.30 g of ethanol were mixed with an excess of aqueous acidified potassium dichromate(VI). The reaction mixture was then boiled under reflux for one hour. The desired organic product was then collected by distillation. The yield of product was 60.0 %.

What mass of product was collected?

A 1.32 g B 1.38 g C 1.80 g D 3.20 g

27 Burnt sugar has a characteristic smell caused partly by the following compound.

O

CH C CHO 3

This compound contains two functional groups.

Which reagent will react with only one of the functional groups?

A acidified potassium dichromate(VI) B 2,4-dinitrophenylhydrazine C hydrogen cyanide D sodium hydroxide

28 The structural formula of a compound X is shown below.

O

CH3CH2CH2OC

H

What is the name of compound X and how does its boiling point compare with that of butanoic acid?

name of X boiling point of X

A methyl propanoate higher B methyl propanoate lower C propyl methanoate higher D propyl methanoate lower

© UCLES 2013 9701/11/M/J/13 [Turn over 10

29 Synthetic resins, plasticisers and many other chemicals can be made by polymerisation of a variety of monomers including prop-2-en-1-ol, CH2=CHCH2OH.

Which structure represents the repeat unit in poly(prop-2-en-1-ol)?

A CH2 CH CH2 OH

B CH2 CH

CH2OH

C CH C

CH2OH

D CH2 CH CH2

OH

30 Some vegetable oils contain ‘trans fats’ that are associated with undesirable increases in the amount of cholesterol in the blood.

In the diagrams below, R1 and R2 are different hydrocarbon chains.

Which diagram correctly illustrates an optically active ‘trans fat’?

A B

H R1CO2CH2 H R1CO2CH2

CH3(CH2)6C C(CH2)7CO2CH CH3(CH2)6C C(CH2)7CO2CH

H R1CO2CH2 H R2CO2CH2

C D

H H R1CO2CH2 H H R1CO2CH2

CH3(CH2)6C C(CH2)7CO2CH CH3(CH2)6C C(CH2)7CO2CH

R CO CH R CO CH 1 2 2 2 2 2

© UCLES 2013 9701/11/M/J/13 11

Section B

For each of the questions in this section, one or more of the three numbered statements 1 to 3 may be correct.

Decide whether each of the statements is or is not correct (you may find it helpful to put a tick against the statements that you consider to be correct).

The responses A to D should be selected on the basis of

A B C D

1, 2 and 3 1 and 2 2 and 3 1 only are only are only are is correct correct correct correct

No other combination of statements is used as a correct response.

31 Use of the Data Booklet is relevant to this question.

Free-radicals play an important part in reactions involving the destruction of the ozone layer and the substitution of by chlorine.

Some free-radicals contain two unpaired electrons. Such species are called diradicals.

Which species are diradicals?

1 O 2 Cl

3 CH3

32 The Group II metals have higher melting points than the Group I metals.

Which factors could contribute towards the higher melting points?

1 There are smaller interatomic distances in the metallic lattices of the Group II metals. 2 More electrons are available from each Group II metal atom for bonding the atom into the metallic lattice. 3 Group II metals have a higher first ionisation energy than the corresponding Group I metal.

33 Valence shell electron pair repulsion theory should be used to answer this question.

Which species are trigonal planar?

1 BH3

+ 2 CH3

3 PH3

© UCLES 2013 9701/11/M/J/13 [Turn over 12

The responses A to D should be selected on the basis of

A B C D

1, 2 and 3 1 and 2 2 and 3 1 only are only are only are is correct correct correct correct

No other combination of statements is used as a correct response.

34 The first ionisation energies of twenty successive elements in the Periodic Table are represented in the graph.

The letters given are not the normal symbols for these elements.

2500

2000

1500 first ionisation _ energy / kJ mol 1 1000

500

0 ABCD E FGH I J K LMNO PQRS T

element

Which statements about this graph are correct?

1 Elements B, J and R are in Group 0 of the Periodic Table. 2 Atoms of elements D and L contain two electrons in their outer shells. 3 Atoms of elements G and O contain a half-filled p subshell.

© UCLES 2013 9701/11/M/J/13 13

35 Solids W, X, Y and Z are compounds of two different Group II metals. Some of their applications are described below.

Compound W is used as a refractory lining material in kilns.

Compound X is used as a building material. It can also be heated in a kiln to form compound Y. When Y is hydrated, it forms compound Z which is used agriculturally to treat soils.

Which statements about these compounds are correct?

1 More acid is neutralised by 1 g of W than by 1 g of X. 2 The metallic element in W reacts with water more quickly than the metallic element in Y. 3 Adding Z to a soil decreases the pH of the soil.

36 When a red-hot platinum wire is plunged into a test tube of hydrogen iodide, the gas is decomposed into its elements. If the experiment is repeated with hydrogen chloride, no change occurs.

Which factors contribute to this behaviour?

1 the strength of the hydrogen-halogen bond 2 the size of the halogen atom 3 the standard enthalpy of formation, , of each of the products of decomposition

37 Which molecules would be present in the mixture produced by the photochemical chlorination of methane?

1 hydrogen 2 hydrogen chloride 3 dichloromethane

38 In which reactions is the organic compound oxidised by the given reagent?

1 CH3CH2CHO + Fehling’s reagent

2 CH3CH2CH2CHO + Tollens’ reagent

3 CH3CHO + 2,4-dinitrophenylhydrazine reagent

© UCLES 2013 9701/11/M/J/13 [Turn over 14

The responses A to D should be selected on the basis of

A B C D

1, 2 and 3 1 and 2 2 and 3 1 only are only are only are is correct correct correct correct

No other combination of statements is used as a correct response.

39 A sun protection cream contains the following ester as its active ingredient.

CH2CH3

CH3O CH CHCO2CH2CH CH CH CH CH 2 2 2 3

Which substances are present in the products of its hydrolysis by aqueous sodium hydroxide?

1 CH3CH2CH2CH2CH(CH2CH3)CH2OH

2 – + CH3O CH CHCO2 Na

3 – + CH3O CO2 Na

40 Which reagents, when used in an excess, can be used to make sodium lactate, CH3CH(OH)CO2Na, from lactic acid, CH3CH(OH)CO2H?

1 Na

2 NaHCO3 3 NaOH

© UCLES 2013 9701/11/M/J/13 UNIVERSITY OF CAMBRIDGE INTERNATIONAL EXAMINATIONS General Certifi cate of Education Advanced Subsidiary Level and Advanced Level *7635733974*

CHEMISTRY 9701/21

Paper 2 Structured Questions AS Core May/June 2013

1 hour 15 minutes Candidates answer on the Question Paper. Additional Materials: Data Booklet

READ THESE INSTRUCTIONS FIRST

Write your Centre number, candidate number and name on all the work you hand in. Write in dark blue or black pen. You may use a soft pencil for any diagrams, graphs or rough working. Do not use staples, paper clips, highlighters, glue or correction fl uid. DO NOT WRITE IN ANY BARCODES.

Answer all questions. Electronic calculators may be used. You may lose marks if you do not show your working or if you do not use appropriate units. A Data Booklet is provided.

At the end of the examination, fasten all your work securely together. The number of marks is given in brackets [ ] at the end of each question or part question.

For Examiner’s Use

1

2

3

4

5

Total

This document consists of 10 printed pages and 2 blank pages.

IB13 06_9701_21/2RP © UCLES 2013 [Turn over 2

For Answer all the questions in the spaces provided. Examiner’s Use

1 A sample of a fertiliser was known to contain ammonium sulfate, (NH4)2SO4, and sand only.

3 A 2.96 g sample of the solid fertiliser was heated with 40.0 cm of NaOH(aq), an excess, and all of the ammonia produced was boiled away.

3 –3 After cooling, the remaining NaOH(aq) was exactly neutralised by 29.5 cm of 2.00 mol dm HCl.

3 In a separate experiment, 40.0 cm of the original NaOH(aq) was exactly neutralised by 3 –3 39.2 cm of the 2.00 mol dm HCl.

(a) (i) Write balanced equations for the following reactions.

NaOH with HCl

......

(NH4)2SO4 with NaOH

......

3 (ii) Calculate the amount, in moles, of NaOH present in the 40.0 cm of the original 3 –3 NaOH(aq) that was neutralised by 39.2 cm of 2.00 mol dm HCl.

3 (iii) Calculate the amount, in moles, of NaOH present in the 40.0 cm of NaOH(aq) that

remained after boiling the (NH4)2SO4.

(iv) Use your answers to (ii) and (iii) to calculate the amount, in moles, of NaOH that

reacted with the (NH4)2SO4.

© UCLES 2013 9701/21/M/J/13 3

For (v) Use your answers to (i) and (iv) to calculate the amount, in moles, of (NH4)2SO4 that Examiner’s reacted with the NaOH. Use

(vi) Hence calculate the mass of (NH4)2SO4 that reacted.

(vii) Use your answer to (vi) to calculate the percentage, by mass, of (NH4)2SO4 present in the fertiliser. Write your answer to a suitable number of signifi cant fi gures.

[9]

(b) The uncontrolled use of nitrogenous fertilisers can cause environmental damage to lakes and streams. This is known as eutrophication.

What are the processes that occur when excessive amounts of nitrogenous fertilisers get into lakes and streams?

......

......

...... [2]

(c) Large quantities of ammonia are manufactured by the Haber process. Not all of this ammonia is used to make fertilisers. State one large-scale use for ammonia, other than in the production of nitrogenous fertilisers.

...... [1]

[Total: 12]

© UCLES 2013 9701/21/M/J/13 [Turn over 4

For 2 Ammonium nitrate fertiliser is manufactured from ammonia. The fi rst reaction in the Examiner’s manufacture of the fertiliser is the catalytic oxidation of ammonia to form nitrogen monoxide, Use 3 NO. This is carried out at about 1 × 10 kPa (10 atmospheres) pressure and a temperature of 700 to 850 °C.

–1 4NH3(g) + 5O2(g) 4NO(g) + 6H2O(g) ∆H = –906 kJ mol

(a) Write the expression for the equilibrium constant, Kp, stating the units.

Kp =

units ...... [2]

(b) What will be the effect on the yield of NO of each of the following? In each case, explain your answer.

(i) increasing the temperature

......

......

......

(ii) decreasing the applied pressure

......

......

...... [4]

© UCLES 2013 9701/21/M/J/13 5

For (c) The standard enthalpy changes of formation of NH3(g) and H2O(g) are as follows. Examiner’s Use –1 –1 NH3(g), ∆Hf = –46.0 kJ mol H2O(g), ∆Hf = –242 kJ mol

Use these data and the value of given below to calculate the standard enthalpy change of formation of NO(g). Include a sign in your answer.

–1 4NH3(g) + 5O2(g) 4NO(g) + 6H2O(g) ∆H = –906 kJ mol

[4]

[Total: 10]

© UCLES 2013 9701/21/M/J/13 [Turn over 6

For 3 This question refers to the elements in the section of the Periodic Table shown below. Examiner’s Use HHe Li Be B C N O F Ne Na Mg Al Si P S Cl Ar K Ca ...... transition elements ...... Ga Ge As Se Br Kr

(a) From this list of elements, identify in each case one element that has the property described. Give the symbol of the element.

(i) An element that fl oats on cold water and reacts readily with it.

......

(ii) An element that forms an oxide that is a reducing agent.

......

(iii) The element that has the smallest fi rst ionisation energy.

......

(iv) The element which has a giant molecular structure and forms an oxide which has a simple molecular structure.

......

(v) The element in Period 3 (Na to Ar) that has the smallest anion.

......

(vi) The element in Period 3 (Na to Ar) which forms a chloride with a low melting point and an oxide with a very high melting point.

...... [6]

© UCLES 2013 9701/21/M/J/13 7

For (b) Use the elements in Period 3 (Na to Ar) in the section of the Periodic Table opposite to Examiner’s identify the oxide(s) referred to below. Use In each case, give the formula of the oxide(s).

(i) An oxide which when placed in water for a long time has no reaction with it.

......

(ii) An oxide which dissolves readily in water to give a strongly alkaline solution.

......

(iii) Two acidic oxides formed by the same element.

...... and ......

(iv) An oxide which is amphoteric.

...... [5]

(c) Fluorine reacts with other elements in Group VII to form a number of different compounds. Two such compounds and their boiling points are given in the table.

compound Cl F3 BrF3

boiling point / °C 12 127

(i) The two molecules have similar electronic confi gurations.

Showing outer electrons only, draw a 'dot-and-cross' diagram of the bonding in Cl F3.

(ii) The two molecules have the same shape. Suggest why the boiling points are signifi cantly different.

......

......

......

...... [4]

[Total: 15]

© UCLES 2013 9701/21/M/J/13 [Turn over 9

For 4 Organic chemistry is the chemistry of carbon compounds. The types of organic reactions that Examiner’s you have studied are listed below. Use

addition elimination hydrolysis

oxidation reduction substitution

Addition and substitution reactions are further described as follows.

electrophilic nucleophilic free radical

Complete the table below. Fill in the central column by using only the types of reaction given in the lists above. Use both lists when appropriate. In the right hand column give the formula(e) of the reagent(s) you would use to carry out the reaction given.

organic reaction type of reaction reagent(s)

CH3CH2CH2CH2Br →

CH3CH2CH2CH2NH2

CH3CH2CH2CH2OH →

BrCH2CH2CH2CH2OH

CH3COCH3 →

CH3C(OH)(CN)CH3

CH3CH(OH)CH2CH3 →

CH3CH=CHCH3

[Total: 11]

© UCLES 2013 9701/21/M/J/13 [Turn over 10

For 5 Crotonaldehyde, CH3CH=CHCHO, occurs in soybean oils. Examiner’s Use (a) In the boxes below, write the structural formula of the organic compound formed when crotonaldehyde is reacted separately with each reagent under suitable conditions. If you think no reaction occurs, write 'NO REACTION' in the box.

reaction reagent product

Br in an inert organic A 2 solvent

BPCl 3

CH2 and Ni catalyst

D NaBH4

+ EK2Cr2O7 / H

[5]

(b) Crotonaldehyde exists in more than one stereoisomeric form. Draw the displayed formulae of the stereoisomers of crotonaldehyde. Label each isomer.

[3]

© UCLES 2013 9701/21/M/J/13 11

For (c) Draw the skeletal formula of crotonaldehyde. Examiner’s Use

[1]

(d) The product of reaction E in the table opposite will react with a solution containing acidifi ed manganate(VII) ions. Draw the structural formulae of the organic products when the reagent is

(i) cold, dilute;

(ii) hot, concentrated.

[3]

[Total: 12]

© UCLES 2013 9701/21/M/J/13 [Turn over

Cambridge International Examinations Cambridge International Advanced Subsidiary and Advanced Level

CHEMISTRY 9701/11

Paper 1 Multiple Choice May/June 2014

1 hour

Additional Materials: Multiple Choice Answer Sheet *4073847616* Soft clean eraser Soft pencil (type B or HB is recommended) Data Booklet

READ THESE INSTRUCTIONS FIRST

Write in soft pencil. Do not use staples, paper clips, glue or correction fluid. Write your name, Centre number and candidate number on the Answer Sheet in the spaces provided unless this has been done for you. DO NOT WRITE IN ANY BARCODES.

There are forty questions on this paper. Answer all questions. For each question there are four possible answers A, B, C and D. Choose the one you consider correct and record your choice in soft pencil on the separate Answer Sheet.

Read the instructions on the Answer Sheet very carefully.

Each correct answer will score one mark. A mark will not be deducted for a wrong answer. Any rough working should be done in this booklet. Electronic calculators may be used.

This document consists of 14 printed pages and 2 blank pages.

IB14 06_9701_11/3RP © UCLES 2014 [Turn over

2

Section A

For each question there are four possible answers, A, B, C, and D. Choose the one you consider to be correct.

1 Use of the Data Booklet is relevant to this question.

Atoms of element X have six unpaired electrons.

What could be element X?

A carbon B chromium C iron D selenium

2 Use of the Data Booklet is relevant to this question.

Iodine is a black, shiny, non-metallic solid and a member of Group VII. It sublimes easily on heating to give a purple vapour.

3 A sample of iodine vapour of mass 6.35 g has a volume of 1.247 dm when maintained at 5 constant temperature and a pressure of 1.00 × 10 Pa.

If iodine vapour acts as an ideal gas, what is the temperature of the iodine vapour?

A 300 K B 600 K C 300 000 K D 600 000 K

3 Enthalpy changes that are difficult to measure directly can often be determined using Hess’ Law to construct an enthalpy cycle.

Which enthalpy change is indicated by X in the enthalpy cycle shown?

C(s) + 2H2(g) + 2O2(g)

X

CH (g) + 2O (g) CO (g) + 2H O(l) 4 2 2 2

A – 4 × the enthalpy of combustion of hydrogen

B + 4 × the enthalpy of combustion of hydrogen

C – 2 × the enthalpy of formation of water

D + 2 × the enthalpy of formation of water

© UCLES 2014 9701/11/M/J/14 3

4 The table shows the physical properties of four substances.

Which substance has a giant covalent structure?

electrical electrical electrical melting point boiling point conductivity conductivity conductivity / °C / °C of aqueous of solid of liquid solution

A –119 39 poor poor insoluble B –115 –85 poor poor good C 993 1695 poor good good D 1610 2230 poor poor insoluble

3 –3 3 5 A student mixed 25.0 cm of 0.350 mol dm sodium hydroxide solution with 25.0 cm of –3 0.350 mol dm hydrochloric acid. The temperature rose by 2.50 °C. Assume that no heat was lost to the surroundings.

–3 –1 The final mixture had a specific heat capacity of 4.20 J cm K .

What is the molar enthalpy change for the reaction?

–1 A –150 kJ mol

–1 B –60.0 kJ mol

–1 C –30.0 kJ mol

–1 D –0.150 kJ mol

6 Al Cl 3 vapour forms molecules with formula Al 2Cl 6 as it is cooled.

What happens to the bond angles during the change from Al Cl 3 to Al 2Cl 6?

A Some decrease, some remain the same. B Some increase, some remain the same. C They all decrease. D They all increase.

© UCLES 2014 9701/11/M/J/14 [Turn over 4

7 The Contact process is used in the manufacture of sulfuric acid. The equation for the main reaction is shown below.

–1 2SO2(g) + O2(g) 2SO3(g) ∆H = –196 kJ mol

Which statement about this reaction is incorrect?

A Increased pressure gives a higher yield of SO3.

B Increased temperature gives a higher yield of SO3. C In the forward reaction the oxidation state of sulfur changes from +4 to +6.

D Vanadium(V) oxide is used as a catalyst.

8 When making sparkler fireworks, a mixture of barium nitrate powder with aluminium powder, water and glue is coated onto wires and allowed to dry. At this stage, the following exothermic reaction may occur.

16Al + 3Ba(NO3)2 + 36H2O → 3Ba(OH)2 + 16Al (OH)3 + 6NH3

Which conditions would be best to reduce the rate of this reaction during the drying process, and would also keep the aluminium and barium nitrate unchanged?

temperature / K pH

A 298 7 B 298 14 C 398 7 D 398 14

9 Which molecular structure will have the smallest overall dipole?

A B C D l l l l l H3CHC 3CCHC C C C 3 C C C C C C C C

Cl H C H H CHCl CHCH C Cl 2 3 3 3 3

10 The equilibrium constant, Kc, for the reaction H2(g) + I2(g) 2HI(g), is 60 at 450 °C.

What is the number of moles of hydrogen iodide in equilibrium with 2 mol of hydrogen and 0.3 mol of iodine at 450 °C? 1 1 A B C 6 D 36 100 10

© UCLES 2014 9701/11/M/J/14 5

11 Boltzmann distributions are shown in the diagrams.

diagram 1 diagram 2 X Y P

number of Q number of molecules molecules

0 0 0 0 molecular energy molecular energy

In diagram 1, one curve, P or Q, corresponds to a temperature higher than that of the other curve.

In diagram 2, one line, X or Y, corresponds to the activation energy in the presence of a catalyst and the other line corresponds to the activation energy of the same reaction in the absence of a catalyst.

Which combination gives the correct curve and line?

higher presence of

temperature catalyst

A P X B P Y C Q X D Q Y

12 Redox reactions occur very frequently in the chemistry of Group VII.

Which statement is correct?

A Chlorine will oxidise bromide ions but not iodide ions.

B Fluorine is the weakest oxidising agent out of F2, Cl 2, Br2 and I2.

– – – – C Iodide ions are the weakest reducing agent out of F , Cl , Br and I . D When chlorine reacts with water, chlorine is both oxidised and reduced.

© UCLES 2014 9701/11/M/J/14 [Turn over 6

13 When equal volumes of saturated solutions of barium hydroxide and calcium hydroxide are mixed, a white precipitate, Y, forms. The mixture is filtered and carbon dioxide is bubbled through the filtrate, producing a second white precipitate, Z.

What are Y and Z?

Y Z

A Ba(OH)2 Ca(OH)2

B Ba(OH)2 CaCO3

C Ca(OH)2 BaCO3

D Ca(OH)2 Ba(OH)2

14 What is the order of increasing melting point of the four chlorides shown?

CCl 4 HCl MgCl 2 PCl 5

lowest highest

melting point melting point

A CCl 4 HCl PCl 5 MgCl 2

B HCl CCl 4 PCl 5 MgCl 2

C HCl PCl 5 CCl 4 MgCl 2

D MgCl 2 PCl 5 CCl 4 HCl

15 When calcium is burnt in oxygen, what colour is the flame?

A green B red C white D yellow

16 Which description of the bonding and acid / base nature of aluminium oxide is correct?

bonding acid / base nature

A covalent amphoteric B covalent basic C ionic amphoteric D ionic basic

© UCLES 2014 9701/11/M/J/14 7

17 Group II nitrates undergo thermal decomposition according to the following equation.

→ 1 X(NO3)2 XO + 2NO2 + 2 O2

Which Group II nitrate requires the highest temperature to bring about its thermal decomposition?

A barium nitrate B calcium nitrate C magnesium nitrate D strontium nitrate

18 Use of the Data Booklet is relevant to this question.

3 A chemist took 2.00 dm of nitrogen gas, measured under room conditions, and reacted it with a large volume of hydrogen gas, in order to produce ammonia. Only 15.0% of the nitrogen gas reacted to produce ammonia.

What mass of ammonia was formed?

A 0.213 g B 0.425 g C 1.42 g D 2.83 g

© UCLES 2014 9701/11/M/J/14 [Turn over 8

19 Use of the Data Booklet is relevant to this question.

Which graph correctly shows relative electronegativity plotted against relative atomic radius for the elements Na, Mg, Al and Si?

A B

Si

Al Al electronegativity Mg electronegativity Mg Si Na Na

atomic radius atomic radius

C D

Na Si

Mg Al l electronegativity A electronegativity Mg Si Na

atomic radius atomic radius

20 Many organic reactions need to be heated before reaction occurs, but some do not require heating.

Which reaction occurs quickly at room temperature?

A C2H4 + Br2 → C2H4Br2

B C2H4 + H2O → CH3CH2OH

C CH3CH2OH → C2H4 + H2O

D CH3CH2OH + HBr → CH3CH2Br + H2O

© UCLES 2014 9701/11/M/J/14 9

21 Hydroxyethanal, HOCH2CHO, is heated under reflux with an excess of acidified potassium dichromate(VI) until no further oxidation takes place.

What is the skeletal formula of the organic product?

A B CD

O O OH O HO HO OH OH OH OH O O O O

22 An ester with an odour of banana has the following formula.

CH3CO2CH2CHCH2CH3

CH 3

Which pair of reactants, under suitable conditions, will produce this ester?

A CH3CH2CHCH2CO2H + CH3OH

CH3 B CH3CH2CHCO2H + CH3CH2OH

CH3 C CH3CO2H + CH3CH2CHCH2OH

CH3 D CH3CO2H + CH3CHCH2CH2OH

CH 3

23 The hydrolysis of 1-chloropropane to produce propan-1-ol is much slower than the corresponding hydrolysis of 1-iodopropane.

Which statement explains this observation?

A Chlorine is more electronegative than iodine.

B The bond strength of the C – I bond is less than that of the C – Cl bond.

C The carbon atom in the C – Cl bond is more δ+ than that in the C – I bond. D The hydrolysis involves a nucleophilic addition reaction.

© UCLES 2014 9701/11/M/J/14 [Turn over 10

24 There are three structural isomers with the formula C5H12.

Which formulae correctly represent these three structural isomers?

A CH3CH2CH2CH2CH3 CH3CH2CHCH3CH3 CH3CH3CCH3CH3

B CH3CH2CH2CH2CH3 CH3CH2(CH)CH3CH3 C(CH3)4

C CH3CH2CH2CH2CH3 CH3CH(CH3)CH2CH3 CH3C(CH3)2CH3

D CH3CH2CH2CH2CH3 CH3CH(CH3)CH2CH3 CH3CH2CH(CH3)CH3

25 CH3CH2COCH2CH3 reacts with hydrogen cyanide to form an organic product called a cyanohydrin.

Which feature applies to the cyanohydrin product?

A It has one chiral centre. B It is formed by electrophilic addition. C It is formed via an intermediate which contains the C–OH group. D Its formation requires the use of cyanide ions as a catalyst.

26 How many moles of oxygen molecules are needed for the complete combustion of one mole of 3-methylpent-2-ene? A B 1 C D 9 92 18 19

27 The hydrocarbon C17H36 can be cracked.

Which compound is the least likely to be produced in this reaction?

A C3H8 B C4H8 C C8H16 D C16H34

28 Compound X has the molecular formula C4H10O2. X has an unbranched carbon chain and contains two OH groups.

On reaction with an excess of hot, acidified, aqueous manganate(VII) ions, X is converted into a compound of molecular formula C4H6O4.

To which two carbon atoms in the chain of X are the two OH groups attached?

A 1st and 2nd B 1st and 3rd C 1st and 4th D 2nd and 3rd

© UCLES 2014 9701/11/M/J/14 11

29 How many geometrical (cis-trans) isomers are there of hex-2,4-diene, CH3CH=CHCH=CHCH3?

A none; hex-2,4-diene does not show geometric isomerism B 2 C 3 D 4

30 Butanoic acid can be produced from 1-bromopropane using reagents X and Y as shown below.

reagent X reagent Y 1-bromopropane compound Q butanoic acid

What could be reagents X and Y?

X Y

A KCN in ethanol HCl (aq) B KCN in ethanol NaOH(aq)

C NH3 in ethanol HCl (aq) + 2– D NaOH(aq) H / Cr2O7 (aq)

© UCLES 2014 9701/11/M/J/14 [Turn over 12

Section B

For each of the questions in this section, one or more of the three numbered statements 1 to 3 may be correct.

Decide whether each of the statements is or is not correct (you may find it helpful to put a tick against the statements that you consider to be correct).

The responses A to D should be selected on the basis of

A B C D

1, 2 and 3 1 and 2 2 and 3 1 only are only are only are is correct correct correct correct

No other combination of statements is used as a correct response.

31 Use of the Data Booklet is relevant to this question.

When the liquid N2F4 is heated, it decomposes into a single product, X.

Which statements are correct?

1 N – F bonds are broken during this decomposition.

–1 2 The enthalpy change when N2F4 decomposes into X is approximately +160 kJ mol . 3 Molecules of X are non-linear.

32 P and Q are two liquid compounds with similar Mr values. Molecules of P attract each other by hydrogen bonds. Molecules of Q attract each other by van der Waals’ forces only.

How do the properties of P and Q differ?

1 P has a higher surface tension than Q. 2 P is less soluble in water than Q. 3 P has a lower melting point than Q.

33 R and S react together.

R + S T

Which factors affect the rate of the forward reaction?

1 the activation energy of the reaction 2 the enthalpy change of the reaction 3 the equilibrium constant of the reaction

© UCLES 2014 9701/11/M/J/14 13

34 The Brønsted-Lowry theory describes acid and base character.

When concentrated sulfuric acid and concentrated nitric acid are mixed, the following reactions occur.

– + H2SO4 + HNO3 HSO4 + H2NO3

+ + H2NO3 H2O + NO2

– + H2O + H2SO4 HSO4 + H3O

Which species are bases in these reactions?

– 1 HSO4

2 HNO3

+ 3 NO2

35 Pollutant oxide Y, which contains non-metallic element X, is formed in a car engine.

Further oxidation of Y to Z occurs in the atmosphere. In this further oxidation, 1 mol of Y reacts with 0.5 mol of gaseous oxygen molecules.

X could be either nitrogen or sulfur.

Which statements about X, Y and Z can be correct?

1 The oxidation number of X increases by two from Y to Z. 2 Y has an unpaired electron in its molecule. 3 Y is a polar molecule.

36 A test-tube of HI gas and a test-tube of HBr gas are placed together in an environment at temperature, T.

Which combinations of observations are possible depending on the temperature, T?

1 A brown vapour appears in one of the test-tubes. No change is apparent in the other test-tube. 2 A brown vapour appears in one of the test-tubes. A purple vapour appears in the other test-tube. 3 No change is apparent in either test-tube.

37 Which pairs of reagents will take part in a redox reaction?

1 CH3COCH3 + Tollens’ reagent

2 CH3CH2CHO + Fehling’s reagent

3 CH3CH=CH2 + Br2

© UCLES 2014 9701/11/M/J/14 [Turn over 14

The responses A to D should be selected on the basis of

A B C D

1, 2 and 3 1 and 2 2 and 3 1 only are only are only are is correct correct correct correct

No other combination of statements is used as a correct response.

38 The molecule responsible for the pineapple flavour used in sweets is CH3CH2CH2CO2CH2CH3.

Which statements about this molecule are correct?

1 The name of this compound is ethyl butanoate. 2 This compound is a structural isomer of hexanoic acid. 3 When this compound is heated with aqueous sodium hydroxide, the products are butan-1-ol and sodium ethanoate.

39 The compound pentan-1,4-diol has two OH groups per molecule and can be oxidised.

Which statements about pentan-1,4-diol or its oxidation products are correct?

1 When one mole of pentan-1,4-diol reacts with an excess of sodium metal, one mole of hydrogen molecules is produced. 2 At least one of the possible oxidation products of pentan-1,4-diol will react with 2,4-dinitrophenylhydrazine reagent.

3 Dehydration of pentan-1,4-diol could produce a compound with empirical formula C5H8.

40 Use of the Data Booklet is relevant to this question.

In an organic synthesis, a 62% yield of product is achieved.

Which conversions are consistent with this information?

1 74.00 g of butan-2-ol → 44.64 g of butanone

2 74.00 g of butan-1-ol → 54.56 g of butanoic acid

3 74.00 g of 2-methylpropan-1-ol → 54.56 g of 2-methylpropanoic acid

© UCLES 2014 9701/11/M/J/14 Cambridge International Examinations Cambridge International Advanced Subsidiary and Advanced Level 

CHEMISTRY 9701/21

Paper 2 Structured Questions AS Core May/June 2014

1 hour 15 minutes Candidates answer on the Question Paper. Additional Materials: Data Booklet

READ THESE INSTRUCTIONS FIRST

Write your Centre number, candidate number and name on all the work you hand in. Write in dark blue or black pen. You may use an HB pencil for any diagrams or graphs. Do not use staples, paper clips, glue or correction fl uid. DO NOT WRITE IN ANY BARCODES.

Answer all questions. Electronic calculators may be used. You may lose marks if you do not show your working or if you do not use appropriate units. A Data Booklet is provided.

At the end of the examination, fasten all your work securely together. The number of marks is given in brackets [ ] at the end of each question or part question.

This document consists of 8 printed pages.

IB14 06_9701_21/2RP © UCLES 2014 [Turn over 2

Answer all the questions in the spaces provided.

1 (a) Explain what is meant by the term ionisation energy.

......

......

...... [3]

–1 (b) The fi rst seven ionisation energies of an element, A, in kJ mol , are

1012 1903 2912 4957 6274 21 269 25 398.

(i) State the group of the Periodic Table to which A is most likely to belong. Explain your answer.

......

......

......

...... [2]

(ii) Complete the electronic confi guration of the element in Period 2 that is in the same group as A.

1s2 ...... [1]

(c) Another element, Z, in the same period of the Periodic Table as A, reacts with chlorine to

form a compound with empirical formula ZCl 2. The percentage composition by mass of ZCl 2 is Z, 31.13; Cl , 68.87.

(i) De fi ne the term relative atomic mass.

......

......

...... [2]

(ii) Calculate the relative atomic mass, Ar, of Z. Give your answer to three signifi cant fi gures.

Ar of Z = ...... [2]

© UCLES 2014 9701/21/M/J/14 3

(d) The chlorides of elements in Period 3 of the Periodic Table show different behaviours on addition to water, depending on their structure and bonding.

(i) Write equations to show the behaviour of sodium chloride, NaCl , and silicon chloride,

SiCl 4 , when separately added to an excess of water.

NaCl ......

SiCl 4 ...... [2]

(ii) State and explain the differences in behaviour of these two chlorides when added to water, in terms of their structure and the bonding found in the compounds.

......

......

......

......

......

...... [4]

(e) Sulfur reacts with fl uorine to form SF6. State the shape and bond angle of SF6.

shape of SF6 ......

bond angle of SF6 ...... [2]

[Total: 18]

© UCLES 2014 9701/21/M/J/14 [Turn over 4

2 The commonest form of iron(II) sulfate is the heptahydrate, FeSO4.7H2O. On heating at 90 C this loses some of its water of crystallisation to form a different hydrated form of iron(II) sulfate,

FeSO4.xH2O.

3 3.40 g of FeSO4.xH2O was dissolved in water to form 250 cm of solution.

3 –3 A 25.0 cm sample of this solution was acidifi ed and titrated with 0.0200 mol dm potassium manganate(VII).

3 In this titration 20.0 cm of this potassium manganate(VII) solution was required to react fully with the Fe2+ ions present in the sample.

– 2+ (a) The MnO4 ions in the potassium manganate(VII) oxidise the Fe ions in the acidifi ed solution.

(i) Explain, in terms of electron transfer, the meaning of the term oxidise in the sentence above.

......

...... [1]

(ii) Complete and balance the ionic equation for the reaction between the manganate(VII) ions and the iron(II) ions.

– 2+ + 3+ MnO 4 (aq) + 5Fe (aq) + ...... H (aq)  ...... (aq) + 5Fe (aq) + ...... H2O(l) [3]

(b) (i) Calculate the number of moles of manganate(VII) used in the titration.

[1]

(ii) Use the equation in (a)(ii) and your answer to (b)(i) to calculate the number of moles of 2+ 3 Fe present in the 25.0 cm sample of solution used.

[1]

(iii) Calculate the number of moles of FeSO4.xH2O in 3.40 g of the compound.

[1]

(iv) Calculate the relative formula mass of FeSO4.xH2O.

[1]

(v) The relative formula mass of anhydrous iron(II) sulfate, FeSO4, is 151.8.

Calculate the value of x in FeSO4.xH2O.

[1]

[Total: 9]

© UCLES 2014 9701/21/M/J/14 5

3 (a) In this question, K, L and M refer to a halogen atom or halide ion. For each part question, read the information and complete the answer lines below.

(i) When concentrated sulfuric acid is added to solid NaK, white fumes are produced that turn damp blue litmus paper red. No other colour changes are observed.

identity of K = ......

equation for reaction ......

explanation of observation ......

...... [3]

(ii) When silver nitrate solution is added to an aqueous solution of NaL, a precipitate forms that remains after the addition of concentrated ammonia solution.

identity of L = ......

colour of precipitate ......

equation for reaction ...... [3]

(iii) M 2 is a liquid at room temperature with a boiling point higher than that of chlorine but lower than that of iodine.

identity of M = ......

explanation ......

......

......

......

...... [2]

© UCLES 2014 9701/21/M/J/14 [Turn over 6

(b) The diagram below is a simplifi ed representation of a diaphragm cell.

product B product C

+ –

concentrated brine

product D

titanium steel anode diaphragm cathode

(i) Identify each of the products.

B ......

C ......

D ...... [3]

(ii) Give the equations for the two electrode reactions.

anode ......

cathode ...... [2]

[Total: 13]

© UCLES 2014 9701/21/M/J/14 7

4 Alkanes and alkenes both react with bromine.

(a) Explain how and why bromine can be used to distinguish between an and an .

......

......

...... [2]

(b) The reaction of ethane with bromine forms a mixture of products.

(i) State the essential conditions for this reaction to occur.

...... [1]

(ii) Give the full name of the mechanism of this reaction.

...... [2]

(iii) Give the equation for a termination step that could occur, producing a hydrocarbon.

...... [1]

(iv) Give the equation for one propagation step involved in the formation of dibromoethane from bromoethane during this reaction.

...... [1]

(c) The reaction of ethene with bromine forms a single product.

(i) Give the full name of the mechanism of this reaction.

...... [2]

(ii) Complete the diagram below to illustrate this mechanism. Include all relevant charges, partial charges, curly arrows and lone pairs.

H H H H

HHC C HHC C

Br Br Br

Br

[4]

© UCLES 2014 9701/21/M/J/14 [Turn over 8

(d) Chloroethene can be polymerised to form a polymer commonly known as PVC. Draw a diagram of the structure of PVC including three repeat units.

[2]

(e) Chloroethane undergoes a series of reactions as shown in the diagram below.

H H H H reaction 1 product Q HHC C CC NaCN Cl H H H

OH–(aq)

product P

(i) Give the reagent and conditions necessary for reaction 1.

......

...... [2]

(ii) Give the skeletal formula of product P.

[1]

(iii) Give the displayed formula and the name of product Q.

...... [2]

[Total: 20]

Permission to reproduce items where third-party owned material protected by copyright is included has been sought and cleared where possible. Every reasonable effort has been made by the publisher (UCLES) to trace copyright holders, but if any items requiring clearance have unwittingly been included the publisher will be pleased to make amends at the earliest possible opportunity.

Cambridge International Examinations is part of the Cambridge Assessment Group. Cambridge Assessment is the brand name of University of Cambridge Local Examinations Syndicate (UCLES), which is itself a department of the University of Cambridge.

© UCLES 2014 9701/21/M/J/14

Cambridge International Examinations Cambridge International Advanced Subsidiary and Advanced Level

CHEMISTRY 9701/11

Paper 1 Multiple Choice May/June 2015

1 hour

Additional Materials: Multiple Choice Answer Sheet *4421037079* Soft clean eraser Soft pencil (type B or HB is recommended) Data Booklet

READ THESE INSTRUCTIONS FIRST

Write in soft pencil. Do not use staples, paper clips, glue or correction fluid. Write your name, Centre number and candidate number on the Answer Sheet in the spaces provided unless this has been done for you. DO NOT WRITE IN ANY BARCODES.

There are forty questions on this paper. Answer all questions. For each question there are four possible answers A, B, C and D. Choose the one you consider correct and record your choice in soft pencil on the separate Answer Sheet.

Read the instructions on the Answer Sheet very carefully.

Each correct answer will score one mark. A mark will not be deducted for a wrong answer. Any rough working should be done in this booklet. Electronic calculators may be used.

This document consists of 15 printed pages and 1 blank page.

IB15 06_9701_11/4RP © UCLES 2015 [Turn over

2

Section A

For each question there are four possible answers, A, B, C, and D. Choose the one you consider to be correct.

1 The diagram shows a Boltzmann distribution of molecular energies for a gaseous mixture. The distribution has a peak, labelled P on the diagram.

P

activation number of energy molecules

0 0 molecular energy

What happens when an effective catalyst is added to the mixture?

A The height of the peak decreases and the activation energy moves to the right. B The height of the peak decreases and the activation energy moves to the left. C The height of the peak remains the same and the activation energy moves to the right. D The height of the peak remains the same and the activation energy moves to the left.

2 Which quantity gives the best indication of the relative strengths of the hydrogen bonds between the molecules in liquid hydrogen halides?

A bond dissociation energies B enthalpy changes of formation C enthalpy changes of solution D enthalpy changes of vaporisation

3 Use of the Data Booklet is relevant to this question.

1.00 g of carbon is combusted in a limited supply of pure oxygen. 0.50 g of the carbon combusts to form CO2 and 0.50 g of the carbon combusts to form CO.

The resultant mixture of CO2 and CO is passed through excess NaOH(aq) and the remaining gas is then dried and collected.

What is the volume of the remaining gas? (All gas volumes are measured at 25 °C and 1 atmosphere pressure.)

3 3 3 3 A 1 dm B 1.5 dm C 2 dm D 3 dm

© UCLES 2015 9701/11/M/J/15 3

4 In oxygen difluoride, OF2, fluorine has an oxidation number of –1. OF2 will react with sulfur dioxide according to the following equation.

OF2 + SO2 → SO3 + F2

What is oxidised and what is reduced in this reaction?

oxygen fluorine sulfur in OF2

A oxidised oxidised reduced B oxidised reduced oxidised C reduced oxidised reduced D reduced reduced oxidised

5 Use of the Data Booklet is relevant to this question.

The gas laws can be summarised in the ideal gas equation below.

pV = nRT

–3 3 0.96 g of oxygen gas is contained in a glass vessel of volume 7.0 × 10 m at a temperature of 30 °C.

Assume the gas behaves as an ideal gas.

What is the pressure in the vessel?

A 1.1 kPa B 2.1 kPa C 10.8 kPa D 21.6 kPa

3 6 One mole of phosphorus(V) chloride, PCl 5, is heated to 600 K in a sealed flask of volume 1 dm . Equilibrium is established and measurements are taken.

PCl 5(g) PCl 3(g) + Cl 2(g)

The experiment is repeated with one mole of phosphorus(V) chloride heated to 600 K in a sealed 3 flask of volume 2 dm .

How will the measurements vary?

A The equilibrium concentrations of PCl 3(g) and Cl 2(g) are higher in the second experiment.

B The equilibrium concentration of PCl 5(g) is lower in the second experiment. C The equilibrium concentrations of all three gases are the same in both experiments. D The value of the equilibrium constant is higher in the second experiment.

© UCLES 2015 9701/11/M/J/15 [Turn over 4

7 The standard enthalpy changes of combustion of glucose and ethanol are given as –2820 and –1 –1368 kJ mol respectively.

Glucose, C6H12O6, can be converted into ethanol.

C6H12O6(s) → 2C2H5OH(l) + 2CO2(g)

What is the standard enthalpy change for this reaction?

–1 A –1452 kJ mol

–1 B –84 kJ mol

–1 C +84 kJ mol

–1 D +1452 kJ mol

8 In which reaction is the underlined substance acting as a base?

+ – A HNO3 + H2SO4 → H2NO3 + HSO4

– – B HSiO3 + HCN → CN + H2O + SiO2

– – C HNO2 + HCO3 → H2O + CO2 + NO2

– – D C6H5O + CH2Cl CO2H → C6H5OH + CH2Cl CO2

9 One molecule of haemoglobin, Hb, can bind with four molecules of oxygen according to the following equation.

Hb(aq) + 4O2(aq) Hb(O2)4(aq)

–6 –3 When the equilibrium concentration of O2 is 7.6 × 10 mol dm , the equilibrium concentrations of Hb and Hb(O2)4 are equal.

What is the value of Kc for this equilibrium?

A 3.0 × 1020 B 1.3 × 105 C 7.6 × 10–6 D 3.3 × 10–21

10 The double bond between the two carbon atoms in an ethene molecule consists of one σ bond and one π bond.

Which orbitals overlap to form each of these bonds?

σ bond π bond

A sp2–sp2 p–p B sp2–sp2 sp2–sp2 C sp3–sp3 p–p D sp3–sp3 sp2–sp2

© UCLES 2015 9701/11/M/J/15 5

11 The diagram shows a reaction pathway for an endothermic reaction.

Which arrow represents the activation energy for the forward reaction?

C

enthalpy B –1 / kJ mol A D

0 0 extent of reaction

12 In the industrial electrolysis of brine to manufacture chlorine, the diaphragm used is a porous screen which allows the flow of electrolytes but keeps other chemicals separate.

Which substance needs to be kept separate from the chlorine by the diaphragm?

A hydrogen B sodium chloride C sodium hydroxide D water

13 The three minerals below are obtained from mines around the world. Each one behaves as a mixture of two carbonate compounds. They can be used as fire retardants because they decompose in the heat, producing CO2. This gas smothers the fire.

barytocite BaCa(CO3)2

dolomite CaMg(CO3)2

huntite Mg3Ca(CO3)4

What is the order of effectiveness as fire retardant, from best to worst?

best worst

A dolomite barytocite huntite B dolomite huntite barytocite C huntite barytocite dolomite D huntite dolomite barytocite

© UCLES 2015 9701/11/M/J/15 [Turn over 6

14 Which observations are made when a sample of silicon chloride, SiCl 4, is added to a beaker of water?

A No visible changes are observed. B Steamy fumes and a white precipitate are both observed. C The appearance of a white precipitate is the only observation. D The appearance of steamy fumes is the only observation.

15 Use of the Data Booklet is relevant to this question.

Which diagram correctly shows the atomic radii of the elements Mg, Al, Si and P plotted against their melting points?

A B

Al Mg

P atomic Mg atomic Si radius radius Al Si P

melting point melting point

C D

Mg Mg

l atomic A atomic Al radius radius Si Si P P

melting point melting point

© UCLES 2015 9701/11/M/J/15 7

16 Chlorine is widely used in water treatment plants.

Which reaction takes place when chlorine dissolves in water?

A Cl 2 + H2O → HCl + HCl O

B 2Cl 2 + 2H2O → 3HCl + HCl O2

C 3Cl 2 + 3H2O → 5HCl + HCl O3

D 4Cl 2 + 4H2O → 7HCl + HCl O4

17 Astatine, At, is below iodine in Group VII of the Periodic Table.

Which statement is most likely to be correct?

A AgAt(s) reacts with excess dilute aqueous ammonia to form a solution of a soluble complex.

B Astatine and KCl (aq) react to form KAt(aq) and chlorine. C KAt(aq) and dilute sulfuric acid react to form white fumes of HAt(g). D NaAt(s) and concentrated sulfuric acid react to form astatine.

18 Mohr’s salt is a pale green crystalline solid which is soluble in water. It contains two cations, one 2+ 2– of which is Fe , and one anion which is SO4 .

The identity of the second cation was determined by heating Mohr’s salt with aqueous sodium hydroxide. A colourless gas was evolved which readily dissolved in water giving an alkaline solution.

A green precipitate was also formed.

What are the identities of the gas and the precipitate?

gas precipitate

A NH3 Fe(OH)2

B NH3 Na2SO4

C SO2 Fe(OH)2

D SO2 Na2SO4

19 Use of the Data Booklet is relevant to this question.

Which mass of solid residue will be obtained from the thermal decomposition of 4.10 g of anhydrous calcium nitrate?

A 0.70 g B 1.00 g C 1.40 g D 2.25 g

© UCLES 2015 9701/11/M/J/15 [Turn over 8

20 Which reagent will give a different observation with compounds P and Q?

HOCH2CH(OH)CHO HOCH2COCH2OH P Q

A Br2(aq)

B hot acidified KMnO4 C silver nitrate in ammonia solution

D warm acidified K2Cr2O7

21 The cyclic compound M is heated with dilute hydrochloric acid.

O

O NH

O compound M

What are the products of the reaction?

A HOCH2CO2H and H2NCH2CO2H

+ B HO2CCH2OH and HO2CCH2NH3

C H2NCOCH2OH and HOCH2CHO

+ D HOCH2CONH3 and HOCH2CHO

22 Cottonseed oil contains large amounts of polyunsaturated carboxylic acids. When this oil is used to make margarine, the C=C double bonds in the unsaturated carboxylic acids are hydrogenated.

What reagents and conditions would be suitable to bring about this hydrogenation reaction?

A H2 gas, nickel catalyst, 400 °C

B LiAl H4 in dry ether

C NaBH4, aqueous solution

D steam, concentrated H2SO4, 300 °C and 60 atm pressure

23 Which intermediate ion forms in the greatest amount during the addition of HBr to propene?

+ A CH3CH CH3

+ B CH3CH2CH2

– C CH3CH CH2Br

– D CH3CHBrCH2

© UCLES 2015 9701/11/M/J/15 9

24 Coniine is the major constituent of the poison ‘oil of hemlock’.

N CH2CH2CH3 H

coniine

Coniine can be synthesised by reacting ammonia with a dibromo compound, X.

NH3 + C8H16Br2 → coniine + 2HBr X

What is the name of compound X?

A 1,1-dibromo-2-propylcyclopentane B 1,2-dibromo-2-propylcyclopentane C 1,4-dibromooctane D 1,5-dibromooctane

25 2-bromopropane reacts with a hot concentrated solution of sodium hydroxide in ethanol.

Which substance is the major product of this reaction?

A propan-1-ol B propan-2-ol C 2-hydroxypropene D propene

26 The presence of a halogen in an organic compound may be detected by warming the organic compound with aqueous silver nitrate.

Which compound would be the quickest to produce a precipitate?

A B C D Cl F F Cl

Cl Cl F Br

l I C F F

© UCLES 2015 9701/11/M/J/15 [Turn over 10

27 Which reagent will give a different observation with compounds W and X?

CH2OH CH3 OH

W X

A hot SOCl 2

B hot acidified K2Cr2O7 C NaOH(aq) D warm Fehling’s reagent

28 In 1869 Ladenburg suggested a structure for benzene, C6H6, in which one hydrogen atom is attached to each carbon atom.

CH HC CH

CH HC CH

Ladenburg structure

A compound C6H4Cl 2 could be formed with the same carbon skeleton as the Ladenburg structure.

How many structural isomers would this compound have?

A 3 B 4 C 5 D 6

© UCLES 2015 9701/11/M/J/15 11

29 The citrus flavour of lemons is due to the compound limonene, present in both the peel and the juice.

limonene

What is the mole ratio of carbon dioxide to water produced when limonene is completely burnt in oxygen?

number of moles number of moles

carbon dioxide water

A 4 3 B 5 4 C 5 8 D 9 7

30 What is the organic product when ethanamide, CH3CONH2, is boiled with excess aqueous sodium hydroxide?

A CH3CN

– + B CH3CO2 NH4

– + C CH3CONH Na

– + D CH3CO2 Na

© UCLES 2015 9701/11/M/J/15 [Turn over 12

Section B

For each of the questions in this section, one or more of the three numbered statements 1 to 3 may be correct.

Decide whether each of the statements is or is not correct (you may find it helpful to put a tick against the statements that you consider to be correct).

The responses A to D should be selected on the basis of

A B C D

1, 2 and 3 1 and 2 2 and 3 1 only are only are only are is correct correct correct correct

No other combination of statements is used as a correct response.

31 Use of the Data Booklet is relevant to this question.

The isotope 99Tc is radioactive and has been found in lobsters and seaweed adjacent to nuclear fuel reprocessing plants.

Which statements are correct about an atom of 99Tc?

1 It has 13 more neutrons than protons. 2 It has 43 protons. 3 It has 99 nucleons.

32 Urea is a product of animal metabolism. It can also be used as a fertiliser.

H

x N H O C H N

H

urea

The diagram shows angle x in this molecule.

Which statements about the structure of urea are correct?

1 Angle x is approximately 120°.

2 The molecule has two π bonds. 3 The molecule has only three lone pairs of electrons.

© UCLES 2015 9701/11/M/J/15 13

33 Aluminium is extracted from aluminium oxide by electrolysis.

Which statements are correct?

1 Aluminium oxide has an extremely high melting point. 2 Bauxite is added to the aluminium oxide to lower its melting point.

3 Oxygen produced at the graphite cathode reacts with the graphite to produce CO2.

34 Why does raising the pressure of a fixed mass of gaseous reactants at a constant temperature cause an increase in the rate of reaction?

1 More collisions occur per second when the pressure is increased. 2 More molecules have energy greater than the activation energy at the higher pressure. 3 Raising the pressure lowers the activation energy.

35 Which statements about the industrial manufacture of sulfuric acid are correct?

1 Sulfur is burned to form sulfur dioxide.

2 The stage that forms sulfur trioxide involves a V2O5 catalyst. 3 The stage that forms sulfur trioxide is non-reversible.

36 X is a Group II metal. It forms a sulfate which is more soluble than barium sulfate. It forms a hydroxide which is more soluble than calcium hydroxide.

What could be the identity of X?

1 strontium 2 magnesium 3 beryllium

37 The diagram shows the structure of an alkene molecule.

CH3 CH3

CH3 CCH C C H H H CH 3

Which statements about this molecule are correct?

1 All the carbon atoms are in the same plane. 2 It has geometrical isomers. 3 It is optically active.

© UCLES 2015 9701/11/M/J/15 [Turn over 14

The responses A to D should be selected on the basis of

A B C D

1, 2 and 3 1 and 2 2 and 3 1 only are only are only are is correct correct correct correct

No other combination of statements is used as a correct response.

38 An organic compound, X, will react with calcium metal to produce a salt with the empirical formula CaC4H4O4.

What could be the identity of X?

1 ethanoic acid 2 butanedioic acid 3 2-methylpropanedioic acid

39 Which compounds will react with HBr to give the compound R?

Br

R

12 3

OH

© UCLES 2015 9701/11/M/J/15 15

40 5-hydroxypentanoic acid is readily converted into the cyclic compound L.

O

O L

Which statements about this reaction are correct?

1 Acidified sodium dichromate(VI) is used as a reagent. 2 A water molecule is produced in the reaction.

3 The reaction is catalysed by concentrated H2SO4.

© UCLES 2015 9701/11/M/J/15 Cambridge International Examinations Cambridge International Advanced Subsidiary and Advanced Level *8582611017*

CHEMISTRY 9701/21

Paper 2 Structured Questions AS Core May/June 2015

1 hour 15 minutes Candidates answer on the Question Paper. Additional Materials: Data Booklet

READ THESE INSTRUCTIONS FIRST

Write your Centre number, candidate number and name on all the work you hand in. Write in dark blue or black pen. You may use an HB pencil for any diagrams or graphs. Do not use staples, paper clips, glue or correction fl uid. DO NOT WRITE IN ANY BARCODES.

Answer all questions. Electronic calculators may be used. You may lose marks if you do not show your working or if you do not use appropriate units. A Data Booklet is provided.

At the end of the examination, fasten all your work securely together. The number of marks is given in brackets [ ] at the end of each question or part question.

This document consists of 11 printed pages and 1 blank page.

IB15 06_9701_21/FP © UCLES 2015 [Turn over 2

Answer all the questions in the spaces provided.

1 (a) Chemists recognise that atoms are made of three types of particle.

Complete the following table with their names and properties.

name of particle relative mass relative charge

0

1/1836

[3]

(b) The relative atomic mass of an element can be determined using data from its mass spectrum.

The mass spectrum of element X is shown, with the percentage abundance of each isotope labelled.

60

49.61 50

40

percentage abundance 30 23.77

20

10 9.37 7.63 8.73

0.89 0 73 74 75 76 77 78 79 80 81 82 83 m / e

(i) De fi ne the terms relative atomic mass and isotope.

relative atomic mass ......

......

......

isotope ......

...... [3]

© UCLES 2015 9701/21/M/J/15 3

(ii) Use the data in the mass spectrum to calculate the relative atomic mass, Ar, of X. Give your answer to two decimal places and suggest the identity of X.

Ar of X ......

identity of X ...... [2]

(c) The element tellurium, Te, reacts with chlorine to form a single solid product, with a relative formula mass of 270. The product contains 52.6% chlorine by mass.

(i) Calculate the molecular formula of this chloride.

molecular formula ...... [3]

(ii) This chloride melts at 224 °C and reacts vigorously with water.

State the type of bonding and structure present in this chloride and explain your reasoning.

......

......

......

...... [2]

(iii) Suggest an equation for the reaction of this chloride with water.

...... [1]

© UCLES 2015 9701/21/M/J/15 [Turn over 4

(d) Sodium and silicon also react directly with chlorine to produce the chlorides shown.

melting difference between the chloride point / °C of the elements NaCl 801 2.2

SiCl 4 –69 1.3

(i) Describe what you would see during the reaction between sodium and chlorine.

......

......

...... [2]

(ii) Explain the differences between the melting points of these two chlorides in terms of their structure and bonding. You should refer to the difference between the electronegativities of the elements in your answer.

NaCl structure and bonding ......

......

SiCl 4 structure and bonding ......

......

explanation ......

......

......

......

......

...... [4]

[Total: 20]

© UCLES 2015 9701/21/M/J/15 5

2 The relationship pV = nRT can be derived from the laws of mechanics by assuming ideal behaviour for gases.

(a) The graph represents the relationship between pV and p for a real gas at three different

temperatures, T1, T2 and T3.

T1 T2

T3

pV

p

(i) Draw one line on the graph to show what the relationship should be for the same amount of an ideal gas. [1]

(ii) State and explain, with reference to the graph, which of T1, T2 or T3 is the lowest temperature.

......

...... [1]

(iii) Explain your answer to (ii) with reference to intermolecular forces.

......

...... [1]

(iv) State and explain the effect of pressure on the extent to which a gas deviates from ideal behaviour.

......

......

......

...... [2]

© UCLES 2015 9701/21/M/J/15 [Turn over 6

(b) A fl ask with a volume of 100 cm3 was fi rst weighed with air fi lling the fl ask, and then with another gas, Y, fi lling the fl ask. The results, measured at 26 °C and 1.00 × 105 Pa, are shown.

Mass of fl ask containing air = 47.930 g

Mass of fl ask containing Y = 47.989 g

Density of air = 0.00118 g cm–3

Calculate the relative molecular mass, Mr, of Y.

Mr of Y = ...... [4]

(c) Although nitrogen gas makes up about 79% of the atmosphere it does not easily form compounds.

(i) Explain why nitrogen is so unreactive.

......

...... [1]

(ii) Explain why the conditions in a car engine lead to the production of oxides of nitrogen.

......

...... [1]

(iii) Give an equation for a reaction involved in the removal of nitrogen monoxide, NO, from a car’s exhaust gases, in the catalytic converter.

...... [1]

© UCLES 2015 9701/21/M/J/15 7

One of the main reasons for reducing the amounts of oxides of nitrogen in the atmosphere is their contribution to the formation of acid rain.

(iv) Write an equation for the formation of nitric acid from nitrogen dioxide, NO2, in the atmosphere.

...... [1]

(v) Write equations showing the catalytic role of nitrogen monoxide, NO, in the oxidation of

atmospheric sulfur dioxide, SO2.

......

...... [2]

[Total: 15]

© UCLES 2015 9701/21/M/J/15 [Turn over 8

3 Ethanal reacts with hydrogen cyanide, in the presence of a small amount of NaCN, as shown.

CH3CHO + HCN → CH3CH(OH)CN

(a) Use bond energies from the Data Booklet to calculate the enthalpy change for this reaction. Include a sign with your answer.

enthalpy change = ...... kJ mol–1 [3]

(b) The product of this reaction shows stereoisomerism as it contains a chiral centre. This reaction produces an equimolar mixture of two optical isomers.

(i) Explain the meanings of the terms stereoisomerism and chiral centre.

stereoisomerism ......

......

......

chiral centre ......

...... [2]

(ii) Suggest why the two optical isomers are produced in equal amounts by this reaction.

......

...... [1]

© UCLES 2015 9701/21/M/J/15 9

(c) (i) Complete the diagram to show the mechanism of this reaction. Include all necessary charges, partial charges, lone pairs and curly arrows and show the structure of the intermediate.

N C–

H CN

H3CC H3CCH+

O OH H C N [5]

(ii) With reference to your mechanism in (i), explain the role of the NaCN in this reaction.

......

...... [1]

[Total: 12]

© UCLES 2015 9701/21/M/J/15 [Turn over 10

4 There are four alcohols, A, B, C and D, which are structural isomers with the molecular formula

C4H10O.

Alcohol A does not react with acidifi ed potassium dichromate(VI) solution but B, C and D do.

All four alcohols react with hot, concentrated sulfuric acid to form products with the molecular

formula C4H8. A, C and D each give a single product in this reaction. B gives a mixture of two structural isomers, one of which shows stereoisomerism.

(a) Give the skeletal formula for each of the four alcohols and complete the diagram with the names of the types of structural isomerism shown by each linked pair of compounds.

A B

...... isomerism

...... isomerism

C D

...... isomerism

[7]

© UCLES 2015 9701/21/M/J/15 11

(b) (i) Give the names of the two structural isomers produced by the reaction of B with hot, concentrated sulfuric acid

...... [2]

(ii) State which of these two isomers shows stereoisomerism. Explain why this molecule is capable of showing stereoisomerism.

......

......

......

...... [2]

(iii) Draw displayed formulae to show the two stereoisomers.

stereoisomer 1 stereoisomer 2 [2]

[Total: 13]

© UCLES 2015 9701/21/M/J/15 [Turn over

Cambridge International Examinations Cambridge International Advanced Subsidiary and Advanced Level

CHEMISTRY 9701/11

Paper 1 Multiple Choice May/June 2016

1 hour

Additional Materials: Multiple Choice Answer Sheet *9811287382* Soft clean eraser Soft pencil (type B or HB is recommended) Data Booklet

READ THESE INSTRUCTIONS FIRST

Write in soft pencil. Do not use staples, paper clips, glue or correction fluid. Write your name, Centre number and candidate number on the Answer Sheet in the spaces provided unless this has been done for you. DO NOT WRITE IN ANY BARCODES.

There are forty questions on this paper. Answer all questions. For each question there are four possible answers A, B, C and D. Choose the one you consider correct and record your choice in soft pencil on the separate Answer Sheet.

Read the instructions on the Answer Sheet very carefully.

Each correct answer will score one mark. A mark will not be deducted for a wrong answer. Any rough working should be done in this booklet. Electronic calculators may be used.

This document consists of 15 printed pages and 1 blank page.

IB16 06_9701_11/3RP © UCLES 2016 [Turn over

2

Section A

For each question there are four possible answers, A, B, C and D. Choose the one you consider to be correct.

Use of the Data Booklet may be appropriate for some questions.

1 Which equation shows the reaction that occurs during the standard enthalpy change of atomisation of bromine?

A Br2(l) → 2Br(g)

B Br2(g) → 2Br(g) C 1 → 2 Br2(l) Br(g) D 1 → 2 Br2(g) Br(g)

2 What is the correct number of bonds of each type in the Al 2Cl 6 molecule?

co-ordinate covalent (dative covalent)

A 6 1 B 6 2 C 7 0 D 7 1

3 Tetraethyl lead, Pb(C2H5)4, has been used as a petrol additive.

What is the percentage by mass of carbon in tetraethyl lead?

A 10.2 B 14.9 C 29.7 D 32.0

© UCLES 2016 9701/11/M/J/16 3

4 When nuclear reactions take place, the elements produced are different from the elements that reacted. Nuclear equations, such as the one below, are used to represent the changes that occur.

235 1 144 89 1 92U + 0n → 56Ba + 36Kr + 3 0n

The nucleon (mass) number total is constant at 236 and the proton number total is constant at 92.

2 In another nuclear reaction, uranium-238 is reacted with deuterium atoms, 1 H. An isotope of a new element, J, is formed as well as two neutrons.

238 2 1 92U + 1H → J + 2 0n

What is isotope J?

A 238Np B 238Pu C 240Np D 240Pu

5 Dicarbon monoxide, C2O, is found in dust clouds in space. The structure of this molecule is C=C=O. The molecule contains no unpaired electrons.

How many lone pairs of electrons are present in a molecule of C2O?

A 1 B 2 C 3 D 4

6 A white powder is known to be a mixture of magnesium oxide and aluminium oxide.

3 –3 100 cm of 2 mol dm NaOH(aq) is just sufficient to cause the aluminium oxide in x grams of the mixture to dissolve.

– – The reaction occurring is Al 2O3 + 2OH + 3H2O → 2Al (OH)4 .

3 –3 800 cm of 2 mol dm HCl (aq) is just sufficient to cause all of the oxide in x grams of the mixture to dissolve.

+ 3+ The reactions occurring are Al 2O3 + 6H → 2Al + 3H2O + 2+ and MgO + 2H → Mg + H2O.

How many moles of each oxide are present in x grams of the mixture?

aluminium magnesium

oxide oxide

A 0.05 0.25 B 0.05 0.50 C 0.10 0.25 D 0.10 0.50

© UCLES 2016 9701/11/M/J/16 [Turn over 4

7 At room temperature and pressure, H2O is a liquid and H2S is a gas.

What is the reason for this difference?

A O has higher first and second ionisation energies than S. B The covalent bond between O and H is stronger than the covalent bond between S and H.

C There is significant hydrogen bonding between H2O molecules but not between H2S molecules.

D The instantaneous dipole-induced dipole forces between H2O molecules are stronger than the instantaneous dipole-induced dipole forces between H2S molecules.

8 Gaseous phosphorus pentachloride can be decomposed into gaseous phosphorus trichloride and chlorine by heating. The table gives the bond energies.

–1 bond bond energy / kJ mol

P–Cl (in both chlorides) 330

Cl –Cl 242

What is the enthalpy change for the decomposition of PCl 5 to PCl 3 and Cl 2?

–1 A –418 kJ mol

–1 B –88 kJ mol

–1 C +88 kJ mol

–1 D +418 kJ mol

9 An aqueous solution was prepared containing a mixture of 1.0 mol of AgNO3 and 1.0 mol of 3 + FeSO4 in 1.00 dm of water. When equilibrium was established, there was 0.44 mol of Ag (aq) in the mixture.

Ag+(aq) + Fe2+(aq) Ag(s) + Fe3+(aq)

What is the numerical value of Kc?

A 0.62 B 1.40 C 1.62 D 2.89

10 The equation for the reaction between carbon monoxide and hydrogen is shown.

CO(g) + 3H2(g) CH4(g) + H2O(g)

What are the units of Kp for this reaction?

A kPa B kPa–1 C kPa2 D kPa–2

© UCLES 2016 9701/11/M/J/16 5

11 Enzymes are biological catalysts. Many enzymes show specificity. An example of an enzyme which shows specificity is glucokinase. Glucokinase is involved in the metabolism of glucose.

What does specificity mean in this context?

A Glucokinase is most effective as a catalyst over a narrow pH range. B Glucokinase is most effective as a catalyst over a narrow range of temperatures. C Glucokinase only operates on a narrow range of substrate molecules. D Glucokinase provides an alternative route for the reactions it catalyses.

12 Why is the ionic radius of a chloride ion larger than the ionic radius of a sodium ion?

A A chloride ion has one more occupied electron shell than a sodium ion. B Chlorine has a higher proton number than sodium. C Ionic radius increases regularly across the third period. D Sodium is a metal, chlorine is a non-metal.

13 Elements D and E are both in Period 3. Element D has the smallest atomic radius in Period 3. There are only two elements in Period 3 which have a lower melting point than element E. Elements D and E react together to form compound L.

Which compound could be L?

A MgCl 2 B MgS C Na2S D PCl 3

14 X and Y are both Group 2 metals.

X and Y both form hydroxide compounds, but X(OH)2 is more soluble in water than Y(OH)2.

If a piece of metal Y is put into cold water a very slow reaction occurs, and only a very few, small hydrogen bubbles can be seen.

What could be the identities of X and Y?

X Y

A barium magnesium B barium strontium C calcium strontium D magnesium calcium

© UCLES 2016 9701/11/M/J/16 [Turn over 6

15 The solids sodium chloride and sodium iodide both react with concentrated sulfuric acid at room temperature.

With NaCl , the products are NaHSO4 and HCl . With NaI, the products are NaHSO4, HI, I2, SO2, H2O, S and H2S.

What is the best explanation for this difference in products?

A Chloride ions will displace iodine from solution. B Hydrogen chloride is more volatile than hydrogen iodide. C Iodide ions are better reducing agents than chloride ions.

D Sulfuric acid is able to act as a dehydrating agent with NaI.

16 In some areas lime, Ca(OH)2, is added to soil to improve crop growth.

Which statement correctly describes a reason why lime improves crop growth?

A Lime acts as a catalyst which speeds up the release of nitrates into the soil. B Lime is an effective pesticide and protects the plants from damage. C Lime is used to reduce the acidity of the soil. D Lime lowers the pH of the soil.

17 A piece of rock has a mass of 2.00 g. It contains calcium carbonate, but no other basic 3 –3 substances. It neutralises exactly 36.0 cm of 0.500 mol dm hydrochloric acid.

What is the percentage of calcium carbonate in the 2.00 g piece of rock?

A 22.5% B 45.0% C 72.0% D 90.1%

18 Which statement about the ammonia molecule and/or the ammonium ion is correct?

A Ammonia molecules are basic because they can donate H+ ions. B Ammonium ions are basic because they can accept H+ ions. C If ammonium ions are heated with NaOH(aq), ammonia molecules are formed.

+ D The bond angle in NH4 is 2.5° less than the bond angle in NH3.

19 Which reaction does not contribute to the problem of acid rain?

A the combustion of fossil fuels B the oxidation of sulfur dioxide to sulfur trioxide catalysed by nitrogen dioxide C the reaction between nitrogen monoxide and carbon monoxide in a catalytic converter D the reaction of sulfur trioxide with water

© UCLES 2016 9701/11/M/J/16 7

20 The diagrams show two different compounds.

O O

1 2

What is

● the total number of structural isomers, including compound 2, that could be formed by adding a second methyl group to the ring of compound 1,

● the number of π electrons in each compound?

number of number of

isomers π electrons

A 3 2 B 3 4 C 5 2 D 5 4

21 The structural formula of compound X is shown below.

O

CH3CH2CH2OC

H X compound

What is the name of compound X and how does its boiling point compare with that of butanoic acid?

name of X boiling point of X

A methyl propanoate higher than butanoic acid B methyl propanoate lower than butanoic acid C propyl methanoate higher than butanoic acid D propyl methanoate lower than butanoic acid

© UCLES 2016 9701/11/M/J/16 [Turn over 8

22 Which pair of reagents will take part in a redox reaction?

A CH3CHCH2 + Br2

B CH3CH2CH2OH + concentrated H3PO4

C CH3COCH3 + HCN

D HCO2C2H5 + dilute H2SO4

23 The first propagation step in the reaction between methane and chlorine is shown.

CH4 + Cl • → CH3• + HCl

How many different first propagation steps are possible in the reaction between pentane and chlorine?

A 2 B 3 C 4 D 5

24 Alcohol Y gives product Z after mild oxidation. Z gives a positive result with Tollens’ reagent and with 2,4-dinitrophenylhydrazine reagent.

What could be the identity of alcohol Y?

A butan-1-ol B butan-2-ol C butan-2,3-diol D 2-methylbutan-2-ol

25 A student prepares pentan-1-ol by the alkaline hydrolysis of 1-iodopentane. She gently warms the reaction mixture for 20 minutes.

– – CH3CH2CH2CH2CH2I + OH → CH3CH2CH2CH2CH2OH + I

When the student uses 1-chloropentane to prepare the same alcohol she has to change the condition of the reaction.

Which change in condition should she use and what is the correct reason for its use?

change in condition reason

A heat under reflux C–Cl bond is more polar than the C–I bond B heat under reflux C–Cl bond is stronger than the C–I bond C room temperature C–Cl bond is more polar than the C–I bond D room temperature C–Cl bond is shorter than the C–I bond

© UCLES 2016 9701/11/M/J/16 9

26 Malic acid is found in apples.

OH

H C CH2CO2H

CO2H

malic acid

Which reagent will react with all three –OH groups present in the malic acid molecule?

A ethanol in the presence of concentrated sulfuric acid B potassium hydroxide C sodium D sodium carbonate

27 Cyclic esters are also known as lactones. Delta lactone is used as a solvent and in the manufacture of polyesters.

O

O

delta lactone

From which compound could delta lactone be made by a single reaction?

A HOCH2CH2CH2CH2CHO

B HOCH2CH2CH2CH2CO2H

C HOCH2CH2CH2CH2CH2OH

D HOCH2CH2CH2CH2CH2CO2H

28 Which reagent cannot be used to distinguish between ethanal and propanone?

A acidified sodium dichromate(VI) solution B alkaline aqueous iodine

C cold acidified potassium manganate(VII) solution D Fehling’s reagent

© UCLES 2016 9701/11/M/J/16 [Turn over 10

29 The ester CH3CH2CH2CO2CH2CH(CH3)2 was hydrolysed under acidic conditions.

What are the organic products of this hydrolysis?

A butanoic acid and 2-methylpropan-1-ol B butanoic acid and 2-methylpropan-2-ol C butan-1-ol and 2-methylpropanoic acid D propanoic acid and 2-methylpropan-1-ol

30 Geranyl ethanoate is present in ginger and cocoa, and is used in shampoos and soaps as a perfume. It reacts with an excess of bromine in an organic solvent to give X, a bromo-derivative.

O

Br2 O bromo-derivative

geranyl ethanoate X

Including geranyl ethanoate, how many cis-trans isomers are there of geranyl ethanoate, and how many chiral centres are there in X?

chiral centres cis-trans isomers in X

A 2 3 B 2 4 C 4 3 D 4 4

© UCLES 2016 9701/11/M/J/16 11

Section B

For each of the questions in this section, one or more of the three numbered statements 1 to 3 may be correct.

Decide whether each of the statements is or is not correct (you may find it helpful to put a tick against the statements that you consider to be correct).

The responses A to D should be selected on the basis of

A B C D

1, 2 and 3 1 and 2 2 and 3 1 only are only are only are is correct correct correct correct

No other combination of statements is used as a correct response.

31 X is a particle with 18 electrons and 20 neutrons.

What could be the symbol of X?

38 1 18 Ar

40 2+ 2 20 Ca

39 + 3 19K

32 What are basic assumptions of the kinetic theory as applied to an ideal gas?

1 Gas particles are in continuous random motion. 2 Gas particles experience no intermolecular forces. 3 The volume of each gas particle is zero.

33 Bromine reacts with water.

Br2 + H2O HOBr + HBr

Which oxidation states of bromine are present in the equilibrium mixture?

1 +3 2 0 3 –1

© UCLES 2016 9701/11/M/J/16 [Turn over 12

The responses A to D should be selected on the basis of

A B C D

1, 2 and 3 1 and 2 2 and 3 1 only are only are only are is correct correct correct correct

No other combination of statements is used as a correct response.

34 A little water is added to each of the following compounds and the mixture warmed.

For which compounds will an acidic gas be evolved?

1 aluminium chloride 2 silicon tetrachloride 3 phosphorous pentachloride

35 The element astatine, At, is below iodine in Group 17 of the Periodic Table.

Which statements concerning At are likely to be correct?

1 It is a dark-coloured solid at room temperature. 2 It is a more powerful oxidising agent than iodine. 3 Its hydride is thermally stable.

36 Which types of reaction can occur with 1-bromobutane?

1 elimination 2 hydrolysis 3 free radical substitution

© UCLES 2016 9701/11/M/J/16 13

37 The equation shows a gas phase reaction.

X(g) → 2Y(g)

The diagram shows the Boltzmann distribution of a fixed mass of X(g) at temperature T in the absence of a catalyst. The line EA indicates the activation energy.

proportion of molecules with a given energy

0 0 E A molecular energy

Which diagrams correctly show the effect of the following changes made separately and independently?

1 adding a catalyst

proportion of molecules with a given energy

0 0 E A molecular energy

2 increasing the pressure of X(g)

proportion of molecules with a given energy

0 0 E A molecular energy

3 increasing the temperature of X(g)

proportion of temperature T ' molecules (T ' > T ) with a given temperature T energy

0 0 E A molecular energy

© UCLES 2016 9701/11/M/J/16 [Turn over 14

The responses A to D should be selected on the basis of

A B C D

1, 2 and 3 1 and 2 2 and 3 1 only are only are only are is correct correct correct correct

No other combination of statements is used as a correct response.

38 The diagram shows the structure of cyclohexene.

cyclohexene

Which structures could be formed by addition reactions with cyclohexene as the only reactant?

123

n n

39 Several steps are involved in the synthesis of 2-hydroxypropanoic acid from ethanol.

C2H5OH → → → CH3CH(OH)CO2H

Which statements concerning this synthesis are correct?

1 The chain length can be increased during a step involving reaction between HCN and an aldehyde. 2 The carboxyl group can be made by hydrolysis of a nitrile by boiling with NaOH(aq) and then acidifying. 3 The ethanol should be first oxidised by heating it under reflux with an excess of acidified potassium dichromate(VI).

© UCLES 2016 9701/11/M/J/16 15

40 The diagram shows the structure of an addition polymer, X.

CH3 H CH3 H CH3 H

C C C C C C

H CO CH H CO CH H CO CH 2 3 2 3 2 3

Which reagents react with polymer X?

1 aqueous sulfuric acid 2 aqueous sodium hydroxide 3 sodium

© UCLES 2016 9701/11/M/J/16 Cambridge International Examinations Cambridge International Advanced Subsidiary and Advanced Level *3721112504*

CHEMISTRY 9701/21

Paper 2 AS Level Structured Questions May/June 2016

1 hour 15 minutes Candidates answer on the Question Paper. Additional Materials: Data Booklet

READ THESE INSTRUCTIONS FIRST

Write your Centre number, candidate number and name on all the work you hand in. Write in dark blue or black pen. You may use an HB pencil for any diagrams or graphs. Do not use staples, paper clips, glue or correction fluid. DO NOT WRITE IN ANY BARCODES.

Answer all questions. Electronic calculators may be used. You may lose marks if you do not show your working or if you do not use appropriate units. A Data Booklet is provided.

At the end of the examination, fasten all your work securely together. The number of marks is given in brackets [ ] at the end of each question or part question.

This document consists of 12 printed pages.

IB16 06_9701_21/4RP © UCLES 2016 [Turn over 2

Answer all the questions in the spaces provided.

1 (a) Complete the table to show the composition and identity of some atoms and ions.

name of nucleon atomic number of number of number of overall element number number protons neutrons electrons charge

lithium 6 3 +1 ......

oxygen 9 10 ......

54 26 26 24 ......

17 18 0 ......

[4]

(b) Beams of protons, neutrons and electrons behave differently in an electric field due to their differing properties.

The diagram shows the path of a beam of electrons in an electric field.

Add and label lines to represent the paths of beams of protons and neutrons in the same field.

electron beam

[3]

© UCLES 2016 9701/21/M/J/16 3

(c) The fifth to eighth ionisation energies of three elements in the third period of the Periodic Table are given. The symbols used for reference are not the actual symbols of the elements.

ionisation energies, kJ mol–1 fifth sixth seventh eighth X 6274 21 269 25 398 29 855 Y 7012 8496 27 107 31 671 Z 6542 9362 11 018 33 606

(i) State and explain the group number of element Y.

group number ......

explanation ......

...... [1]

(ii) State and explain the general trend in first ionisation energies across the third period.

......

......

...... [2]

(iii) Explain why the first ionisation energy of element Y is less than that of element X.

......

......

......

...... [2]

(iv) Complete the electronic configuration of elementZ .

1s2 ...... [1]

© UCLES 2016 9701/21/M/J/16 [Turn over 4

(d) A sample of strontium exists as a mixture of four isotopes. Information about three of these isotopes is given in the table.

mass number 86 87 88 abundance 9.86% 7.00% 82.58%

(i) Calculate the abundance of the fourth isotope.

abundance = ...... % [1]

(ii) The relative atomic mass of this sample of strontium is 87.71.

Calculate the mass number of the fourth isotope.

mass number = ...... [2]

[Total: 16]

© UCLES 2016 9701/21/M/J/16 5

2 D , E, F, and G are four consecutive elements in the fourth period of the Periodic Table. (The letters are not the actual symbols of the elements.)

D is a soft, silvery metal with a melting point just above room temperature. Its amphoteric oxide,

D2O3, has a melting point of 1900 °C and can be formed by heating D in oxygen.

G is a solid that can exist as several different allotropes, most of which contain G8 molecules.

G burns in air to form GO2 which dissolves in water to form an acidic solution. This solution reacts

with sodium hydroxide to form the salt Na2GO3.

(a) Suggest the identities of D and G.

D ...... G ...... [1]

(b) Write equations for the reactions of D2O3 with

(i) hydrochloric acid,

...... [2]

(ii) sodium hydroxide.

...... [2]

(c) Suggest the type of bonding and structure in D2O3.

...... [1]

(d) Write an equation for the formation of an acidic solution when GO2 dissolves in water.

...... [1]

[Total: 7]

© UCLES 2016 9701/21/M/J/16 [Turn over 6

3 The elements in Group 2, and their compounds, show many similarities and trends in their properties.

(a) Magnesium, calcium, strontium and barium all react with cold water.

(i) Describe what you would see when some calcium is added to cold water.

......

......

...... [3]

(ii) Write an equation for the reaction taking place in (i).

...... [1]

(iii) Describe how the reaction of barium with cold water would differ from the reaction of calcium in (i) in terms of what you would see.

......

...... [1]

(b) Magnesium oxide can be formed by the reaction of magnesium and oxygen in the air.

(i) Draw a fully labelled reaction pathway diagram for the reaction between magnesium and oxygen.

energy

reaction pathway [2]

(ii) Explain why there is no visible reaction when a piece of magnesium ribbon is exposed to the air.

......

......

...... [2]

© UCLES 2016 9701/21/M/J/16 7

(iii) Magnesium oxide is used to manufacture heat-resistant bricks for furnace linings in the steel-making industry.

State and explain the property of magnesium oxide that makes it suitable for this use.

......

......

......

...... [2]

(iv) Suggest a reason why magnesium oxide cannot be used as a lining for any furnaces containing acidic materials.

......

...... [1]

(c) The nitrates and carbonates of the Group 2 elements, from magnesium to barium, decompose when heated.

(i) State the trend in the temperature of thermal decomposition of these Group 2 nitrates and carbonates.

......

...... [1]

(ii) Give the equation for the thermal decomposition of magnesium carbonate.

...... [1]

(iii) Give the equation for the thermal decomposition of calcium nitrate.

...... [1]

[Total: 15]

© UCLES 2016 9701/21/M/J/16 [Turn over 8

4 This question is about molecules with molecular formula C4H8.

(a) Give the structures of a pair of positional isomers with the formula C4H8.

[1]

(b) Give the structures of a pair of chain isomers with the formula C4H8, that do not exhibit stereoisomerism.

[1]

(c) Give the structures and full names of a pair of stereoisomers with the formula C4H8.

...... [2]

(d) The structure of a molecule, A, of formula C4H8 is shown.

Draw a functional group isomer of molecule A in box B. Explain how molecules A and B could be distinguished by a chemical test.

H2C CH2

H2C CH2

A B

......

......

...... [3]

[Total: 7] © UCLES 2016 9701/21/M/J/16 10

5 A reaction sequence is shown.

O reaction 1 reaction 2 H3CCH2 Br H3CCH2 CN H3CCH2 C bromoethane propanenitrile OH reaction reaction propanoic acid 4 5

LiAl H4 reaction 3 H3CCH2 OH H2CCH2 ethanol ethene W

(a) Complete the diagram to show the mechanism of reaction 1. Include all necessary charges, partial charges, lone pairs and curly arrows.

H H

H3CC Br H3CC CN +

H H

–CN [2]

(b) (i) Give the name of the type of reaction involved in reaction 3.

...... [1]

The infra-red spectrum of the propanoic acid produced by reaction 2 is shown.

100

transmittance 50

0 4000 3000 2000 1500 1000 500 wavenumber / cm–1

(ii) Describe and explain the main difference between the infra-red spectrum of W and that of propanoic acid.

......

...... [2]

© UCLES 2016 9701/21/M/J/16 11

(c) (i) Reactions 4 and 5 use the same reagent.

Give the reagent and conditions needed for reaction 4.

reagent ......

conditions ...... [2]

(ii) Give the conditions needed for reaction 5.

...... [1]

(d) Under appropriate conditions, ethanol and propanoic acid undergo a condensation reaction.

(i) State the condition necessary for the reaction.

...... [1]

(ii) Draw the skeletal formula of the organic product of this reaction.

[1]

(iii) Name the organic product of this reaction.

...... [1]

Question 5 continues over the page.

© UCLES 2016 9701/21/M/J/16 [Turn over 12

(e) V reacts with acidified manganate(VII) ions in two different ways depending on the conditions, as shown in the reaction sequence below.

O hot, concentrated cold, dilute H3CCH2 C – + V – + T MnO4 / H MnO4 / H OH propanoic acid

V decolourises bromine water.

When the acidified manganate(VII) is hot and concentrated, propanoic acid is the only organic product.

When the acidified manganate(VII) is cold and dilute, the organic product is T which has two chiral centres.

(i) Give the structural formulae of V and T.

V ...... T ...... [2]

(ii) Identify the types of stereoisomerism shown by V and T.

V ...... T ...... [2]

[Total: 15]

Permission to reproduce items where third-party owned material protected by copyright is included has been sought and cleared where possible. Every reasonable effort has been made by the publisher (UCLES) to trace copyright holders, but if any items requiring clearance have unwittingly been included, the publisher will be pleased to make amends at the earliest possible opportunity.

To avoid the issue of disclosure of answer-related information to candidates, all copyright acknowledgements are reproduced online in the Cambridge International Examinations Copyright Acknowledgements Booklet. This is produced for each series of examinations and is freely available to download at www.cie.org.uk after the live examination series.

Cambridge International Examinations is part of the Cambridge Assessment Group. Cambridge Assessment is the brand name of University of Cambridge Local Examinations Syndicate (UCLES), which is itself a department of the University of Cambridge.

© UCLES 2016 9701/21/M/J/16

Cambridge International Examinations Cambridge International Advanced Subsidiary and Advanced Level

CHEMISTRY 9701/11

Paper 1 Multiple Choice May/June 2017

1 hour

Additional Materials: Multiple Choice Answer Sheet *35 Soft clean eraser Soft pencil (type B or HB is recommended)

0 Data Booklet 9 3

0 READ THESE INSTRUCTIONS FIRST 7

7

4 Write in soft pencil. 8 Do not use staples, paper clips, glue or correction fluid. * Write your name, Centre number and candidate number on the Answer Sheet in the spaces provided unless this has been done for you. DO NOT WRITE IN ANY BARCODES.

There are forty questions on this paper. Answer all questions. For each question there are four possible answers A, B, C and D. Choose the one you consider correct and record your choice in soft pencil on the separate Answer Sheet.

Read the instructions on the Answer Sheet very carefully.

Each correct answer will score one mark. A mark will not be deducted for a wrong answer. Any rough working should be done in this booklet. Electronic calculators may be used.

This document consists of 13 printed pages and 3 blank pages.

IB17 06_9701_11_VI_BLU/FP © UCLES 2017 [Turn over

2

Section A

For each question there are four possible answers, A, B, C and D. Choose the one you consider to be correct.

Use of the Data Booklet may be appropriate for some questions.

1 Which molecule contains six bonding electrons?

A C2H4 B H2S C NCl 3 D SF6

2 The mass spectrum of a sample of lithium shows that it contains two isotopes, 6Li and 7Li.

The isotopic abundances are shown in the table.

isotope isotopic abundance

6Li 7.42% 7Li 92.58%

What is the relative atomic mass of this sample of lithium, given to three significant figures?

A 6.07 B 6.50 C 6.90 D 6.93

2 3 A sports medal has a total surface area of 150 cm . It was evenly coated with silver by electrolysis. Its mass increased by 0.216 g.

How many atoms of silver were deposited per cm2 on the surface of the medal?

A 8.0 × 1018 B 1.8 × 1019 C 8.7 × 1020 D 1.2 × 1021

4 Which property of an atom does not affect its first ionisation energy?

A the atomic radius B the number of electron shells C the number of neutrons D the number of protons

5 Which molecule has the largest overall dipole?

A B C D

l l H CH3 CH3 C H3C C CC OC OC CC

H CH CH Cl Cl CH 3 3 3

© UCLES 2017 9701/11/M/J/17 3

3 6 The complete combustion of 2 moles of a straight chain alkane produces 400 dm of 5 carbon dioxide measured at 301 K and 1 × 10 Pa. Carbon dioxide can be assumed to behave as an ideal gas under these conditions.

What is the formula of the straight chain alkane?

A C8H18 B C16H34 C C20H42 D C40H82

7 Which expression gives the standard enthalpy change of combustion of methane?

A (CH4) + (CO2) – 2 (H2O)

B (CO2) + 2 (H2O) + (CH4)

C (CH4) + 2 (H2O) – (CO2)

D (CO2) + 2 (H2O) – (CH4)

8 Solutions containing chlorate(I) ions are used as household bleaches and disinfectants. These solutions decompose on heating as shown.

– – – 3Cl O → Cl O3 + 2Cl

Which oxidation state is shown by chlorine in each of these three ions?

– – – Cl O Cl O3 Cl

A +1 +3 –1 B –1 +3 +1 C +1 +5 –1 D –1 +5 +1

9 When K2MnO4 is dissolved in water, the following reaction occurs.

2– – – a MnO4 (aq) + b H2O(l) → c MnO4 (aq) + d MnO2(s) + e OH (aq)

What are the values of a and c in the balanced chemical equation?

a c

A 2 1 B 3 2 C 4 3 D 5 4

© UCLES 2017 9701/11/M/J/17 [Turn over 4

10 Methanol can be produced from hydrogen and carbon monoxide.

2H2(g) + CO(g) CH3OH(g)

What is the expression for K p for this reaction?

(2p )2 × p H2 CO A K p = p CH3OH

(p )2 × p H2 CO B K p = p CH3OH p CH3OH C K p = (p )2 × p H2 CO p CH3OH D K p = p × (2p )2 CO H2

3 –3 11 When 4 g of powdered calcium carbonate, Mr = 100, were added to 100 cm of 0.10 mol dm hydrochloric acid the volume of carbon dioxide produced was recorded.

time / s 30 60 90 120 150 180 210 240 total volume of carbon 3 40 70 88 101 110 116 120 120 dioxide given off / cm

Which row gives the correct explanations about these results?

why the rate of the why the reaction stops reaction changes with time A fewer collisions between the calcium carbonate is used up reacting molecules occur B fewer collisions between the hydrochloric acid is used up reacting molecules occur C more collisions between the calcium carbonate is used up reacting molecules occur D more collisions between the hydrochloric acid is used up reacting molecules occur

© UCLES 2017 9701/11/M/J/17 5

12 Why is the second ionisation energy of sodium larger than the second ionisation energy of magnesium?

A The attraction between the nucleus and the outer electron is greater in Na+ than in Mg+. B The nuclear charge of Na+ is greater than that of Mg+. C The outer electron of Na+ is more shielded than the outer electron of Mg+. D The outer electron of Na is in the same orbital as the outer electron of Mg.

13 Which graph correctly shows the relative melting points of the elements Mg, Al, Si and P plotted against their relative electronegativities?

A B

Si Al

melting point melting point l Mg Si Mg A

P P

electronegativity electronegativity

C D Si

Al

melting point Al Mg melting point Mg Si P P

electronegativity electronegativity

© UCLES 2017 9701/11/M/J/17 [Turn over 6

14 An excess of MgO is shaken with water. The resulting mixture is filtered into test-tube X.

An excess of BaO is shaken with water. The resulting mixture is filtered into test-tube Y.

Which oxide reacts more readily with water and which filtrate has the lower pH?

oxide reacts more test-tube with

readily with water filtrate of lower pH

A barium oxide X B barium oxide Y C magnesium oxide X D magnesium oxide Y

15 Samples of magnesium carbonate, MgCO3, are placed in crucibles R and S. The sample in crucible R is heated until there is no further loss in mass, and then allowed to cool. The sample in crucible S is left unheated.

Dilute hydrochloric acid is then added to both crucibles.

On adding the dilute hydrochloric acid, which observations are correct?

R S

A gas produced gas produced B gas produced no gas produced C no gas produced gas produced D no gas produced no gas produced

16 Which statement about nitrogen or its compounds is correct?

A In the Haber process the temperature is kept high to give a good equilibrium yield of ammonia. B Nitrogen gas is unreactive because of the strong nitrogen-nitrogen double bond. C Nitrogen monoxide will react with carbon monoxide under suitable conditions.

D The formula of ammonium sulfate is NH4SO4.

© UCLES 2017 9701/11/M/J/17 7

17 When concentrated sulfuric acid reacts with sodium iodide the products include sulfur, iodine, hydrogen sulfide and sulfur dioxide.

Which statement is correct?

A Hydrogen sulfide is the product of a reduction reaction. B Iodide ions are stronger oxidising agents than sulfate ions. C Sulfur atoms from the sulfuric acid are both oxidised and reduced. D Sulfur atoms from the sulfuric acid are oxidised to make sulfur dioxide.

18 A solution of sodium hydroxide reacts with 3 mol of chlorine under certain conditions. The reaction produces 5 mol of sodium chloride and 1 mol of X, the only other chlorine-containing product.

What is the formula of compound X?

A NaCl O B NaCl O2 C NaCl O3 D NaCl O4

19 Redox reactions are common in the chemistry of Group 17.

Which statement is correct?

– A Br ions will reduce Cl 2 but not I2.

– – B Cl 2 will oxidise Br ions but not I ions.

C F2 is the weakest oxidising agent out of F2, Cl 2, Br2 and I2.

– – – – – D I ions are the weakest reducing agent out of F , Cl , Br and I .

20 Structural isomerism and stereoisomerism should be considered when answering this question.

Each of the following carbonyl compounds is reacted with NaBH4. The product of each reaction is heated with Al 2O3 at 600 °C, generating one product or a mixture of isomers.

Which carbonyl compound will produce the most isomers?

A butanal B butanone C pentan-3-one D propanone

© UCLES 2017 9701/11/M/J/17 [Turn over 8

21 The drug cortisone has the formula shown.

OH

OH O O

O

In addition to those chiral centres marked by an asterisk (*), how many other chiral centres are present in the cortisone molecule?

A 0 B 1 C 2 D 3

– 22 An alkene is reacted with acidified manganate(VII) ions, MnO4 . The desired organic product has a relative molecular mass greater than that of the alkene by 34.

What conditions should be used?

– A cold, concentrated MnO4

– B cold, dilute MnO4

– C hot, concentrated MnO4

– D hot, dilute MnO4

23 The diagram shows a short length of an addition polymer chain.

H H H H

C C C C

H H H H

The polymer has a relative molecular mass of approximately 10 000.

Approximately how many monomer units are joined together in each polymer molecule?

A 180 B 360 C 625 D 710

© UCLES 2017 9701/11/M/J/17 9

24 Lactide is an intermediate in the manufacture of a synthetic fibre.

O O

O O

lactide

Which compound, on heating with an acid catalyst, can produce lactide?

A hydroxyethanoic acid B 2-hydroxybutanoic acid C 2-hydroxypropanoic acid D 3-hydroxypropanoic acid

25 Diols in which both hydroxy groups are bonded to the same carbon atom spontaneously eliminate a molecule of water to produce a carbonyl compound.

Which compound is hydrolysed to form a product that gives a positive reaction with 2,4-dinitrophenylhydrazine but not with Fehling’s reagent?

A 1,1-dibromopropane B 1,2-dibromopropane C 1,3-dibromopropane D 2,2-dibromopropane

26 X and Y are the reagents required to convert 1-bromopropane into butanoic acid.

H H H H H H H H H O X Y H C C C BrH C C C CN H C C C C OH H H H H H H H H H

What are the correct identities of X and Y?

X Y

A NH3 HCl (aq)

B KCN in C2H5OH NaOH(aq)

C KCN in C2H5OH HCl (aq) D HCN NaOH(aq)

© UCLES 2017 9701/11/M/J/17 [Turn over 10

27 Q is a compound with the molecular formula C4H10O. Q can be oxidised with acidified potassium dichromate(VI). Q cannot be made by reducing a carboxylic acid with LiAl H4.

What is the structure of Q?

A CH3CH(OH)CH2CH3

B CH3CH2CH2CH2OH

C (CH3)3COH

D (CH3)2CHCH2OH

28 A sample of 2.30 g of ethanol was mixed with an excess of aqueous acidified potassium dichromate(VI). The reaction mixture was then boiled under reflux for one hour. The required organic product was then collected by distillation. The yield of product was 60.0%.

Which mass of product was collected?

A 1.32 g B 1.38 g C 1.80 g D 3.00 g

29 Which compound gives a positive test with alkaline aqueous iodine and does not show optical isomerism?

A CH3COCH2CH2OH

B CH3CH2CH(OH)CHO

C CH3COCH(OH)CH3

D (CH3)2C(OH)CHO

30 Citral is found in lemongrass oil. It can react to give compound W.

CHO an excess of hot, concentrated VW VII H2 / Ni acidified manganate( ) under reflux

citral

What could compound W be?

A B C D

CHO CO2H CO2H O CO2H

CO2H

© UCLES 2017 9701/11/M/J/17 11

Section B

For each of the questions in this section, one or more of the three numbered statements 1 to 3 may be correct.

Decide whether each of the statements is or is not correct (you may find it helpful to put a tick against the statements that you consider to be correct).

The responses A to D should be selected on the basis of

A B C D

1, 2 and 3 1 and 2 2 and 3 1 only are only are only are is correct correct correct correct

No other combination of statements is used as a correct response.

Use of the Data Booklet may be appropriate for some questions.

31 Beams of charged particles are deflected by an electric field. In identical conditions the angle of deflection of a particle is proportional to its charge / mass ratio.

In an experiment, protons are deflected by an angle of +15°. In another experiment under identical conditions, particle Y is deflected by an angle of –5°.

What could be the composition of particle Y?

protons neutrons electrons

1 1 2 2 2 3 3 5 3 4 5 1

32 Graphene, graphite and the fullerene C60 are allotropes of carbon.

Which statements are correct for all three of these allotropes of carbon?

1 Delocalised electrons are present in the structure.

2 All bond angles are 120°. 3 It has a giant molecular crystalline lattice structure.

© UCLES 2017 9701/11/M/J/17 [Turn over 12

The responses A to D should be selected on the basis of

A B C D

1, 2 and 3 1 and 2 2 and 3 1 only are only are only are is correct correct correct correct

No other combination of statements is used as a correct response.

33 A reaction between carbon and oxygen is shown.

1 C(s) + 2 O2(g) → CO(g)

How can the enthalpy change of this reaction be described correctly?

1 enthalpy change of formation 2 enthalpy change of combustion 3 enthalpy change of atomisation

34 Which changes can be used to measure the rates of chemical reactions?

1 the decrease in concentration of a reactant per unit time 2 the rate of appearance of a product 3 the increase in total volume per unit time at constant pressure

35 Which statements describe a trend in Period 3 between every pair of adjacent elements from sodium to chlorine?

1 The atomic radius decreases. 2 The 1st ionisation energy decreases. 3 The melting point decreases.

36 X is either nitrogen or sulfur and forms pollutant oxide Y in a car engine.

Further oxidation of Y to Z occurs in the atmosphere. In this further oxidation, 1 mol of Y reacts with 0.5 mol of gaseous oxygen molecules.

Which statements about X, Y and Z can be correct?

1 The oxidation number of X increases by two from Y to Z. 2 Y has an unpaired electron in its molecule. 3 Y is a polar molecule.

© UCLES 2017 9701/11/M/J/17 13

37 P and Q are a pair of cis-trans isomers.

What must be the same for P and Q?

1 their empirical formula 2 their functional groups 3 their skeletal formula

38 The following statements are about the reaction of NaOH(aq) with the three chloroalkanes shown.

CH3CH2CHCl CH3 (CH3)2CHCH2Cl (CH3)3CCl

Which statements are correct?

1 (CH3)2CHCH2Cl reacts with NaOH(aq) by an SN2 mechanism. 2 The tertiary chloroalkane reacts more quickly than the others because the carbon atom bonded to the Cl atom is more positive in this molecule. 3 The Cl atoms in the three chloroalkanes are attacked by OH–.

39 For which reactions are the colour changes described correctly?

reagents colour change

1 pentanal + hot, acidified potassium dichromate(VI) orange to green 2 pentan-2-one + warm Fehling’s reagent no change 3 cyclohexane + cold, acidified potassium manganate(VII) purple to colourless

40 Which statements about ethanol and ethanoic acid are correct?

1 Both react with a suitable reagent to form an ester. 2 Both react with sodium. 3 Both are soluble in water.

© UCLES 2017 9701/11/M/J/17 Cambridge International Examinations Cambridge International Advanced Subsidiary and Advanced Level *7040470612*

CHEMISTRY 9701/21

Paper 2 AS Level Structured Questions May/June 2017

1 hour 15 minutes Candidates answer on the Question Paper. Additional Materials: Data Booklet

READ THESE INSTRUCTIONS FIRST

Write your Centre number, candidate number and name on all the work you hand in. Write in dark blue or black pen. You may use an HB pencil for any diagrams or graphs. Do not use staples, paper clips, glue or correction fluid. DO NOT WRITE IN ANY BARCODES.

Answer all questions. Electronic calculators may be used. You may lose marks if you do not show your working or if you do not use appropriate units. A Data Booklet is provided.

At the end of the examination, fasten all your work securely together. The number of marks is given in brackets [ ] at the end of each question or part question.

This document consists of 10 printed pages and 2 blank pages.

IB17 06_9701_21/FP © UCLES 2017 [Turn over 2

Answer all the questions in the spaces provided.

1 Combustion data can be used to calculate the empirical formula, molecular formula and relative molecular mass of many organic compounds.

(a) Define the term relative molecular mass.

......

......

......

...... [2]

3 (b) T is an alcohol, CxHyO. A gaseous sample of T occupied a volume of 20 cm at 120 °C and 100 kPa.

The sample was completely burned in 200 cm3 of oxygen (an excess). The final volume, measured under the same conditions as the gaseous sample, was 250 cm3.

Under these conditions, all water present is vaporised. Removal of the water vapour from the gaseous mixture decreased the volume to 170 cm3.

Treating the remaining gaseous mixture with concentrated alkali, to absorb carbon dioxide, decreased the volume to 110 cm3.

The equation for the complete combustion of T can be represented as shown.

y C H O + zO xCO + H O x y 2 2 2 2 (i) Use the data given to calculate the value of x.

x = ...... [1]

(ii) Use the data given to calculate the value of y.

y = ...... [1]

© UCLES 2017 9701/21/M/J/17 3

If you were unable to calculate values for x and y then use x = 4 and y = 10 for the remaining parts of this question. These are not the correct values.

(iii) Complete the equation for the complete combustion of the alcohol, T.

...... + ...... O2 ...... CO2 + ...... H2O [1]

(iv) Give the skeletal formulae for two possible structures of T.

Name each alcohol.

...... [2]

(v) Use the general gas equation to calculate the mass of T present in the original 20 cm3 gaseous sample, which was measured at 120 °C and 100 kPa.

Give your answer to three significant figures. Show your working.

mass = ...... g [3]

[Total: 10]

© UCLES 2017 9701/21/M/J/17 [Turn over 4

2 Structure and bonding can be used to explain many of the properties of substances.

(a) Copper, ice, silicon(IV) oxide, iodine and sodium chloride are all crystalline solids.

Complete the table with: ● the name of a type of bonding found in each crystalline solid, ● the type of lattice structure for each crystalline solid.

crystalline solid type of bonding type of lattice structure

copper

ice

silicon(IV) oxide

iodine

sodium chloride [5]

(b) (i) Name the strongest type of intermolecular force in ice.

...... [1]

(ii) Draw a fully labelled diagram of two water molecules in ice, showing the force in (i) and how it forms.

[3]

© UCLES 2017 9701/21/M/J/17 5

(c) The graph represents how the temperature of a sample of copper (melting point 1085 °C) changes as it is gradually cooled from 1200 °C.

1200

temperature, T / °C T1

XY Z

time / t

(i) Identify the state(s) of matter present during each stage of the process shown in the graph.

X ......

Y ......

Z ...... [2]

(ii) State what is happening to the energy and movement of the particles in the copper during stage X.

......

......

...... [2]

(iii) Explain why the temperature stays constant at T1 during stage Y.

......

......

......

...... [2]

[Total: 15]

© UCLES 2017 9701/21/M/J/17 [Turn over 6

3 The properties of elements and their compounds show similarities, differences and trends depending on the positions of the elements in the Periodic Table.

(a) The positions of some elements are indicated. The letters used are not the symbols of the elements.

E B H D A

FG C

From the elements labelled, give the letter for;

(i) the element that forms an amphoteric oxide, ...... [1]

(ii) the element with the highest first ionisation energy, ...... [1]

(iii) the element that forms a soluble hydroxide and an insoluble sulfate, ...... [1]

(iv) the most volatile element in a group that contains elements in all three states of matter at room temperature and pressure, ...... [1]

(v) the element that forms the largest cation...... [1]

(b) The elements in Group 2 all react with oxygen and with water.

(i) State and explain the conditions needed for magnesium to react with oxygen.

......

...... [2]

(ii) State what would be seen during the reaction in (b)(i).

......

...... [1]

(iii) Write an equation for the reaction of magnesium with cold water. Include state symbols.

...... [2]

© UCLES 2017 9701/21/M/J/17 7

(c) The carbonates and nitrates of the elements in Group 2 can all be decomposed by heating.

(i) Write an equation for the thermal decomposition of magnesium nitrate.

...... [1]

(ii) The thermal decomposition of calcium carbonate forms a solid product that is industrially important. This solid product reacts with water to form a compound commonly known as slaked lime.

Write equations for the thermal decomposition of calcium carbonate and the reaction of the solid product to form slaked lime.

thermal decomposition ......

formation of slaked lime ...... [2]

(d) Calcium carbonate and calcium hydroxide both have an important use in agriculture.

(i) Describe this use and explain what makes these two compounds suitable for it.

......

......

...... [2]

(ii) Write an ionic equation to illustrate this use of calcium carbonate.

...... [1]

[Total: 16]

© UCLES 2017 9701/21/M/J/17 [Turn over 8

4 P, Q and R all have the molecular formula C3H6O. They are all structural isomers of each other.

P and Q each contain an oxygen atom bonded directly to a carbon atom that is sp2 hybridised. R contains an oxygen atom bonded directly to a carbon atom that is sp3 hybridised.

(a) (i) Explain the meaning of the term structural isomers.

......

......

......

...... [2]

(ii) Explain how sp2 and sp3 hybridisation can occur in carbon atoms.

sp2 hybridisation ......

......

sp3 hybridisation ......

...... [2]

(iii) State the bond angles normally associated with each type of hybridisation in carbon atoms.

sp2 ......

sp3 ...... [2]

(b) R contains two different functional groups, one of which is an alkene group. R reacts with cold, dilute, acidified manganate(VII) ions to form propane-1,2,3-triol.

H H H

HO C C C OH

H OH H

propane-1,2,3-triol

(i) Give the displayed formula of R.

[1]

© UCLES 2017 9701/21/M/J/17 9

(ii) State the type of reaction and what you would observe when R reacts with bromine water.

......

...... [2]

(iii) Draw the structure of the product formed when R reacts with bromine water.

[1]

(iv) Identify the gaseous product formed when R reacts with hot, concentrated, acidified manganate(VII) ions.

...... [1]

(c) P and Q (C3H6O) both form an orange precipitate when reacted with 2,4-DNPH. Only Q produces a yellow precipitate when reacted with alkaline aqueous iodine.

(i) Name P and Q.

P ......

Q ...... [2]

(ii) Identify the yellow precipitate formed by the reaction of Q with alkaline aqueous iodine.

...... [1]

(d) P and Q each react with hydrogen cyanide to form a single product. The product formed from P exists as a pair of optical isomers. The product formed from Q does not exhibit optical isomerism.

(i) Explain the meaning of the term optical isomers.

......

......

......

...... [2]

© UCLES 2017 9701/21/M/J/17 [Turn over 10

(ii) Ethanal, CH3CHO, also reacts with hydrogen cyanide. The product of this reaction is

CH3CH(OH)CN.

Draw the mechanism of this reaction. Include all necessary charges, dipoles, lone pairs and curly arrows.

[3]

[Total: 19]

© UCLES 2017 9701/21/M/J/17

Cambridge International Examinations Cambridge International Advanced Subsidiary and Advanced Level

CHEMISTRY 9701/11

Paper 1 Multiple Choice May/June 2018

1 hour

Additional Materials: Multiple Choice Answer Sheet *2218847432* Soft clean eraser Soft pencil (type B or HB is recommended) Data Booklet

READ THESE INSTRUCTIONS FIRST

Write in soft pencil. Do not use staples, paper clips, glue or correction fluid. Write your name, Centre number and candidate number on the Answer Sheet in the spaces provided unless this has been done for you. DO NOT WRITE IN ANY BARCODES.

There are forty questions on this paper. Answer all questions. For each question there are four possible answers A, B, C and D. Choose the one you consider correct and record your choice in soft pencil on the separate Answer Sheet.

Read the instructions on the Answer Sheet very carefully.

Each correct answer will score one mark. A mark will not be deducted for a wrong answer. Any rough working should be done in this booklet. Electronic calculators may be used.

This document consists of 14 printed pages and 2 blank pages.

IB18 06_9701_11/2RP © UCLES 2018 [Turn over

2

Section A

For each question there are four possible answers, A, B, C and D. Choose the one you consider to be correct.

Use of the Data Booklet may be appropriate for some questions.

1 This question refers to isolated gaseous atoms.

In which atom are all electrons paired?

A Ba B Br C S D Si

2 Which compound has a boiling point that is influenced by hydrogen bonding?

A CH3CHO B CH3OCH3 C HCO2CH3 D HCO2H

3 Which fuel would produce the largest mass of CO2 when 10 kg of the fuel undergo complete combustion?

A biodiesel, C17H34O2

B ethanol, C2H6O

C , C8H18

D propane, C3H8

4 The diagram shows the Boltzmann distribution of energies in a gas. The gas can take part in a reaction with an activation energy, Ea. The gas is maintained at a constant temperature.

P

proportion of molecules with a given energy

0 E 0 a molecular energy

Which statement is correct?

A If a catalyst is added, peak P will be lower and Ea will move to the left.

B If a catalyst is added, peak P will be lower and Ea will move to the right.

C If a catalyst is added, peak P will be the same and Ea will move to the left.

D If a catalyst is added, peak P will be the same and Ea will move to the right.

© UCLES 2018 9701/11/M/J/18 3

5 Which molecule has no overall dipole?

A CH3Cl B CH2Cl 2 C CHCl 3 D CCl 4

6 Which solid contains more than one type of bonding?

A iodine B silicon dioxide C sodium chloride D zinc

7 Enthalpy changes of combustion can be used to determine enthalpy changes of formation. The following equation represents the enthalpy change of formation of butane.

4C(s) + 5H2(g) → C4H10(g)

By using the following standard enthalpy of combustion data, what is the value of the standard enthalpy change of formation, , of butane?

substance C(s) –394

H2(g) –286

C4H10(g) –2877

–1 A –5883 kJ mol

–1 B –129 kJ mol

–1 C +129 kJ mol

–1 D +2197 kJ mol

2– 8 Ethanedioate ions, C2O4 , react with a suitable reagent to form CO2. A half-equation for this reaction is shown.

2– – C2O4 → 2CO2 + 2e

Which row is correct?

oxidation state of 2– type of reaction carbon in C2O4

A +3 oxidation B +3 reduction C +5 oxidation D +5 reduction

© UCLES 2018 9701/11/M/J/18 [Turn over 4

9 Oxidation numbers should be used to answer this question.

+ A redox reaction takes place between hydroxylammonium ions, [NH3OH] , and acidified iron(III) ions, Fe3+. The products are iron(II) ions, Fe2+, H+ ions, water and a compound of nitrogen.

The mole ratio of reacting hydroxylammonium ions to reacting iron(III) ions is 1 : 2.

Which nitrogen-containing compound could be formed in the reaction?

A NH3 B N2O C NO D NO2

10 Element X has a higher first ionisation energy than element Y.

Two students state what they believe is one factor that helps to explain this.

student 1 “X has a higher first ionisation energy than Y because an atom of X has more protons in its nucleus than an atom of Y.”

student 2 “X has a higher first ionisation energy than Y because X has a smaller atomic radius than Y.”

Only one of the two students is correct.

What could X and Y be?

X Y

A carbon boron B magnesium aluminium C oxygen nitrogen D oxygen sulfur

11 Hydrogen ions catalyse the hydrolysis of esters.

Which statement is correct?

A The hydrogen ions act as a heterogeneous catalyst. B The hydrogen ions are in the same phase as the reactants. C The hydrogen ions are used up in the reaction. D The hydrogen ions have no effect on the activation energy of the reaction.

© UCLES 2018 9701/11/M/J/18 5

12 Silicon is heated in an excess of chlorine, producing compound J.

Excess water is added to the sample of J produced.

Which row is correct?

Is HCl produced structure of J when water is added to J?

A giant molecular no B giant molecular yes C simple molecular no D simple molecular yes

13 Which element has the second smallest atomic radius in its group and the third lowest first ionisation energy in its period?

A boron B calcium C magnesium D sodium

14 Chlorine reacts with cold aqueous sodium hydroxide to produce sodium chloride, water and compound X.

Chlorine reacts with hot aqueous sodium hydroxide to produce sodium chloride, water and compound Y.

What are the oxidation states of chlorine in compound X and compound Y?

X Y

A –1 –5 B –1 +5 C +1 –5 D +1 +5

15 In which reaction does ammonia behave as a Brønsted-Lowry base?

A NH3 + CH3CH2Br → CH3CH2NH2 + HBr

B NH3 + H2O + CO2 → (NH4)HCO3

C 2NH3 + 2Na → 2NaNH2 + H2

D 4NH3 + 3O2 → 2N2 + 6H2O

© UCLES 2018 9701/11/M/J/18 [Turn over 6

16 Aqueous silver nitrate is added to a solution of potassium iodide.

Aqueous ammonia is then added.

What would be observed?

A a cream precipitate that dissolves on addition of aqueous ammonia B a cream precipitate that does not dissolve on addition of aqueous ammonia C a yellow precipitate that dissolves on addition of aqueous ammonia D a yellow precipitate that does not dissolve on addition of aqueous ammonia

17 Oxides of nitrogen are present in the environment due to natural and man-made sources.

Which row is correct?

natural source of nitrogen oxides man-made source of nitrogen oxides

A electrical discharges in the atmosphere internal combustion engines B electrical discharges in the atmosphere as a by-product of the Haber process C decomposition of dead plants in rivers internal combustion engines D decomposition of dead plants in rivers as a by-product of the Haber process

18 When 3.00 g of an anhydrous nitrate of a Group 2 metal is decomposed, 1.53 g of gas is produced.

What is the nitrate compound?

A beryllium nitrate B calcium nitrate C magnesium nitrate D strontium nitrate

19 Which row correctly describes one property of barium and one property of barium oxide?

observation when pH of solution obtained when barium metal is a spatula measure of BaO is 3 added to water added to 100 cm of water

A a few gas bubbles 8 form on the metal surface B a few gas bubbles 13 form on the metal surface C rapid effervescence is seen 8 D rapid effervescence is seen 13

© UCLES 2018 9701/11/M/J/18 7

20 Fructose is a sugar with more than one chiral centre. The fructose molecule is shown with X, Y and Z indicating three carbon atoms.

OH

HHC

X C O

HOC H

HOHC

Y HOHC

HHC

Z OH

fructose

Which carbon atoms are chiral centres?

A X, Y and Z B X and Y only C X only D Y only

21 Which pair of alcohols are isomers of each other?

A butan-1-ol and 2,2-dimethylpropan-1-ol B butan-2-ol and 2-methylpropan-2-ol C pentan-1-ol and 2-methylpropan-2-ol D propan-2-ol and 2-methylpropan-2-ol

+ – 22 Sodium methoxide, Na CH3O , reacts with 2-chloro-2-methylpropane in a nucleophilic – substitution reaction. The nucleophile is the CH3O ion.

Which row is correct?

intermediate or product transition state

+ A (CH3)3C (CH3)3COCH3

+ B (CH3)3C (CH3)3CCH2OH

– C [HOCH2- - -C(CH3)3- - -Cl ] HOCH2C(CH3)3

– D [H3CO- - -C(CH3)3- - -Cl ] H3COC(CH3)3

© UCLES 2018 9701/11/M/J/18 [Turn over 8

23 Which alcohol can be dehydrated to give two products which are structural isomers of each other?

A B C D

H H H H H H CH3 H HH H CH3

H C C C COHH C C C OH H C C C OH H C C OH

H H H H H H H H CH H H CH 3 3

24 Which reagent could detect the presence of alcohol in a mixture consisting mainly of alkanes and alkenes?

A Na

B Br2 (in CCl 4)

C KMnO4(aq) D 2,4-dinitrophenylhydrazine

25 Compound Q

● contains a chiral centre, ● gives a positive result with Fehling’s reagent, ● gives a positive result with alkaline aqueous iodine.

What could compound Q be?

A 1-hydroxybutanone B 2-hydroxybutanal C 3-hydroxybutanal D 3-hydroxybutanone

© UCLES 2018 9701/11/M/J/18 9

26 What is the mechanism for the reaction of ethanal, CH3CHO, with hydrogen cyanide, HCN, in the presence of NaCN?

O O– O

A – CH3 CH CH3 CH CH3 C+H

CN– CN CN

O –O HCN OH

B – CH3 CH CH3 CH CH3 CH + CN

CN– CN CN

O HCN OH OH

C + CH3 C H CH3 CH CH3 CH

CN– CN

D initiation HCN H• +CN•

O O•

propagation CH3 C + •CN CH3 C H

H CN

O• OH

CH3 C H + H CN CH3 C H + •CN

CN CN

O• OH

termination CH3 C H + H• CH3 C H

CN CN

© UCLES 2018 9701/11/M/J/18 [Turn over 10

27 Ethyl propanoate is refluxed with aqueous sodium hydroxide. The alcohol produced is then reacted with methyl propanoic acid to make a second ester.

What is the structural formula of this second ester?

A B O O

CH3 CH2 O C CH CH3 CH3 CH2 CH2 O C CH CH3

CH3 CH3

C D O O

CH3 C O CH2 CH CH3 CH3 CH2 C O CH2 CH CH3

CH CH 3 3

28 Which reagent could be used to carry out the following reaction?

O O OH

A a solution containing acidified dichromate(VI) ions

B a solution containing dilute, acidified manganate(VII) ions

C a solution containing hot, concentrated, acidified manganate(VII) ions D concentrated sulfuric acid

29 Four reactions of propanoic acid to form salts and other products are shown.

Which reaction does not show the formulae of all the correct products?

A B – + – 2+ CH3CH2CO2 K (aq) + H2O(l) (CH3CH2CO2 )2Mg (aq) + H2(g)

KOH Mg(s)

CH3CH2CO2H(aq) propanoic Na2CO3 acid Ca(OH)2

C D CH CH CO – Na+(aq) + CO (g) (CH CH CO –) Ca2+(aq) + H O(l) 3 2 2 2 3 2 2 2 2

© UCLES 2018 9701/11/M/J/18 11

30 Compound X contains three carbon atoms. Part of a simplified infra-red spectrum of compound X is shown.

100

transmittance 50

0 4000 3000 2000 1500 1000 500

–1 wavenumber / cm

Which compound could be X?

A CH3CH2CHO

B CH3CH2CO2H

C CH3CH2CH2OH

D CH3CO2CH3

© UCLES 2018 9701/11/M/J/18 [Turn over 12

Section B

For each of the questions in this section, one or more of the three numbered statements 1 to 3 may be correct.

Decide whether each of the statements is or is not correct (you may find it helpful to put a tick against the statements that you consider to be correct).

The responses A to D should be selected on the basis of

A B C D 1, 2 and 3 1 and 2 2 and 3 1 only are only are only are is correct correct correct correct

No other combination of statements is used as a correct response.

Use of the Data Booklet may be appropriate for some questions.

31 One mole of sulfuric acid is used to make an aqueous solution. The solution contains H2SO4 + 2– – molecules, H ions, SO4 ions and HSO4 ions.

Which statements are correct?

1 The solution contains 6.02 × 1023 sulfur atoms.

+ – 2 The solution contains an exactly equal number of H ions and HSO4 ions.

2– 3 One mole of SO4 ions contains two moles of electrons.

32 Which statements are correct?

1 The hydrogen bonds in ice are more regularly arranged than in water. 2 The solidification of water to form ice is exothermic. 3 Pure water is less dense than ice.

33 Calcium reacts with water to form calcium hydroxide and hydrogen.

Ca(s) + 2H2O(l) → Ca(OH)2(s) + H2(g)

–1 The standard enthalpy change for this reaction is – 414 kJ mol .

What further information is needed in order to calculate the standard enthalpy change of formation of calcium hydroxide, Ca(OH)2(s)?

1 for H2O(l)

2 for H2(g) 3 first and second ionisation energies of Ca

© UCLES 2018 9701/11/M/J/18 13

34 Which statements are correct when a reversible reaction is at equilibrium?

1 All species are at equal concentration. 2 The concentrations of all species remain constant. 3 The rate of the forward reaction equals the rate of the reverse reaction.

35 Each of the three mixtures shown can result in a chemical reaction.

Which mixtures result in a redox reaction?

1 bromine + hydrogen 2 sodium chloride + concentrated sulfuric acid 3 potassium iodide + silver nitrate

36 Which statements correctly describe an oxide of nitrogen acting as an atmospheric pollutant?

1 Nitrogen monoxide, NO, reacts with oxygen to form nitrogen dioxide which contributes to acid rain. 2 Nitrogen dioxide reacts with sulfur dioxide to form sulfur trioxide which reacts with water to form sulfuric acid. 3 Nitrogen oxides react with unburnt hydrocarbons in sunlight to form other pollutants.

37 Polymer Z contains the length of polymer chain shown below.

This short length of chain is found many times within the chains of polymer Z, although it is not the repeat unit.

– CH2 – CHCl – CH2 –

What could be the name of polymer Z?

1 poly(2-chloropropene) 2 poly(chloroethene) 3 PVC

© UCLES 2018 9701/11/M/J/18 [Turn over 14

The responses A to D should be selected on the basis of

A B C D 1, 2 and 3 1 and 2 2 and 3 1 only are only are only are is correct correct correct correct

No other combination of statements is used as a correct response.

38 Limonene is found in lemons.

limonene

Limonene is heated with concentrated acidified potassium manganate(VII).

Which compounds are produced?

1 CH3COCH2CH2CH(CH2CO2H)2

2 CO2

3 CH3COCH2CH2CH(COCH3)CH2CO2H

39 Chlorofluoroalkanes have been used as the refrigerant in refrigerators but care has to be taken in disposing of old refrigerators.

Which statements about chlorofluoroalkanes are correct?

1 C–Cl bonds more readily undergo homolytic fission than C–F bonds. 2 Care is taken in the disposal of old refrigerators because of possible ozone depletion.

3 C2H4Cl F is more volatile than C2H6.

40 The Mr of compound X is 72. The composition by mass of X is 66.7% carbon, 11.1% hydrogen and 22.2% oxygen. X gives an orange precipitate with 2,4-dinitrophenylhydrazine reagent. X does not react with Fehling’s reagent.

What can be deduced from this information?

1 X is a carbonyl compound. 2 X is a ketone. 3 X is butanone.

© UCLES 2018 9701/11/M/J/18 Cambridge International Examinations Cambridge International Advanced Subsidiary and Advanced Level *7987105346*

CHEMISTRY 9701/21 Paper 2 AS Level Structured Questions May/June 2018

1 hour 15 minutes Candidates answer on the Question Paper. Additional Materials: Data Booklet

READ THESE INSTRUCTIONS FIRST

Write your Centre number, candidate number and name on all the work you hand in. Write in dark blue or black pen. You may use an HB pencil for any diagrams or graphs. Do not use staples, paper clips, glue or correction fluid. DO NOT WRITE IN ANY BARCODES.

Answer all questions. Electronic calculators may be used. You may lose marks if you do not show your working or if you do not use appropriate units. A Data Booklet is provided.

At the end of the examination, fasten all your work securely together. The number of marks is given in brackets [ ] at the end of each question or part question.

This document consists of 11 printed pages and 1 blank page.

IB18 06_9701_21/6RP © UCLES 2018 [Turn over 2

Answer all the questions in the spaces provided.

1 Sulfuric acid is manufactured by the Contact process.

One stage in this process is the conversion of sulfur dioxide into sulfur trioxide in the presence of a

heterogeneous catalyst of vanadium(V) oxide, V2O5.

O = 2 O=S=O(g) + O=O(g) 2 O=S=O(g) ΔH = –196 kJ mol–1

(a) (i) State the effect of a catalyst on a reaction. Explain how a catalyst causes this effect.

......

......

......

...... [2]

(ii) State the meaning of the term heterogeneous as applied to catalysts.

......

......

...... [1]

(b) Some bond energies are given.

bond bond energy / kJ mol–1

S=O (in SO2) 534 O=O 496

Use the data, and the enthalpy change for the conversion of sulfur dioxide into sulfur trioxide,

to calculate a value for the S=O bond energy in SO3.

–1 S=O bond energy in SO3 = ...... kJ mol [2]

© UCLES 2018 9701/21/M/J/18 3

The Contact process is usually carried out at a temperature of about 400 °C and a pressure just above atmospheric pressure. Using a higher or lower temperature and pressure would affect both the rate of production of sulfur trioxide and the yield of sulfur trioxide.

(c) A reaction pathway diagram for both the catalysed and uncatalysed reactions between SO2

and O2 is shown.

A

energy C E 2SO2 + O2 D B

2SO3

progress of reaction

The letters A–E represent energy changes.

Complete the table by stating which letter, A–E, represents the energy change described.

energy change letter

the energy change for the production of SO3

the activation energy for the production of SO3 in the absence of a catalyst the activation energy for the first step in thedecomposition of

SO3 in the presence of a catalyst [3]

© UCLES 2018 9701/21/M/J/18 [Turn over 4

The equation for this stage of the Contact Process is shown.

–1 2SO2(g) + O2(g) 2SO3(g) ΔH = –196 kJ mol

(d) (i) State and explain the effect of increasing temperature on the rate of production of SO3.

......

......

......

...... [3]

(ii) State and explain the effect of increasing temperature on the yield of SO3.

......

......

......

...... [3]

(e) The SO3 produced is converted to sulfuric acid in two stages. In the first stage the SO3 is

reacted with concentrated sulfuric acid to produce oleum, H2S2O7. The oleum is then reacted with water to form sulfuric acid.

Suggest an equation for the reaction of oleum, H2S2O7, with water to form sulfuric acid.

...... [1]

© UCLES 2018 9701/21/M/J/18 5

(f) SO2 reacts with water to form . Sulfurous acid is a weak Brønsted‑Lowry acid, while sulfuric acid is a strong Brønsted‑Lowry acid.

(i) Complete the ‘dot-and-cross’ diagram to show the bonding in a molecule of SO2. Show outer electrons only.

[1]

(ii) State the meaning of the term strong Brønsted‑Lowry acid.

......

......

...... [2]

(iii) Write an equation to show the acid-base behaviour of sulfuric acid with water. Include state symbols.

...... [2]

[Total: 20]

© UCLES 2018 9701/21/M/J/18 [Turn over 6

2 Crude oil is a complex mixture of hydrocarbon molecules.

The hydrocarbon molecules in crude oil are separated by fractional distillation. Fractional distillation is used because the different hydrocarbon molecules in crude oil have different boiling points.

(a) Explain why the hydrocarbon molecules in crude oil have different boiling points.

......

......

......

......

......

......

...... [2]

(b) Some of the hydrocarbon molecules obtained from crude oil are processed further by cracking.

Suggest why some hydrocarbon molecules are processed further by cracking.

......

......

...... [1]

(c) Cracking one mole of dodecane, C12H26, produces two moles of ethene and one mole of another hydrocarbon molecule.

(i) Write the equation for this cracking reaction.

...... [1]

The ethene can be used in the production of poly(ethene).

(ii) Give the full name of the process used to produce poly(ethene) from ethene.

...... [1]

© UCLES 2018 9701/21/M/J/18 7

(iii) Give two reasons why poly(ethene) should be reused or recycled rather than just thrown away.

......

......

......

...... [2]

(iv) Part of a polymer chain, produced by the same type of process as poly(ethene), is shown.

COOCH3 COOCH3 COOCH3

CH2 C CH2 C CH2 C

CH3 CH3 CH3

Give the displayed formula of the monomer used to produce this polymer.

[2]

[Total: 9]

© UCLES 2018 9701/21/M/J/18 [Turn over 8

3 The elements in the third period exhibit periodicity in both their chemical and physical properties.

(a) A graph of the atomic and ionic radii across the third period is shown.

= atomic radius / nm = ionic radius / nm 0.25

0.20

atomic or 0.15 ionic radius / nm 0.10

0.05

0.00 Na Na+ Mg Mg2+ Al Al 3+ Si Si4+ P P3– S S2– Cl Cl – atoms and ions

(i) Explain the decrease in atomic radius across the third period.

......

......

......

...... [2]

(ii) Explain why, for sodium to silicon, the ionic radii are less than the atomic radii.

......

...... [1]

(iii) Explain why, for phosphorus to chlorine, the ionic radii are greater than the atomic radii.

......

......

...... [2]

(b) The first ionisation energies of the elements across the third period show a general increase.

Aluminium and sulfur do not follow this general trend.

(i) Explain why aluminium has a lower first ionisation energy than magnesium.

......

......

...... [2]

© UCLES 2018 9701/21/M/J/18 9

(ii) Explain why sulfur has a lower first ionisation energy than phosphorus.

......

......

...... [2]

(c) The elements in the third period, from sodium to silicon, can react with chlorine to form chlorides.

(i) State and explain the pattern of change of oxidation number which occurs to both chlorine and the different Period 3 elements when they react together.

......

......

......

...... [3]

(ii) Give the equations to show the reactions of sodium chloride and silicon(IV) chloride when separately added to water.

sodium chloride ......

silicon(IV) chloride ...... [2]

(iii) Complete the table to describe the structure and bonding in sodium chloride and silicon(IV) chloride.

structure bonding

sodium chloride

silicon(IV) chloride

[2]

[Total: 16]

© UCLES 2018 9701/21/M/J/18 [Turn over 10

4 X is CH3CH(OH)CH2CH3.

(a) The reaction between X and alkaline aqueous iodine produces a yellow precipitate.

(i) Give the name of the compound formed as a yellow precipitate in this reaction.

...... [1]

(ii) Give the name of X.

...... [1]

(b) There are three structural isomers of X that are alcohols.

Draw the structures of these three isomers of X.

[2]

(c) Two reactions of X are shown.

reaction 1 CH3CH(OH)CH2CH3 CH3COCH2CH3 X

reaction 2

C4H8

(i) Identify the type of reaction involved in reaction 1.

...... [1]

(ii) Identify the reagents for reaction 1.

...... [1]

© UCLES 2018 9701/21/M/J/18 11

(iii) Reaction 2 can be carried out by passing the vapour of X over hot aluminium oxide.

The product of reaction 2, C4H8, is actually a mixture of three isomers.

Give the full names of the three isomers formed by reaction 2.

1 ......

2 ......

3 ...... [3]

(d) The reaction of methylpropene, (CH3)2CCH2, with hydrogen bromide, HBr, produces a mixture of two halogenoalkanes.

One of the halogenoalkanes, 2‑bromo‑2‑methylpropane, is formed as the major product while 1‑bromo‑2‑methylpropane is formed in small quantities.

(i) Complete the mechanism to show the reaction of methylpropene with HBr to form the major product.

Include the structure of the intermediate and all necessary charges, dipoles, lone pairs and curly arrows. The structure of 2‑bromo‑2‑methylpropane is not required.

CH3 H

H3C C C H 2-bromo-2-methylpropane

H

Br [4]

(ii) Explain why 2-bromo-2-methylpropane is the major product of this reaction.

......

......

......

...... [2]

[Total: 15]

© UCLES 2018 9701/21/M/J/18 [Turn over

UNIVERSITY OF CAMBRIDGE INTERNATIONAL EXAMINATIONS General Certificate of Education Advanced Subsidiary Level and Advanced Level

CHEMISTRY 9701/11

Paper 1 Multiple Choice October/November 2013

1 hour

Additional Materials: Multiple Choice Answer Sheet *9509356916* Soft clean eraser Soft pencil (type B or HB is recommended) Data Booklet

READ THESE INSTRUCTIONS FIRST

Write in soft pencil. Do not use staples, paper clips, highlighters, glue or correction fluid. Write your name, Centre number and candidate number on the Answer Sheet in the spaces provided unless this has been done for you. DO NOT WRITE IN ANY BARCODES.

There are forty questions on this paper. Answer all questions. For each question there are four possible answers A, B, C and D. Choose the one you consider correct and record your choice in soft pencil on the separate Answer Sheet.

Read the instructions on the Answer Sheet very carefully.

Each correct answer will score one mark. A mark will not be deducted for a wrong answer. Any rough working should be done in this booklet. Electronic calculators may be used.

This document consists of 15 printed pages and 1 blank page.

IB13 11_9701_11/5RP © UCLES 2013 [Turn over

2

Section A

For each question there are four possible answers, A, B, C, and D. Choose the one you consider to be correct.

1 At the age of 17, in a woodshed in Ohio, Charles Martin Hall discovered the commercial process for the production of aluminium metal by the electrolysis of a mixture of bauxite, Al 2O3, and cryolite, Na3Al F6.

What is the main purpose of the cryolite?

3– 3+ A Al 2O3 is covalent, and Al F6 ions interact with it to produce Al ions which can be discharged at the cathode.

B Cryolite is a base, forming NaAl O2 with bauxite, enabling aluminium to be discharged at the anode. C Cryolite minimises the release of O2– ions at the graphite anodes, which are otherwise burnt away to CO. D Cryolite reduces the melting point of the bauxite.

2 In which reaction does a single nitrogen atom have the greatest change in oxidation number?

A 4NH3 + 5O2 → 4NO + 6H2O

B 3NO2 + H2O → 2HNO3 + NO

C 2NO + O2 → 2NO2

D 4NH3 + 6NO → 5N2 + 6H2O

3 The following half reactions occur when potassium iodate(V), KIO3, in hydrochloric acid solution – oxidises iodine to ICl 2 .

– – + – – IO3 + 2Cl + 6H + 4e → ICl 2 + 3H2O

– – – I2 + 4Cl → 2ICl 2 + 2e

– What is the ratio of IO3 to I2 in the balanced chemical equation for the overall reaction?

A 1 : 1 B 1 : 2 C 1 : 4 D 2 : 1

4 Use of the Data Booklet is relevant to this question.

In which set do all species contain the same number of electrons?

A Co2+, Co3+, Co4+

– – – B F, Br , Cl

+ 2+ 3+ C Na , Mg , Al

D K2SO4, K2SeO4, K2TeO4

© UCLES 2013 9701/11/O/N/13 3

5 An autocatalytic reaction is a reaction in which one of the products catalyses the reaction.

Which curve was obtained if the rate of reaction was plotted against time for an autocatalytic reaction?

A B C D

rate rate rate rate

0 0 0 0 0 time 0 time 0 time 0 time

6 The diagrams below show the Boltzmann distribution for air at two temperatures.

The solid line represents the distribution at –20 °C.

The dotted line represents the distribution at –10 °C.

Which diagram is correct?

A B

number of number of molecules molecules

energy energy

C D

number of number of molecules molecules

energy energy

© UCLES 2013 9701/11/O/N/13 [Turn over 4

7 Which stage in the free radical substitution of methane by chlorine will have the lowest activation energy?

A CH3• + Cl 2 → CH3Cl + Cl •

B Cl • + Cl • → Cl 2

C Cl • + CH4 → CH3• + HCl

D Cl 2 → Cl • + Cl •

8 Use of the Data Booklet is relevant to this question.

The approximate percentage composition of the atmosphere on four different planets is given in the table below.

3 The density of a gas may be defined as the mass of 1 dm of the gas measured at s.t.p.

Which mixture of gases has the greatest density?

major gases / planet % by number of molecules

A Jupiter H2 89.8, He 10.2

B Neptune H2 80.0, He 19.0, CH4 1.0

C Saturn H2 96.3, He 3.25, CH4 0.45

D Uranus H2 82.5, He 15.2, CH4 2.3

9 Nitrogen reacts with hydrogen to produce ammonia.

N2(g) + 3H2(g) 2NH3(g)

A mixture of 2.00 mol of nitrogen, 6.00 mol of hydrogen, and 2.40 mol of ammonia is allowed to 3 reach equilibrium in a sealed vessel of volume 1 dm under certain conditions. It was found that 2.32 mol of nitrogen were present in the equilibrium mixture.

What is the value of Kc under these conditions?

(1.76)2 A (2.32)(6.96)3

(1.76)2 B (2.32)(6.32)3

(2.08)2 C (2.32)(6.32)3

(2.40)2 D (2.32)(6.00)3

© UCLES 2013 9701/11/O/N/13 5

10 Use of the Data Booklet is relevant to this question.

Which sodium compound contains 74.2 % by mass of sodium?

A sodium carbonate B sodium chloride C sodium hydroxide D sodium oxide

11 Use of the Data Booklet is relevant to this question.

A student carried out an experiment to determine the enthalpy change for the combustion of methanol.

The following results were obtained by the student.

start temperature of the water 20 °C

final temperature of the water 53 °C

mass of alcohol burner before burning 259.65 g

mass of alcohol burner after burning 259.15 g

mass of glass beaker plus water 150.00 g

mass of glass beaker 50.00 g

How much of the heat energy produced by the burning of methanol went into the water?

A 209 J B 13 794 J C 20 691 J D 22 154 J

© UCLES 2013 9701/11/O/N/13 [Turn over 6

12 Strontium metal can be obtained by the electrolysis of molten strontium bromide, SrBr2, using the apparatus shown in the diagram.

steel cathode

atmosphere of argon graphite anode molten strontium

molten strontium bromide

heat

Why is an atmosphere of argon used around the cathode?

A A thin film of a compound of strontium and argon forms on the surface protecting the freshly formed metal. B The argon keeps the strontium molten. C The argon stops the molten strontium rising too high in the tube. D Without the argon, strontium oxide would form in the air.

13 A metal, X, reacts with water to produce a colourless solution which gives a white precipitate when mixed with aqueous sulfuric acid.

What is metal X?

A barium B magnesium C potassium D sodium

14 Which property increases in value going down Group II?

A electronegativity B ionic radius C maximum oxidation number D second ionisation energy

© UCLES 2013 9701/11/O/N/13 7

15 Which row correctly identifies the uses of some of the compounds of Group II metals?

used as a used in agriculture refractory lining to increase the pH in kilns of a soil

A CaO Ca(OH)2

B CaO Mg(OH)2

C MgO Ca(OH)2

D MgO Mg(OH)2

16 Solid potassium halides react with concentrated sulfuric acid, according to the following equations.

reaction 1 2KCl + H2SO4 → K2SO4 + 2HCl

reaction 2 2KBr + 2H2SO4 → K2SO4 + SO2 + Br2 + 2H2O

reaction 3 8KI + 5H2SO4 → 4K2SO4 + H2S + 4I2 + 4H2O

What is the largest change in the oxidation number of sulfur in each of these reactions?

reaction 1 reaction 2 reaction 3

A 0 0 4 B 0 2 4 C 0 2 8 D 0 4 8

17 Which statement explains the observation that magnesium hydroxide dissolves in aqueous ammonium chloride, but not in aqueous sodium chloride?

+ 2+ + A The ionic radius of the NH4 ion is similar to that of Mg but not that of Na .

B NH4Cl dissociates less fully than NaCl. C The Na+ and Mg2+ ions have the same number of electrons.

+ D The NH4 ion can donate a proton.

© UCLES 2013 9701/11/O/N/13 [Turn over 8

18 Transition metals and their compounds are used as catalysts.

Which row is correct?

transition metal present transition metal present in the catalyst used in in the catalyst used in the Contact process the Haber process

A iron iron B iron vanadium C vanadium iron D vanadium vanadium

19 Consecutive elements X, Y and Z are in the third period of the Periodic Table. Element Y has the highest first ionisation energy and the lowest melting point of these three elements.

What could be the identities of X, Y and Z?

A sodium, magnesium, aluminium B magnesium, aluminium, silicon C aluminium, silicon, phosphorus D silicon, phosphorus, sulfur

20 A new industrial preparation of ethyl ethanoate has been developed using cheap sources of ethanol.

Cu catalyst Cu catalyst CH3CH2OHCH3CHO CH3CH(OH)OCH2CH3 –2[H] + CH3CH2OH Cu catalyst –2[H]

CH CO CH CH 3 2 2 3

Which process is involved at some stage in this reaction sequence?

A electrophilic addition B nucleophilic addition C nucleophilic substitution D reduction

© UCLES 2013 9701/11/O/N/13 9

21 Which reaction will give the best yield of 1-chloropropane?

A chlorine gas with propene gas in the dark

B propan-1-ol with dilute NaCl (aq)

C propan-1-ol with PCl 5

D propene with dilute HCl (aq)

22 The compound ‘leaf alcohol’ is partly responsible for the smell of new-mown grass.

CH3CH2CH=CHCH2CH2OH leaf alcohol

What will be formed when ‘leaf alcohol’ is oxidised using an excess of hot, acidified K2Cr2O7(aq)?

A CH3CH2CH(OH)CH(OH)CH2CO2H

B CH3CH2COCOCH2CO2H

C CH3CH2CH=CHCH2CO2H

D CH3CH2CO2H and HO2CCH2CO2H

23 Which compound exhibits stereoisomerism?

A CH3CHCl CH3

B CH3CHCl CH2Cl

C CH3CCl 2CH3

D CH2Cl CH2CH2Cl

24 A carbanion is an organic ion in which a carbon atom has a negative charge. A carbocation is an organic ion in which a carbon atom has a positive charge.

The reaction between aqueous sodium hydroxide and 1-bromobutane proceeds by an SN2 mechanism.

How should the first step in the mechanism be described?

A attack by a nucleophile on a carbon atom with a partial positive charge B heterolytic bond fission followed by an attack by an electrophile on a carbanion C heterolytic bond fission followed by an attack by a nucleophile on a carbocation D homolytic bond fission followed by an attack by a nucleophile on a carbocation

© UCLES 2013 9701/11/O/N/13 [Turn over 10

25 What are the only structures formed when butan-2-ol is heated with concentrated H2SO4?

CH3CH2 H CH3 H A CC CC

H H H CH3

CH3 CH3 CH3 H H H B CC CC CC

H H H CH3 CH3CH2 H

H H H H

C CC CC

CH3CH2 H CH3 CH3

CH3 H CH3CH2 H H H D CC CC CC

CH3 H H H CH3 CH3

26 How many moles of hydrogen, H2, are evolved when an excess of sodium metal is added to one mole of citric acid?

CO2H

HOC CH2CO2H

CH2CO2H

citric acid

A 1 B 2 C 3 D 4

© UCLES 2013 9701/11/O/N/13 11

27 Primary alcohols can be oxidised to aldehydes using either acidified potassium dichromate(VI) or acidified potassium manganate(VII). Both these oxidising agents change colour as they are reduced.

What is the colour of each oxidising agent before and after it has reacted?

acidified potassium dichromate(VI) acidified potassium manganate(VII)

before after before after

A green orange purple colourless B orange green colourless purple C orange green purple colourless D purple colourless orange green

28 In which reaction is the organic compound oxidised?

A CH3CH2OH + concentrated H3PO4

B CH3CH2CH2CHO + Tollens’ reagent

C CH3COCH3 + 2,4-dinitrophenylhydrazine reagent

D CH3CN + dilute H2SO4

29 How many of the compounds shown will react with aqueous sodium hydroxide to form the sodium salt of a carboxylic acid?

H3C O H O

C CH2 CH3 C CH2 CH3 O O

O O

H C H3C CH C

O CH2 CH2 CH3 CH O H 3

A 1 B 2 C 3 D 4

© UCLES 2013 9701/11/O/N/13 [Turn over 12

30 Which types of bond breakage and bond formation occur in the addition polymerisation of alkenes?

bond breakage bond formation

A π only σ only B π only σ and π C σ and π σ only D σ and π σ and π

© UCLES 2013 9701/11/O/N/13 13

Section B

For each of the questions in this section, one or more of the three numbered statements 1 to 3 may be correct.

Decide whether each of the statements is or is not correct (you may find it helpful to put a tick against the statements that you consider to be correct).

The responses A to D should be selected on the basis of

A B C D

1, 2 and 3 1 and 2 2 and 3 1 only are only are only are is correct correct correct correct

No other combination of statements is used as a correct response.

31 X is a particle with 18 electrons and 20 neutrons.

What could be the symbol of X?

38 1 18 Ar

40 2+ 2 20 Ca

39 + 3 19K

32 Use of the Data Booklet is relevant to this question.

Carbon and nitrogen are adjacent in the Periodic Table.

Which properties do they both have?

1 There is an empty 2p orbital in one atom of the element. 2 The principal quantum number of the highest occupied orbital is 2. 3 They form compounds in which their atoms form bonds with four other atoms.

33 What are necessary properties of a dynamic equilibrium?

1 Equal amounts of reactants and products are present. 2 Concentrations of reactants and products remain constant. 3 The rate of the forward reaction is the same as the rate of the reverse reaction.

© UCLES 2013 9701/11/O/N/13 [Turn over 14

The responses A to D should be selected on the basis of

A B C D

1, 2 and 3 1 and 2 2 and 3 1 only are only are only are is correct correct correct correct

No other combination of statements is used as a correct response.

34 If N2O4 gas is placed in a sealed vessel the following equilibrium is established.

N2O4(g) 2NO2(g)

The forward reaction is endothermic.

What happens when the temperature is increased?

1 The equilibrium constant increases.

2 The partial pressure of NO2 increases. 3 The activation energy is unchanged.

35 Which types of bonding are present in ammonium carbonate, (NH4)2CO3?

1 ionic 2 covalent 3 co-ordinate (dative covalent)

36 Sulfur dioxide and sulfites are used in food preservation.

Why are they used for this purpose?

1 They are reducing agents which slow down the oxidation of food. 2 They inhibit the growth of aerobic bacteria.

3 They react with NO2(g) converting it to NO(g).

37 The organic compound X gives a precipitate when warmed with aqueous silver nitrate. This precipitate dissolves when concentrated aqueous ammonia is added.

What is a possible identity for X?

1 1-bromopropane 2 2-chlorobutane 3 2-iodo,2-methylpropane

© UCLES 2013 9701/11/O/N/13 15

38 An organic compound Y, molecular formula C6H14O, may be oxidised to compound Z, molecular formula C6H12O2.

What could be the structural formula of Y?

1 CH3CH2CH(CH2OH)CH2CH3

2 (CH3)3CCH2CH2OH

3 CH3CH2CH(CH3)CH2CH2OH

39 Which reactions can be used to make an alcohol in the laboratory?

1 hydrolysis of a bromoalkane with NaOH(aq)

2 reduction of a ketone with NaBH4

3 reduction of an aldehyde with NaBH4

40 The compounds below are treated with hydrogen cyanide.

Which compounds react and produce a molecule containing a chiral centre?

1 butanal 2 pentan-3-one 3 2-chlorobutane

© UCLES 2013 9701/11/O/N/13 UNIVERSITY OF CAMBRIDGE INTERNATIONAL EXAMINATIONS General Certifi cate of Education Advanced Subsidiary Level and Advanced Level *1650953752*

CHEMISTRY 9701/21

Paper 2 Structured Questions AS Core October/November 2013

1 hour 15 minutes Candidates answer on the Question Paper. Additional Materials: Data Booklet

READ THESE INSTRUCTIONS FIRST

Write your Centre number, candidate number and name on all the work you hand in. Write in dark blue or black pen. You may use a soft pencil for any diagrams, graphs or rough working. Do not use staples, paper clips, highlighters, glue or correction fl uid. DO NOT WRITE IN ANY BARCODES.

Answer all questions. Electronic calculators may be used. You may lose marks if you do not show your working or if you do not use appropriate units. A Data Booklet is provided.

At the end of the examination, fasten all your work securely together. The number of marks is given in brackets [ ] at the end of each question or part question.

For Examiner’s Use

1

2

3

4

5

Total

This document consists of 9 printed pages and 3 blank pages.

IB13 11_9701_21/2RP © UCLES 2013 [Turn over 2

For Answer all the questions in the spaces provided. Examiner’s Use

1 Valence Shell Electron Pair Repulsion theory (VSEPR) is a model of electron-pair repulsion (including lone pairs) that can be used to deduce the shapes of, and bond angles in, simple molecules.

(a) Complete the table below by using simple hydrogen-containing compounds. One example has been included.

formula of number of number of shape of a molecule bond pairs lone pairs molecule with this shape

3 0 trigonal planar BH3

40

31

22

[3]

(b) Tellurium, Te, proton number 52, is used in photovoltaic cells.

When fl uorine gas is passed over tellurium at 150 °C, the colourless gas TeF6 is formed.

(i) Draw a ‘dot-and-cross’ diagram of the TeF6 molecule, showing outer electrons only.

(ii) What will be the shape of the TeF6 molecule?

......

(iii) What is the F–Te–F bond angle in TeF6?

...... [3]

[Total: 6]

© UCLES 2013 9701/21/O/N/13 3

For 2 The molecular formula C3H6 represents the compounds propene and cyclopropane. Examiner’s Use H

H C H H C C CH3CH CH2 H H propene cyclopropane

(a) What is the H–C–H bond angle at the terminal =CH2 group in propene?

...... [1]

(b) Under suitable conditions, propene and cyclopropane each react with chlorine.

(i) With propene, 1,2-dichloropropane, CH3CHCl CH2Cl is formed.

State fully what type of reaction this is.

...... [1]

(ii) When cyclopropane reacts with chlorine, three different compounds with the

molecular formula C3H4Cl 2 can be formed.

Draw displayed structures of each of these three compounds.

[3]

[Total: 5]

© UCLES 2013 9701/21/O/N/13 [Turn over 4

For 3 Chlorine gas is manufactured by the electrolysis of brine using a diaphragm cell. Examiner’s Use (a) (i) Write half-equations, including state symbols, for the reactions occurring at each of the electrodes of a diaphragm cell.

anode ......

cathode ......

(ii) In the diaphragm cell, the anode is made of titanium and the cathode is made of steel.

Suggest why steel is never used for the anode.

......

...... [3]

(b) Chlorine is very reactive and will form compounds by direct combination with many elements.

Describe what you would see when chlorine is passed over separate heated samples of sodium and phosphorus. In each case write an equation for the reaction.

sodium

......

......

......

phosphorus

......

......

...... [4]

© UCLES 2013 9701/21/O/N/13 5

For (c) Chlorine reacts with aqueous sodium hydroxide in two different ways, depending on the Examiner’s conditions used. In each case, water, sodium chloride and one other chlorine-containing Use compound are formed.

For each condition below, give the formula of the other chlorine-containing compound and state the oxidation number of chlorine in it.

formula of other oxidation number of condition chlorine-containing compound chlorine in this compound

cold dilute NaOH(aq)

hot concentrated NaOH(aq)

[4]

(d) Magnesium chloride, MgCl 2, and silicon tetrachloride, SiCl 4, each dissolve in or react with water.

Suggest the approximate pH of the solution formed in each case.

MgCl 2 ...... SiCl 4 ......

Explain, with the aid of an equation, the difference between the two values.

......

......

......

...... [5]

[Total: 16]

© UCLES 2013 9701/21/O/N/13 [Turn over 6

For 4 Compound R is a weak diprotic (dibasic) acid which is very soluble in water. Examiner’s Use 3 (a) A solution of R was prepared which contained 1.25 g of R in 250 cm of solution. 3 –3 3 When 25.0 cm of this solution was titrated with 0.100 mol dm NaOH, 21.6 cm of the alkali were needed for complete reaction.

(i) Using the formula H2X to represent R, construct a balanced equation for the reaction

between H2X and NaOH.

......

(ii) Use the data above to calculate the amount, in moles, of OH– ions used in the titration.

(iii) Use your answers to (i) and (ii) to calculate the amount, in moles, of R present in 3 25.0 cm of solution.

3 (iv) Calculate the amount, in moles, of R present in 250 cm of solution.

(v) Calculate Mr of R.

[5]

(b) Three possible structures for R are shown below.

ST U

HO2CCH=CHCO2HHO2CCH(OH)CH2CO2HHO2CCH(OH)CH(OH)CO2H

(i) Calculate the Mr of each of these acids.

Mr of S = ...... Mr of T = ...... Mr of U = ......

(ii) Deduce which of the structures, S, T or U, correctly represents the structure of the acid, R.

R is represented by ...... [2]

© UCLES 2013 9701/21/O/N/13 7

For It is possible to convert S, T, or U into one another. Examiner’s Use (c) State the reagent(s) and essential conditions that would be used for the following conversions.

S into T

......

S into U

......

T into S

...... [5]

(d) Give the structural formula of the organic product formed in each of the following reactions.

T reacting with an excess of Na

U reacting with an excess of Na2CO3

[2]

(e) The acid S shows stereoisomerism. Draw structures to show this isomerism. Label each isomer.

[2]

(f) When one of the isomers of S is heated at 110 °C in the absence of air, a cyclic compound

V, with molecular formula C4H2O3, is formed. The other isomer of S does not react at this temperature.

Suggest the displayed formula of V.

[2]

[Total: 18]

© UCLES 2013 9701/21/O/N/13 [Turn over 8

For 5 Propane, C3H8, and butane, C4H10, are components of Liquefi ed Petroleum Gas (LPG) which Examiner’s is widely used as a fuel for domestic cooking and heating. Use

(a) (i) To which class of compounds do these two hydrocarbons belong?

......

(ii) Write a balanced equation for the complete combustion of butane.

...... [2]

(b) When propane or butane is used in cooking, the saucepan may become covered by a solid black deposit.

(i) What is the chemical name for this black solid?

......

(ii) Write a balanced equation for its formation from butane.

...... [2]

(c) Propane and butane have different values of standard enthalpy change of combustion.

De fi ne the term standard enthalpy change of combustion.

......

......

...... [2]

3 (d) A 125 cm sample of propane gas, measured at 20 °C and 101 kPa, was completely burnt in air. The heat produced raised the temperature of 200 g of water by 13.8 °C. Assume no heat losses occurred during this experiment.

(i) Use the equation pV = nRT to calculate the mass of propane used.

© UCLES 2013 9701/21/O/N/13 9

For (ii) Use relevant data from the Data Booklet to calculate the amount of heat released in Examiner’s this experiment. Use

(iii) Use the data above and your answers to (i) and (ii) to calculate the energy produced by the burning of 1 mol of propane.

[5]

(e) The boiling points of methane, ethane, propane, and butane are given below.

compound CH4 CH3CH3 CH3CH2CH3 CH3(CH2)2CH3

boiling point / K 112 185 231 273

(i) Suggest an explanation for the increase in boiling points from methane to butane.

......

......

......

(ii) The isomer of butane, 2-methylpropane, (CH3)3CH, has a boiling point of 261 K. Suggest an explanation for the difference between this value and that for butane in the table above.

......

......

...... [4]

[Total: 15]

© UCLES 2013 9701/21/O/N/13 [Turn over

Cambridge International Examinations Cambridge International Advanced Subsidiary and Advanced Level

CHEMISTRY 9701/11

Paper 1 Multiple Choice October/November 2014

1 hour

Additional Materials: Multiple Choice Answer Sheet *6140222537* Soft clean eraser Soft pencil (type B or HB is recommended) Data Booklet

READ THESE INSTRUCTIONS FIRST

Write in soft pencil. Do not use staples, paper clips, glue or correction fluid. Write your name, Centre number and candidate number on the Answer Sheet in the spaces provided unless this has been done for you. DO NOT WRITE IN ANY BARCODES.

There are forty questions on this paper. Answer all questions. For each question there are four possible answers A, B, C and D. Choose the one you consider correct and record your choice in soft pencil on the separate Answer Sheet.

Read the instructions on the Answer Sheet very carefully.

Each correct answer will score one mark. A mark will not be deducted for a wrong answer. Any rough working should be done in this booklet. Electronic calculators may be used.

This document consists of 13 printed pages and 3 blank pages.

IB14 11_9701_11/3RP © UCLES 2014 [Turn over

2

Section A

For each question there are four possible answers, A, B, C, and D. Choose the one you consider to be correct.

1 Which solid-line curve most accurately represents the distribution of molecular energies in a gas at 500 K if the dotted-line curve represents the corresponding distribution for the same gas at 300 K?

A B

number of number of molecules molecules

molecular energy molecular energy

C D

number of number of molecules molecules

molecular energy molecular energy

2 In which reaction does hydrogen behave as an oxidising agent?

A H2 + Cl 2 → 2HCl

B C2H4 + H2 → C2H6

C N2 + 3H2 → 2NH3

D 2Na + H2 → 2NaH

3 Ethanol is increasingly being used as a fuel for cars.

–1 The standard enthalpy change of formation of carbon dioxide is –393 kJ mol . –1 The standard enthalpy change of formation of water is –286 kJ mol . –1 The standard enthalpy change of formation of ethanol is –277 kJ mol .

What is the standard enthalpy change of combustion of ethanol?

–1 A –1921 kJ mol

–1 B –1367 kJ mol

–1 C –956 kJ mol

–1 D – 402 kJ mol

© UCLES 2014 9701/11/O/N/14 3

4 Two glass vessels M and N are connected by a closed valve.

MN

5 M contains helium at 20 °C at a pressure of 1 × 10 Pa. N has been evacuated, and has three times the volume of M. In an experiment, the valve is opened and the temperature of the whole apparatus is raised to 100 °C.

What is the final pressure in the system?

4 A 3.18 × 10 Pa

4 B 4.24 × 10 Pa

5 C 1.25 × 10 Pa

5 D 5.09 × 10 Pa

5 The table shows the physical properties of four substances.

Which substance could be hydrogen chloride?

electrical electrical electrical melting point conductivity conductivity conductivity / °C of aqueous of solid of liquid solution

A –119 poor poor insoluble B –115 poor poor good C –50 poor poor poor D 993 poor good good

6 Aluminium carbide, Al 4C3, reacts readily with aqueous sodium hydroxide. The two products of the reaction are NaAl O2 and a hydrocarbon. Water molecules are also involved as reactants.

What is the formula of the hydrocarbon?

A CH4 B C2H6 C C3H8 D C6H12

© UCLES 2014 9701/11/O/N/14 [Turn over 4

7 In an experiment to calculate the enthalpy change of combustion of a fuel, 1.5 g (0.0326 mol) of the fuel was used to heat 200 g of water. The temperature of the water rose from 25 °C to 55 °C. –1 –1 The specific heat capacity of water is 4.18 J g K .

There is significant heat loss in this experiment. Therefore, the experimental value for the enthalpy change of combustion, ∆Hc, of the fuel will be different from the theoretical value.

Using the information above, what is the experimental value for the enthalpy change of combustion, ∆Hc, of the fuel?

–1 A –1410 kJ mol

–1 B –769 kJ mol

–1 C –30.7 kJ mol

–1 D –16.7 kJ mol

8 The reaction pathway diagram below illustrates the energies of the reactants, the products and the transition state of a reaction.

transition state E1

E2 energy reactants

E 3 products

extent of reaction

Which expression represents the activation energy of the forward reaction?

A E1 – E2 B E2 – E1 C E2 – E3 D E3 – E2

9 Methylpropan-1-ol and butan-1-ol are structural isomers. Methylpropan-1-ol has a lower boiling point.

Which statement explains why the boiling point of methylpropan-1-ol is lower than that of butan-1-ol?

A Methylpropan-1-ol cannot form hydrogen bonds. B Methylpropan-1-ol has weaker covalent bonds than butan-1-ol. C Methylpropan-1-ol has weaker van der Waals’ forces than butan-1-ol. D Methylpropan-1-ol molecules have more surface area than butan-1-ol molecules.

© UCLES 2014 9701/11/O/N/14 5

10 Which row correctly describes the electrodes used in the electrolysis cell for the production of aluminium?

anode cathode

A carbon carbon B carbon steel C steel carbon D steel steel

11 For which equation is the enthalpy change correctly described as an enthalpy change of formation?

A C(g) + O2(g) → CO2(g) B 1 → C(s) + 2 O2(g) CO(g)

C 2N(g) + 4O(g) → N2O4(g)

D 2NO(g) + O2(g) → 2NO2(g)

12 Element X, in Period 3, has the following properties.

● Its oxide has a giant structure. ● It forms covalent bonds with chlorine.

● Its oxide will neutralise HCl (aq).

What is element X?

A Mg B Al C Si D P

13 Which property is not associated with the element sodium?

A It can react with cold water to form hydrogen. B It forms a basic oxide. C It forms a neutral chloride. D It is an oxidising agent.

© UCLES 2014 9701/11/O/N/14 [Turn over 6

14 Use of the Data Booklet is relevant to this question.

Sir Humphrey Davy discovered boron, calcium, magnesium and sodium.

Which of these elements has the second smallest atomic radius in its group and the third lowest first ionisation energy in its period?

A boron B calcium C magnesium D sodium

15 Use of the Data Booklet is relevant to this question.

3 3 A sample of potassium oxide, K2O, is dissolved in 250 cm of distilled water. 25.0 cm of this –3 3 solution is titrated against sulfuric acid of concentration 2.00 mol dm . 15.0 cm of this sulfuric acid is needed for complete neutralisation.

3 Which mass of potassium oxide was originally dissolved in 250 cm of distilled water?

A 2.83 g B 28.3 g C 47.1 g D 56.6 g

16 Chlorine gas reacts with cold aqueous sodium hydroxide. It can also react with hot aqueous sodium hydroxide.

What are the oxidation numbers of chlorine in the products of these reactions?

cold aqueous hot aqueous

sodium hydroxide sodium hydroxide

A –1, +1 –1, +5 B –1, +1 +1, +6 C –1, +2 –1, +5 D –1, +2 +1, +6

17 Under standard conditions, which statement is correct?

– A Cl (aq) can oxidise Br2(aq).

– B Cl (aq) can reduce Br2(aq).

– C Cl 2(aq) can oxidise Br (aq).

– D Cl 2(aq) can reduce Br (aq).

© UCLES 2014 9701/11/O/N/14 7

18 Total removal of the pollutant sulfur dioxide, SO2, is difficult. The quantities emitted from furnace chimneys can be lowered by using desulfurisation plants. The gases are reacted with calcium hydroxide to remove the SO2.

What is the main product formed initially?

A Ca(HSO4)2 B CaS C CaSO3 D CaSO4

19 Which oxide does not react with cold dilute sodium hydroxide to produce a salt?

A Al 2O3 B P4O10 C SO2 D SiO2

20 Which row correctly describes the reaction between propene and bromine, Br2(l)?

reaction mechanism organic product

A electrophilic addition CH3CHBrCH2Br

B electrophilic addition CH3CH2CH2Br

C nucleophilic substitution CH3CH2CH2Br

D nucleophilic substitution CH3CHBrCH2Br

21 Use of the Data Booklet is relevant to this question.

Which compound has an Mr of 84 and will react with HBr to give a product with an Mr of 164.9?

ABCD

O

22 1,1-dichloropropane reacts with aqueous sodium hydroxide in a series of steps to give propanal.

NaOH(aq) CH CH CHCl CH CH CHO 3 2 2 3 2

Which term describes the first step of this reaction?

A addition B elimination C oxidation D substitution

© UCLES 2014 9701/11/O/N/14 [Turn over 8

23 Considering only structural isomers, what is the number of alcohols of each type with the formula C5H12O?

primary secondary tertiary

A 3 3 2 B 4 2 2 C 4 3 1 D 5 2 1

24 Lactic acid, CH3CH(OH)CO2H, causes pain when it builds up in muscles.

Which reagent reacts with both of the –OH groups in lactic acid?

A acidified potassium dichromate(VI) B ethanol C sodium D sodium hydroxide

25 In the hydrolysis of bromoethane by aqueous sodium hydroxide, what is the nature of the attacking group and of the leaving group?

attacking group leaving group

A electrophile electrophile B electrophile nucleophile C nucleophile electrophile D nucleophile nucleophile

26 Corticosterone is a hormone involved in the metabolism of carbohydrates and proteins.

O OH

CH3 HO

CH3

O

corticosterone

How many chiral centres are there in one molecule of corticosterone?

A 5 B 6 C 7 D 8

© UCLES 2014 9701/11/O/N/14 9

27 Methyl methylpropenoate is the monomer used to make Perspex.

Which diagram correctly shows methyl methylpropenoate?

AB

CH3

CH2 C CH3 CH CH CO2CH3

CO2CH3

CD

CH3

CH2 CH CO2CH2CH3 C CH CO2CH3 CH 3

28 The ester CH3CH2CH2CO2CH3 is responsible for the aroma of apples.

When this ester is hydrolysed by acid in the stomach, what is the empirical formula of the organic acid produced?

A CH2O B CH4O C C2H4O D C3H6O2

29 Which equation correctly represents the balanced equation for the complete combustion of a hydrocarbon with the formula CxHy?

y y A CxHy + (x + )O2 → xCO2 + H2O 2 2

y B CxHy + (x + )O2 → xCO2 + yH2O 4

y y C CxHy + (x + )O2 → xCO2 + H2O 4 4

y y D CxHy + (x + )O2 → xCO2 + H2O 4 2

© UCLES 2014 9701/11/O/N/14 [Turn over 10

30 β-carotene is responsible for the orange colour of carrots.

CH3 CH CH CH 3 3 3 CH3

CH3 CH3 CH3 CH3 CH3 β-carotene

β-carotene is oxidised by hot, concentrated, acidified KMnO4.

When an individual molecule of β-carotene is oxidised in this way, many product molecules are formed.

How many of these product molecules contain a ketone functional group?

A 4 B 6 C 9 D 11

© UCLES 2014 9701/11/O/N/14 11

Section B

For each of the questions in this section, one or more of the three numbered statements 1 to 3 may be correct.

Decide whether each of the statements is or is not correct (you may find it helpful to put a tick against the statements that you consider to be correct).

The responses A to D should be selected on the basis of

A B C D

1, 2 and 3 1 and 2 2 and 3 1 only are only are only are is correct correct correct correct

No other combination of statements is used as a correct response.

31 Use of the Data Booklet is relevant to this question.

Which ions contain one or more unpaired electrons?

1 Cu2+ 2 Mn3+ 3 V3+

32 Use of the Data Booklet is relevant to this question.

–1 The bond energy of the Br – O bond is 235 kJ mol .

Which reactions are exothermic?

1 OH• + HBr → H2 + BrO•

2 OH• + HBr → H2O + Br•

3 H• + HBr → H2 + Br•

33 A reversible reaction is catalysed.

Which statements about the effects of the catalyst on this system are correct?

1 The catalyst alters the mechanism of the reaction. 2 The catalyst reduces the activation energy for both the forward and the backward reaction. 3 The catalyst alters the composition of the equilibrium mixture.

© UCLES 2014 9701/11/O/N/14 [Turn over 12

The responses A to D should be selected on the basis of

A B C D

1, 2 and 3 1 and 2 2 and 3 1 only are only are only are is correct correct correct correct

No other combination of statements is used as a correct response.

34 A student borrowed a friend’s chemistry notes and copied out the notes in the box below.

Which statements are correct?

A gas behaves less like an ideal gas when the gas

1 is at low pressure. 2 is at low temperature. 3 can be easily liquefied.

35 On being heated, hydrogen iodide breaks down more quickly than hydrogen chloride.

Which statements explain this faster rate?

1 The HI bond is weaker than the HCl bond.

2 The reaction of the breakdown of HI has a smaller activation energy than that of HCl.

3 The breakdown of HI is more exothermic than that of HCl.

36 Which statements about calcium oxide are correct?

1 It reacts with cold water. 2 It is produced when calcium nitrate is heated. 3 It can be reduced by heating with magnesium.

37 Propanal will react with hydrogen cyanide to form 2-hydroxybutanenitrile. A suitable catalyst for this reaction is sodium cyanide.

NaCN

CH3CH2CHO + HCN CH3CH2CH(OH)CN

Which statements about the reaction of propanal with hydrogen cyanide are correct?

1 The CN– ion attacks the propanal molecule to form an intermediate ion. 2 The product of the reaction has a chiral carbon atom. 3 The CN– ion is a stronger electrophile than the HCN molecule.

© UCLES 2014 9701/11/O/N/14 13

38 X is an organic compound that gives a precipitate with aqueous silver nitrate. This precipitate remains undissolved when concentrated aqueous ammonia is added.

What is a possible identity for X?

1 iodomethane 2 2-bromobutane 3 2-chlorobutane

39 For which mixtures of reagents are the colour changes described correctly?

reagents colour change

1 pentanal + hot, acidified potassium dichromate(VI) orange to green 2 pentan-2-one + warm Fehling’s reagent no change 3 cyclohexane + cold, acidified potassium manganate(VII) purple to colourless

40 A reaction pathway diagram is shown.

energy

extent of reaction

Which reactions would have this profile?

1 (CH3)3CBr + NaOH → (CH3)3COH + NaBr

2 CH3CH2Br + NaOH → CH3CH2OH + NaBr

3 (CH3)3CCH2CH2Cl + 2NH3 → (CH3)3CCH2CH2NH2 + NH4Cl

© UCLES 2014 9701/11/O/N/14 Cambridge International Examinations Cambridge International Advanced Subsidiary and Advanced Level *5627253124*

CHEMISTRY 9701/21

Paper 2 Structured Questions AS Core October/November 2014

1 hour 15 minutes Candidates answer on the Question Paper. Additional Materials: Data Booklet

READ THESE INSTRUCTIONS FIRST

Write your Centre number, candidate number and name on all the work you hand in. Write in dark blue or black pen. You may use an HB pencil for any diagrams or graphs. Do not use staples, paper clips, glue or correction fl uid. DO NOT WRITE IN ANY BARCODES.

Answer all questions. Electronic calculators may be used. You may lose marks if you do not show your working or if you do not use appropriate units. A Data Booklet is provided.

At the end of the examination, fasten all your work securely together. The number of marks is given in brackets [ ] at the end of each question or part question.

This document consists of 10 printed pages and 2 blank pages.

IB14 11_9701_21/3RP © UCLES 2014 [Turn over 2

Answer all the questions in the spaces provided.

1 (a) Successive ionisation energies for the elements magnesium to barium are given in the table.

1st ionisation 2nd ionisation 3rd ionisation element –1 –1 –1 energy / kJ mol energy / kJ mol energy / kJ mol

Mg 736 1450 7740 Ca 590 1150 4940 Sr 548 1060 4120 Ba 502 966 3390

(i) Explain why the fi rst ionisation energies decrease down the group.

......

......

......

...... [3]

(ii) Explain why, for each element, there is a large increase between the 2nd and 3rd ionisation energies.

......

......

...... [2]

(b) A sample of strontium, atomic number 38, gave the mass spectrum shown. The percentage abundances are given above each peak.

100

82.58

percentage abundance

9.86 7.007.00 0.56 0 80 81 82 83 84 85 86 87 88 89 90 atomic mass units

© UCLES 2014 9701/21/O/N/14 3

(i) Complete the full electronic confi guration of strontium.

1s2 2s2 2p6 ...... [1]

(ii) Explain why there are four different peaks in the mass spectrum of strontium.

......

...... [1]

(iii) Calculate the atomic mass, Ar, of this sample of strontium. Give your answer to three signifi cant fi gures.

Ar = ...... [2]

(c) A compound of barium, A, is used in fi reworks as an oxidising agent and to produce a green colour.

(i) Explain, in terms of electron transfer, what is meant by the term oxidising agent.

......

...... [1]

(ii) A has the following percentage composition by mass: Ba, 45.1; Cl , 23.4; O, 31.5.

Calculate the empirical formula of A.

empirical formula of A ...... [3]

© UCLES 2014 9701/21/O/N/14 [Turn over 4

(d) Some reactions involving magnesium and its compounds are shown in the reaction scheme below.

water steam X(aq) Mg(s) Y(s)

reaction reaction HNO (aq) 1 3 2

Z(aq) Z(s)

(i) Give the formulae of the compounds X, Y and Z.

X ......

Y ......

Z ...... [3]

(ii) Name the reagent needed to convert Y(s) into Z(aq) in reaction 1 and write an equation for the reaction.

reagent ......

equation ...... [2]

(iii) How would you convert a sample of Z(s) into Y(s) in reaction 2?

...... [1]

(iv) Give equations for the conversions of Mg into X, and Z(s) into Y.

Mg to X ......

Z to Y ...... [2]

[Total: 21]

© UCLES 2014 9701/21/O/N/14 6

2 The Contact process for the manufacture of sulfuric acid was originally patented in the 19th century and is still in use today.

The key step in the overall process is the reversible conversion of sulfur dioxide to sulfur trioxide in the presence of a vanadium(V) oxide catalyst.

–1 2SO2(g) + O2(g) 2SO3(g) ∆H = –196 kJ mol

(a) One way in which the sulfur dioxide for this reaction is produced is by heating the sulfi de ore

iron pyrites, FeS2, in air. Iron(III) oxide is also produced. Write an equation for this reaction.

...... [2]

(b) The sulfur trioxide produced in the Contact process is reacted with 98% sulfuric acid. The resulting compound is then reacted with water to produce sulfuric acid.

(i) Explain why the sulfur trioxide is not fi rst mixed directly with water.

......

...... [1]

(ii) Write equations for the two steps involved in the conversion of sulfur trioxide into sulfuric acid.

......

...... [2]

(c) (i) Sulfur dioxide and sulfur trioxide both contain only S=O double bonds.

Draw labelled diagrams to show the shapes of these two molecules.

SO2 SO3

[2]

(ii) For your diagrams in (i), name the shapes and suggest the bond angles.

SO2 shape ...... SO3 shape ......

SO2 bond angle ...... SO3 bond angle ...... [2]

© UCLES 2014 9701/21/O/N/14 7

(d) The conversion of sulfur dioxide into sulfur trioxide is carried out at a temperature of 400 °C.

(i) With reference to Le Chatelier’s Principle and reaction kinetics, state and explain one advantage and one disadvantage of using a higher temperature.

......

......

......

......

......

...... [4]

(ii) State the expression for the equilibrium constant, Kp, for the formation of sulfur trioxide from sulfur dioxide.

Kp =

[1]

(iii) 2.00 moles of sulfur dioxide and 2.00 moles of oxygen were put in a fl ask and left to reach equilibrium. 5 At equilibrium, the pressure in the fl ask was 2.00 × 10 Pa and the mixture contained 1.80 moles of sulfur trioxide.

Calculate Kp. Include the units.

Kp = ......

units = ...... [5]

[Total: 19]

© UCLES 2014 9701/21/O/N/14 [Turn over 8

3 P , Q and R are structural isomers with the molecular formula C4H8.

All three compounds readily decolourise bromine in the dark.

P and Q do not exhibit stereoisomerism but R exists as a pair of geometrical (cis-trans) isomers.

All three compounds react with hot concentrated, acidifi ed potassium manganate(VII) to produce a variety of products as shown in the table.

compound products

P CO2 and S (C3H6O)

Q CO2 and CH3CH2CO2H

R CH3CO2H only

S reacts with 2,4-dinitrophenylhydrazine reagent, 2,4-DNPH, to form an orange crystalline product but does not react with Fehling’s reagent.

(a) Give the structural formulae of P, Q, R and S.

P ...... Q ......

R ...... S ...... [4]

(b) (i) Explain what is meant by the term stereoisomerism.

......

......

...... [2]

© UCLES 2014 9701/21/O/N/14 9

(ii) Draw the displayed formulae of the geometrical isomers of R and name them both.

name ...... name ...... [2]

(c) State a reagent that could be used for the reduction of S and name the organic product of this reduction.

reagent ...... product ...... [2]

[Total: 10]

© UCLES 2014 9701/21/O/N/14 [Turn over 10

4 A series of reactions based on propanoic acid is shown.

CaCO3 CH3CH2CO2H three products

reaction 1 reaction 2

reaction 3 CH3CH2CH2OH CH3CO2CH2CH2CH3

(a) Write an equation for reaction 1, using [H] to represent the reducing agent.

...... [2]

(b) (i) What type of reaction is reaction 2?

...... [1]

(ii) Suggest a suitable reagent and conditions for reaction 2.

...... [2]

(c) Write an equation for the reaction of propanoic acid with calcium carbonate, CaCO3.

...... [2]

(d) (i) Suggest a suitable reagent and conditions for reaction 3.

......

...... [2]

(ii) Identify the other product of reaction 3.

...... [1]

[Total: 10]

© UCLES 2014 9701/21/O/N/14

Cambridge International Examinations Cambridge International Advanced Subsidiary and Advanced Level

CHEMISTRY 9701/11

Paper 1 Multiple Choice October/November 2015

1 hour

Additional Materials: Multiple Choice Answer Sheet *2857618227* Soft clean eraser Soft pencil (type B or HB is recommended) Data Booklet

READ THESE INSTRUCTIONS FIRST

Write in soft pencil. Do not use staples, paper clips, glue or correction fluid. Write your name, Centre number and candidate number on the Answer Sheet in the spaces provided unless this has been done for you. DO NOT WRITE IN ANY BARCODES.

There are forty questions on this paper. Answer all questions. For each question there are four possible answers A, B, C and D. Choose the one you consider correct and record your choice in soft pencil on the separate Answer Sheet.

Read the instructions on the Answer Sheet very carefully.

Each correct answer will score one mark. A mark will not be deducted for a wrong answer. Any rough working should be done in this booklet. Electronic calculators may be used.

This document consists of 13 printed pages and 3 blank pages.

IB15 11_9701_11/FP © UCLES 2015 [Turn over

2

Section A

For each question there are four possible answers, A, B, C, and D. Choose the one you consider to be correct.

1 The table gives the successive ionisation energies for an element X.

1st 2nd 3rd 4th 5th 6th

–1 ionisation energy / kJ mol 950 1800 2700 4800 6000 12 300

What could be the formula of a chloride of X?

A XCl B XCl 2 C XCl 3 D XCl 4

2 Which set of conditions gives the highest yield of ammonia at equilibrium?

o –1 N2(g) + 3H2(g) 2NH3(g) ∆H = –92 kJ mol

catalyst pressure temperature

A absent high low B absent low high C present high high D present low low

3 Use of the Data Booklet is relevant to this question.

The compound S2O7 is hydrolysed by water to produce sulfuric acid and oxygen only.

Which volume of oxygen, measured at room temperature and pressure, is evolved when 0.352 g of S2O7 is hydrolysed?

3 3 3 3 A 12 cm B 24 cm C 48 cm D 96 cm

4 Nitrogen, N2, and carbon monoxide, CO, both have Mr = 28.

The boiling point of N2 is 77 K.

The boiling point of CO is 82 K.

What could be responsible for this difference in boiling points?

A CO molecules have a permanent dipole, the N2 molecules are not polar.

B N2 has σ and π bonding, CO has σ bonding only.

C N2 has a strong N≡N bond, CO has a C=O bond.

D The CO molecule has more electrons than the N2 molecule.

© UCLES 2015 9701/11/O/N/15 3

5 Some car paints contain small flakes of silica, SiO2.

In the structure of solid SiO2

● each silicon atom is bonded to x oxygen atoms, ● each oxygen atom is bonded to y silicon atoms, ● each bond is a z type bond.

What is the correct combination of x, y and z in these statements?

x y z

A 2 1 covalent B 2 1 ionic C 4 2 covalent D 4 2 ionic

6 Solid sulfur consists of molecules made up of eight atoms covalently bonded together.

The bonding in sulfur dioxide is O=S=O.

–1 enthalpy change of combustion of S8, S8(s)= –2376kJ mol –1 energy required to break 1 mole S8(s) into gaseous atoms = 2232 kJ mol

–1 O=O bond enthalpy = 496 kJ mol

Using these data, what is the value of the S=O bond enthalpy?

–1 –1 –1 –1 A 239 kJ mol B 257 kJ mol C 319 kJ mol D 536 kJ mol

7 Use of the Data Booklet is relevant for this question.

In an experiment, the burning of 1.45 g (0.025 mol) of propanone was used to heat 100 g of water. The initial temperature of the water was 20.0 °C and the final temperature of the water was 78.0 °C.

Which experimental value for the enthalpy change of combustion for propanone can be calculated from these results?

–1 A –1304 kJ mol

–1 B –970 kJ mol

–1 C –352 kJ mol

–1 D –24.2 kJ mol

© UCLES 2015 9701/11/O/N/15 [Turn over 4

8 Which row correctly describes the electrodes that can be used in a diaphragm cell for the production of chlorine, hydrogen and sodium hydroxide?

anode cathode

A steel graphite B steel titanium C titanium graphite D titanium steel

9 Hexamine is a crystalline solid used as a fuel in portable stoves.

The diagram shows its skeletal structure.

N

NN N

What is the empirical formula of hexamine?

A CH2N B C3H6N2 C C4H8N4 D C6H12N4

10 A mixture of nitrogen and hydrogen gases, at a temperature of 500 K, was put into an evacuated 3 vessel of volume 6.0 dm . The vessel was then sealed.

N2(g) + 3H2(g) 2NH3(g)

The mixture was allowed to reach equilibrium. It was found that 7.2 mol of N2 and 12.0 mol of H2 were present in the equilibrium mixture. The value of the equilibrium constant, Kc, for this –2 equilibrium is 6.0 × 10 at 500 K.

What is the concentration of ammonia present in the equilibrium mixture at 500 K?

–3 A 0.58 mol dm

–3 B 0.76 mol dm

–3 C 3.5 mol dm

–3 D 27 mol dm

© UCLES 2015 9701/11/O/N/15 5

11 Ammonia is made by the Haber process. The reactants are nitrogen and hydrogen.

N2(g) + 3H2(g) 2NH3(g) ∆H –ve

What will increase the rate of the forward reaction?

A adding argon to the mixture but keeping the total volume constant B decreasing the temperature C increasing the total pressure by reducing the total volume at constant temperature D removing ammonia as it is made but keeping the total volume of the mixture the same

12 X is a Group II metal. The carbonate of X decomposes when heated in a Bunsen flame to give carbon dioxide and a white solid residue as the only products. This white solid residue is sparingly soluble in water. Even when large amounts of the solid residue are added to water the pH of the saturated solution is less than that of limewater.

What could be the identity of X?

A magnesium B calcium C strontium D barium

13 Rat poison needs to be insoluble in rain water but soluble at the low pH of stomach contents.

What is a suitable barium compound to use for rat poison?

A barium carbonate B barium chloride C barium hydroxide D barium sulfate

14 Use of the Data Booklet is relevant to this question.

Which of the elements sodium, magnesium, aluminium, silicon, phosphorus, sulfur and chlorine

• has a lower first ionisation energy than the preceding element in the Periodic Table,

• conducts electricity and

• has a lower atomic radius than the preceding element in the Periodic Table?

A aluminium B magnesium C phosphorus D sulfur

© UCLES 2015 9701/11/O/N/15 [Turn over 6

15 The melting points of the Period 3 elements sodium to aluminium are shown in the table.

element Na Mg Al

mp / K 371 923 932

Which factor explains the increase in melting points from sodium to aluminium?

A the changes in first ionisation energy from sodium to aluminium B the increase in electronegativity from sodium to aluminium

C the increase in the Ar of the elements from sodium to aluminium D the increase in the number of outer electrons in each atom from sodium to aluminium

16 X is the oxide of a Period 3 element. X reacts with water to give an acidic solution.

A solution is prepared by reacting 0.100 g of X with excess water. This solution was neutralised 3 –3 by exactly 25.0 cm of 0.100 mol dm sodium hydroxide solution.

What could be the identity of X?

A Al 2O3 B MgO C P4O10 D SO3

17 Which statement about bromine is correct?

A Bromine is insoluble in non-polar solvents. B Bromine vapour is more dense than air. C Bromine will not vaporise significantly under normal conditions. D Gaseous bromine is purple.

18 The addition of aqueous silver nitrate to aqueous barium chloride produces a white precipitate which dissolves in excess dilute aqueous ammonia to form a colourless solution.

The addition of excess dilute nitric acid to the colourless solution produces a white precipitate, Z.

What is Z?

A AgCl B BaCl 2 C Ba(NO3)2 D NH4NO3

© UCLES 2015 9701/11/O/N/15 7

19 Element X forms a pollutant oxide Y. Y can be further oxidised to Z. Two students made the following statements.

Student P ‘The molecule of Y contains lone pairs of electrons.’ Student Q ‘The oxidation number of X increases by 1 from Y to Z.’

X could be carbon or nitrogen or sulfur.

Which student(s) made a correct statement?

A P only B Q only C both P and Q D neither P nor Q

20 How many isomeric esters have the molecular formula C4H8O2?

A 2 B 3 C 4 D 5

21 A new jet fuel has been produced that is a mixture of different structural isomers of compound Q.

Q

Which skeletal formula represents a structural isomer of Q?

AB

CD

© UCLES 2015 9701/11/O/N/15 [Turn over 8

22 Crude oil is a mixture of many hydrocarbons ranging in size from 1 to 40 carbon atoms per molecule. The alkanes in crude oil can be separated because they have different boiling points.

The table below shows the boiling points of some alkanes.

boiling point alkane Mr / °C

butane 0 58 pentane 36 72 69 86 2-methylbutane 28 72 dimethylpropane 10 72 2,3-dimethylbutane 58 86

What is the correct explanation for the difference in the boiling points of the three isomers with Mr = 72?

A Boiling point is dependent upon the length of the carbon chain only. B Increased branching on a carbon chain increases the boiling point. C Increased branching reduces the strength of the intermolecular hydrogen bonding. D Increased branching reduces the strength of the intermolecular van der Waals’ forces.

23 Compound Q contains three double bonds per molecule.

O O Y X CH2 CH CH2 C CH2 C OH

Q

Which bond, X or Y, will be ruptured by hot, concentrated acidified KMnO4 and how many lone pairs of electrons are present in one molecule of Q?

bond ruptured by hot, number of

concentrated acidified KMnO4 lone pairs

A X 5 B X 6 C Y 5 D Y 6

© UCLES 2015 9701/11/O/N/15 9

24 Which compound undergoes an SN1 substitution reaction with NaOH(aq)?

A CH3CH2CH2Br

B (CH3)3CCH2I

CH3 Cl C

D CH =CHCl 2

25 If the starting material is iodoethane, which sequence of reactions will produce propanoic acid as the main final product in good yield?

A add NaOH(aq), isolate the organic product, add acidified K2Cr2O7 and boil under reflux

B add NaOH(aq), isolate the organic product, add H2SO4(aq) and boil under reflux

C heat with HCN in ethanol, isolate the organic product, add H2SO4(aq) and boil under reflux

D heat with KCN in ethanol, isolate the organic product, add H2SO4(aq) and boil under reflux

26 Which compound cannot be oxidised by acidified potassium dichromate(VI) solution but does react with sodium metal?

A (CH3)3COH

B CH3COCH2CH3

C CH3CH2CH2CH2OH

D CH3CH2CH(OH)CH3

27 Butan-2-ol can be made by reducing X with H2 / Ni.

Butan-2-ol can be dehydrated to form Y and Z which are structural isomers of each other.

Which row is correct?

cis-trans isomerism X is is shown by

A an aldehyde both Y and Z B an aldehyde only one of Y and Z C a ketone both Y and Z D a ketone only one of Y and Z

© UCLES 2015 9701/11/O/N/15 [Turn over 10

28 Tollens’ reagent can be used to help identify compounds P, Q and R.

O O OH H C C CH C C 3 3 H C H H C CH 3 3 3 H

P Q R

Which compound(s) form a silver precipitate on warming with Tollens’ reagent?

A P and Q B P only C Q only D R only

29 Sorbitol is a naturally-occurring compound with a sweet taste. It is often used as a substitute for sucrose by the food industry.

H

HCOH

HCOH

HCOH

HCOH

HCOH

HCOH

H

sorbitol

How many chiral centres are present in sorbitol?

A 3 B 4 C 5 D 6

30 Which compound produces butan-2-ol and ethanoic acid on hydrolysis?

A CH3CO2CH(CH3)2

B CH3CO2CH(CH3)CH2CH3

C CH3CH(CH3)CO2CH2CH3

D CH3CH2CO2CH(CH3)CH2CH3

© UCLES 2015 9701/11/O/N/15 11

Section B

For each of the questions in this section, one or more of the three numbered statements 1 to 3 may be correct.

Decide whether each of the statements is or is not correct (you may find it helpful to put a tick against the statements that you consider to be correct).

The responses A to D should be selected on the basis of

A B C D

1, 2 and 3 1 and 2 2 and 3 1 only are only are only are is correct correct correct correct

No other combination of statements is used as a correct response.

31 Which statements about orbitals in a krypton atom are correct?

1 The 1s and 2s orbitals have the same energy as each other but different sizes. 2 The third energy level (n=3) has three subshells and nine orbitals. 3 The 3d subshell has five orbitals that have the same energy as each other in an isolated atom.

32 The Group IV elements carbon, silicon and germanium can all exist in the giant molecular structure which is also found in diamond. The bond lengths in these structures are given below.

element X C Si Ge

bond length X–X / nm 0.154 0.234 0.244

Why does the bond length increase down the group?

1 Orbital overlap decreases down the group. 2 Atomic radius increases down the group. 3 Nuclear charge increases down the group.

© UCLES 2015 9701/11/O/N/15 [Turn over 12

The responses A to D should be selected on the basis of

A B C D

1, 2 and 3 1 and 2 2 and 3 1 only are only are only are is correct correct correct correct

No other combination of statements is used as a correct response.

33 The salt NaCl O3 is used as a non-selective weedkiller.

On careful heating, this reaction occurs: 4NaCl O3 → NaCl + 3NaCl O4.

On strong heating this reaction occurs: NaCl O4 → NaCl + 2O2.

The overall reaction is 2NaCl O3 → 2NaCl + 3O2.

What do these equations show?

1 NaCl O3 can behave as an oxidising agent.

2 NaCl O3 can behave as a reducing agent. 3 The oxidation numbers of chlorine in the three compounds shown are +6, +8 and –1.

34 Which statements correctly describe an effect of a rise in temperature on a gas-phase reaction?

1 More particles now have energies greater than the activation energy. 2 The energy distribution profile changes with more particles having the most probable energy. 3 The activation energy of the reaction is decreased.

35 Which statements concerning the Group II elements magnesium, calcium and barium are correct?

1 Their reactivity increases with increasing relative atomic mass. 2 The oxidation number exhibited in their stable compounds is +2. 3 On strong heating, their nitrates give off oxygen only.

36 Sulfur dioxide is used as a food preservative.

Which statements about sulfur dioxide, SO2, are correct?

1 SO2 behaves as an antioxidant.

2– 2 Aqueous SO2 contains SO3 ions.

3 SO2 inhibits the growth of mould and yeasts.

© UCLES 2015 9701/11/O/N/15 13

37 An oxidising agent that can oxidise ethanal to ethanoic acid, or to ethanoate ions, will also oxidise methanoic acid, HCO2H, to carbon dioxide and water.

Which reagents, on heating, will react differently with HCO2H and CH3CO2H?

1 Na2CO3(aq) 2 Fehling’s reagent

3 dilute acidified KMnO4

38 Each of the compounds below is treated separately with excess NaBH4. The product of each reaction is then heated with excess concentrated H2SO4.

In each case, one or more products are formed with molecular formula C7H10.

Which compounds give only one final product with the molecular formula C7H10?

1 2 3

O O

O

O O O

39 Which reactions result in the formation of propanoic acid?

1 CH3CH2CO2Na with dilute H2SO4(aq)

+ – 2 CH3CH=CHCH3 with hot, concentrated H / MnO4 (aq)

+ 2– 3 CH3CH2OH with H / Cr2O7 (aq)

40 The diagram shows the structure of propanamide.

H H O H

H C C C N

H H H

propanamide

Which statements about the hydrolysis of propanamide are correct?

1 Propanamide can be hydrolysed by heating under reflux with H2SO4(aq). 2 Propanamide can be hydrolysed by heating under reflux with NaOH(aq). 3 Propanamide can be hydrolysed by cold water.

© UCLES 2015 9701/11/O/N/15 Cambridge International Examinations Cambridge International Advanced Subsidiary and Advanced Level *7466726655*

CHEMISTRY 9701/21

Paper 2 Structured Questions AS Core October/November 2015

1 hour 15 minutes Candidates answer on the Question Paper. Additional Materials: Data Booklet

READ THESE INSTRUCTIONS FIRST

Write your Centre number, candidate number and name on all the work you hand in. Write in dark blue or black pen. You may use an HB pencil for any diagrams or graphs. Do not use staples, paper clips, glue or correction fl uid. DO NOT WRITE IN ANY BARCODES.

Answer all questions. Electronic calculators may be used. You may lose marks if you do not show your working or if you do not use appropriate units. A Data Booklet is provided.

At the end of the examination, fasten all your work securely together. The number of marks is given in brackets [ ] at the end of each question or part question.

This document consists of 8 printed pages.

IB15 11_9701_21/FP © UCLES 2015 [Turn over 2

Answer all the questions in the spaces provided.

1 Aluminium is a metal in Period 3 and Group III of the Periodic Table.

(a) Describe the structure of solid aluminium.

......

......

...... [2]

(b) A common use of aluminium is to make the conducting cables in long distance overhead power lines.

(i) Suggest two properties of aluminium that make it suitable for this use.

......

......

...... [2]

The cables are attached to pylons by ceramic supports.

(ii) Describe the structure of a ceramic material.

......

...... [1]

(iii) State the property of a ceramic material that makes it suitable for this use.

......

...... [1]

© UCLES 2015 9701/21/O/N/15 3

(c) Aluminium reacts with chlorine to form a white, solid chloride that contains 79.7% chlorine and sublimes (changes straight from a solid to a gas) at 180 °C.

(i) Describe the structure and bonding in this compound. Suggest how it explains the low sublimation temperature.

......

......

......

......

...... [2]

(ii) Calculate the empirical formula of the chloride. You must show your working.

empirical formula = ...... [2]

At 200 °C and 100 kPa, a 1.36 g sample of this chloride occupied a volume of 200 cm3.

(iii) Calculate the relative molecular mass, Mr, of the chloride. Give your answer to three signifi cant fi gures.

Mr = ...... [2]

(iv) Deduce the molecular formula of this chloride at 200 °C.

...... [1]

[Total: 13]

© UCLES 2015 9701/21/O/N/15 [Turn over 4

2 (a) (i) Explain the meaning of the term enthalpy change of formation.

......

......

...... [2]

(ii) Give the equation for the reaction for which the enthalpy change corresponds to the

standard enthalpy change of formation of liquid sulfur trioxide, SO3. Include state symbols.

...... [1]

(b) Ammonia is manufactured by the Haber process.

N2(g) + 3H2(g) 2NH3(g)

(i) Use bond energies from the Data Booklet to calculate the enthalpy change of reaction for the Haber process. Include a sign in your answer.

enthalpy change ...... kJ mol–1 [3]

(ii) State the essential operating conditions for the Haber process.

......

......

...... [3]

(iii) Explain the choices of temperature and pressure for the Haber process.

......

......

......

......

...... [4]

© UCLES 2015 9701/21/O/N/15 5

(c) One of the major uses of ammonia is in the manufacture of fertilisers such as diammonium

hydrogen phosphate, (NH4)2HPO4.

(i) Write an equation for the formation of diammonium hydrogen phosphate by the reaction

between ammonia and phosphoric acid, H3PO4.

...... [1]

(ii) Explain this reaction in terms of the Brønsted-Lowry theory.

......

......

......

...... [2]

(d) The use of nitrate fertilisers can give rise to environmental consequences in terms of effects on both rivers and the atmosphere.

(i) Explain how the uncontrolled use of nitrate fertilisers can result in a severe reduction in water quality in rivers.

......

......

......

...... [3]

(ii) Oxides of nitrogen are produced by the action of bacteria on nitrate fertilisers.

Explain the problems associated with the release of oxides of nitrogen into the atmosphere. Include an equation in your answer.

......

......

...... [2]

[Total: 21]

© UCLES 2015 9701/21/O/N/15 [Turn over 6

3 , C7H16, is an undesirable component of petrol as it burns explosively causing ‘knocking’ in an engine.

(a) There are nine structural isomers with the formula C7H16, only two of which contain chiral centres.

(i) Explain the meanings of the terms structural isomers and chiral.

structural isomers ......

......

......

chiral ......

......

...... [2]

(ii) Give the structures and names of the two structural isomers of C7H16 which contain a chiral centre.

[4]

(b) (i) Write an equation for the complete combustion of heptane.

...... [1]

(ii) Write an equation for the incomplete combustion of heptane leading to the production of a solid pollutant.

...... [1]

(iii) Incomplete combustion can also lead to emission of unburnt hydrocarbons.

State one environmental consequence of this.

...... [1]

© UCLES 2015 9701/21/O/N/15 7

(c) The reaction of heptane with chlorine in the presence of UV light produces a wide variety of products.

Formation of the monochloroheptanes can be represented by the following equation.

C7H16 + Cl 2 → C7H15Cl + HCl

(i) Name the mechanism of the reaction between heptane and chlorine in the presence of UV light.

...... [1]

(ii) Describe this mechanism, using suitable equations and including the names of each stage in the process.

......

......

......

......

......

......

......

......

...... [5]

[Total: 15]

© UCLES 2015 9701/21/O/N/15 [Turn over 8

4 Some reactions involving ethanol are shown.

reaction 1 distil with CH CH Cl CH CH OH Y 3 2 3 2 2– + reaction 2 Cr2O7 + H heat with NaOH in ethanol

X

(a) (i) Give an equation for reaction 2 including the reagent needed for the conversion.

...... [2]

(ii) State the reagent and conditions required for reaction 1.

...... [2]

(b) (i) Identify the organic product X.

...... [1]

(ii) Nitric acid is added to the products of reaction of CH3CH2Cl with NaOH in ethanol. Silver nitrate solution is then added to this mixture.

State what you would observe.

...... [1]

(iii) Write an ionic equation, including state symbols, for the reaction responsible for the observation in (ii).

...... [1]

(c) (i) Identify the organic product Y which is distilled out of the reaction mixture.

...... [1]

(ii) Explain, in terms of the properties of and intermolecular forces in CH3CH2OH and Y, why the chosen conditions for the reaction ensure that Y is the product.

......

......

...... [3]

[Total: 11]

Permission to reproduce items where third-party owned material protected by copyright is included has been sought and cleared where possible. Every reasonable effort has been made by the publisher (UCLES) to trace copyright holders, but if any items requiring clearance have unwittingly been included, the publisher will be pleased to make amends at the earliest possible opportunity.

To avoid the issue of disclosure of answer-related information to candidates, all copyright acknowledgements are reproduced online in the Cambridge International Examinations Copyright Acknowledgements Booklet. This is produced for each series of examinations and is freely available to download at www.cie.org.uk after the live examination series.

Cambridge International Examinations is part of the Cambridge Assessment Group. Cambridge Assessment is the brand name of University of Cambridge Local Examinations Syndicate (UCLES), which is itself a department of the University of Cambridge.

© UCLES 2015 9701/21/O/N/15

Cambridge International Examinations Cambridge International Advanced Subsidiary and Advanced Level

CHEMISTRY 9701/11

Paper 1 Multiple Choice October/November 2016

1 hour

Additional Materials: Multiple Choice Answer Sheet *1087312072* Soft clean eraser Soft pencil (type B or HB is recommended) Data Booklet

READ THESE INSTRUCTIONS FIRST

Write in soft pencil. Do not use staples, paper clips, glue or correction fluid. Write your name, Centre number and candidate number on the Answer Sheet in the spaces provided unless this has been done for you. DO NOT WRITE IN ANY BARCODES.

There are forty questions on this paper. Answer all questions. For each question there are four possible answers A, B, C and D. Choose the one you consider correct and record your choice in soft pencil on the separate Answer Sheet.

Read the instructions on the Answer Sheet very carefully.

Each correct answer will score one mark. A mark will not be deducted for a wrong answer. Any rough working should be done in this booklet. Electronic calculators may be used.

This document consists of 16 printed pages.

IB16 11_9701_11/4RP © UCLES 2016 [Turn over

2

Section A

For each question there are four possible answers, A, B, C and D. Choose the one you consider to be correct.

Use of the Data Booklet may be appropriate for some questions.

1 The diagram represents, for a given temperature, the Boltzmann distribution of the kinetic energies of the molecules in a mixture of two gases that react together. The activation energy for the reaction, Ea , is marked.

proportion of molecules with a given energy

0 E 0 a molecular energy

The dotted curves below show the Boltzmann distribution for the same reaction at a higher temperature. On these diagrams, H represents the activation energy at the higher temperature.

Which diagram is correct?

A B

proportion of proportion of molecules molecules with a with a given energy given energy

0 E 0 E 0 a molecular 0 a molecular energy H energy H C D

proportion of proportion of molecules molecules with a with a given energy given energy 0 0 0 E 0 E a molecular a molecular energy energy H H

© UCLES 2016 9701/11/O/N/16 3

2 In the ideal gas equation, pV = nRT, what are the units of n and T ?

n T

A no units °C B no units K C mol °C D mol K

2– 2+ 3 The reaction between acidified dichromate(VI) ions, Cr2O7 , and aqueous Fe ions results in the dichromate(VI) ions being reduced to Cr3+ ions.

What is the correct equation for this reaction?

2– 2+ + 3+ 3+ A Cr2O7 + Fe + 14H → 2Cr + Fe + 7H2O

2– 2+ + 3+ 3+ B Cr2O7 + 2Fe + 14H → 2Cr + 2Fe + 7H2O

2– 2+ + 3+ 3+ C Cr2O7 + 3Fe + 14H → 2Cr + 3Fe + 7H2O

2– 2+ + 3+ 3+ D Cr2O7 + 6Fe + 14H → 2Cr + 6Fe + 7H2O

4 Sodium azide, NaN3 is an explosive used to inflate airbags in cars when they crash. It consists of positive sodium ions and negative azide ions.

What are the numbers of electrons in the sodium ion and the azide ion?

sodium ion azide ion

A 10 20 B 10 22 C 12 20 D 12 22

5 The 68Ge isotope is medically useful because it undergoes a natural radioactive process to give an isotope of a different element, 68X, which can be used to detect tumours. This transformation of 68Ge occurs when an electron enters the nucleus and changes a proton into a neutron.

Which statement about the composition of an atom of 68X is correct?

A It has 4 electrons in its outer p orbitals. B It has 13 electrons in its outer shell. C It has 37 neutrons. D Its proton number is 32.

© UCLES 2016 9701/11/O/N/16 [Turn over 4

6 Histamine is produced in the body to help fight infection. Its shape allows it to fit into receptors which expand blood vessels.

H

H H H N C

x N C CC N y z H C H H H H histamine

What are the bond angles x, y and z in histamine, from the smallest to the largest?

smallest largest

bond angle bond angle

A x y z B y x z C y z x D z y x

7 The approximate percentage composition of the atmospheres on four different planets is given in the table below.

Which mixture of gases has the greatest density?

major gases / planet % by number of molecules

H2 He CH4

A Jupiter 89.8 10.2 0.0 B Neptune 80.0 19.0 1.0 C Saturn 96.3 3.3 0.4 D Uranus 82.5 15.2 2.3

8 An important reaction in the manufacture of nitric acid is the catalytic oxidation of ammonia.

4NH3(g) + 5O2(g) 4NO(g) + 6H2O(g)

For every mole of O2 that reacts in this way, 181.8 kJ of energy are released.

5 A factory makes 2.50 × 10 mol of NO every day.

How much energy, in kJ, is released every day?

A 3.64 × 107 B 4.55 × 107 C 5.68 × 107 D 2.27 × 108

© UCLES 2016 9701/11/O/N/16 5

9 Sodium chromate(VI), Na2CrO4, is manufactured by heating chromite, FeCr2O4, with sodium 2– carbonate in an oxidising atmosphere. Chromite contains Cr2O4 ions.

1 2FeCr2O4 + 4Na2CO3 + 3 2 O2 → 4Na2CrO4 + Fe2O3 + 4CO2

What happens in this reaction?

A Chromium and iron are the only elements oxidised. B Chromium, iron and carbon are oxidised. C Only chromium is oxidised. D Only iron is oxidised.

10 The table shows the partial pressures in an equilibrium mixture formed by the Haber process.

substance partial pressure / kPa

nitrogen 7000 3H2(g) + N2(g) 2NH3(g)

hydrogen 8000 ammonia 4000

What is the numerical value of the equilibrium constant, Kp, for this reaction?

A 4.46 × 10–9

B 4.76 × 10–5

C 7.14 × 10–5

D 2.24 × 108

11 An autocatalytic reaction is a reaction in which one of the products catalyses the reaction.

Which curve would be obtained if the rate of an autocatalytic reaction is plotted against time?

A B C D

rate rate rate rate

0 0 0 0 0 time 0 time 0 time 0 time

© UCLES 2016 9701/11/O/N/16 [Turn over 6

3 12 1.15 g of a metallic element needs 300 cm of oxygen for complete reaction, at 298 K and 1 atm pressure, to form an oxide which contains O2– ions.

What could be the identity of this metallic element?

A calcium B magnesium C potassium D sodium

13 The diagram shows the melting points of eight elements with consecutive atomic numbers.

Which element could be sodium?

D melting point / K C A

B

atomic number

14 The properties of chlorine, bromine and their compounds are compared.

Which property is smaller for chlorine than for bromine?

A bond strength of the hydrogen-halide bond B first ionisation energy

C solubility of the silver halide in NH3(aq) D strength of the van der Waals’ forces between molecules of the element

© UCLES 2016 9701/11/O/N/16 7

15 At 550 °C nitrogen dioxide reacts with unburnt hydrocarbon fragments such as CH3• in the catalytic converter of a motor vehicle.

1 4CH3• + 7NO2 → 3 2 N2 + 4CO2 + 6H2O

The following table lists types of energy change for this reaction and possible reasons for them.

Which row gives the energy change for this reaction and the reason for it?

energy change reason why the reaction is endothermic

of reaction or exothermic

A endothermic chemical energy is converted to heat energy B endothermic the N≡N bond energy is very high

C exothermic CO2 and H2O have large negative values

D exothermic double bonds are broken in NO2

16 Magnesium nitrate, Mg(NO3)2, will decompose when heated to give a white solid and a mixture of gases. One of the gases released is an oxide of nitrogen, X.

7.4 g of anhydrous magnesium nitrate is heated until no further reaction takes place.

What mass of X is produced?

A 1.5 g B 2.3 g C 3.0 g D 4.6 g

17 Compound T is a white crystalline solid.

When a sample of T was mixed with aqueous sodium hydroxide and heated, a pungent smelling gas was produced which turned damp red litmus paper blue. This same gas produced dense white smoke with hydrogen chloride gas.

Further testing of a solution of T with barium chloride solution produced a dense white precipitate which did not dissolve when dilute hydrochloric acid was added to the mixture.

What is the identity of compound T?

A ammonium carbonate B ammonium sulfate C sodium carbonate D sodium sulfate

© UCLES 2016 9701/11/O/N/16 [Turn over 8

18 Which row of the table gives correct comparisons between the solubilities of calcium and barium hydroxide and the thermal stabilities of calcium and barium carbonate?

solubility thermal stability

calcium hydroxide barium hydroxide calcium carbonate barium carbonate

A higher lower higher lower B higher lower lower higher C lower higher higher lower D lower higher lower higher

19 X, Y and Z represent different halogens. The table shows the results of nine experiments in which aqueous solutions of X2, Y2 and Z2 were separately added to separate aqueous solutions – – – containing X , Y and Z ions.

– – – X (aq) Y (aq) Z (aq)

X2(aq) no reaction no reaction no reaction

Y2(aq) X2 formed no reaction Z2 formed

Z2(aq) X2 formed no reaction no reaction

– – – Which row of the following table contains the ions X , Y and Z in order of their decreasing strength as reducing agents?

strongest weakest

– – – A X Y Z

– – – B X Z Y

– – – C Y Z X

– – – D Z X Y

© UCLES 2016 9701/11/O/N/16 9

20 In 1865 Kekulé suggested a ring structure for benzene, C6H6, in which a hydrogen atom is attached to each carbon atom.

H

H C H C C

CC H C H

H

Kekulé structure

In this structure all of the bonds remain in the places shown. Assuming this is the structure of benzene, how many isomers of dichlorobenzene, C6H4Cl 2, would exist?

A 3 B 4 C 5 D 6

21 The diagram shows the structure of vitamin A.

CH3 CH3 H3CCH3

OH

CH3

vitamin A

How many chiral centres are present in one vitamin A molecule?

A 0 B 1 C 2 D 3

22 PVC is difficult to dispose of. Two possible methods are burying it in landfill sites and disposal by combustion.

Which row of the table is correct?

rate of biodegradation gases produced when

of PVC in landfill sites PVC combusts

A fast CO2, H2O, HCl

B fast CO2, H2O, Cl 2

C slow CO2, H2O, Cl 2

D slow CO2, H2O, HCl

© UCLES 2016 9701/11/O/N/16 [Turn over 10

23 Part of the structure of a fungicide, strobilurin, is shown. R and R' are inert groups.

O OCH3

R' R

strobilurin

In this reaction, strobilurin is warmed with aqueous sulfuric acid producing compound X. Compound X is then treated with hydrogen in the presence of a nickel catalyst producing compound Y.

What could be the structure of compound Y?

AB

O OH HO OCH3

R' R' R R

CD

OH O OH OH OH R' R' R R

OH

24 Chloroethane can be used to make sodium propanoate.

chloroethane → Q → sodium propanoate

The intermediate, Q, is hydrolysed with boiling aqueous sodium hydroxide to give sodium propanoate.

Which reagent would produce the intermediate, Q, from chloroethane?

A concentrated ammonia solution B dilute sulfuric acid C hydrogen cyanide in water D potassium cyanide in ethanol

© UCLES 2016 9701/11/O/N/16 11

25 X, Y and Z are three isomeric alcohols.

X CH3CH2CH2CH2CH2OH

Y CH3CH2CH(OH)CH2CH3

Z (CH3)2C(OH)CH2CH3

Two or more of these alcohols react with mild oxidising agents.

One of these alcohols, when dehydrated, will give a pair of cis-trans isomers with molecular formula C5H10.

Which row is correct?

reacts with mild gives cis-trans

oxidising reagents isomers

A X, Y and Z Y only B X, Y and Z Z only C X and Y only Y only D X and Y only Z only

26 Propanone reacts with an aqueous mixture of HCN and NaCN by a nucleophilic addition mechanism.

The first stage of the mechanism involves attack by cyanide ions.

Which diagram correctly represents this?

A B

CH3 CH3 – δ+ δ– – δ+ δ– NC CO NC CO

CH3 CH3

C D

CH3 CH3 – δ+ δ– – δ+ δ– CN CO NC CO CH CH 3 3

© UCLES 2016 9701/11/O/N/16 [Turn over 12

27 P, Q and R are carbonyl compounds.

CH2 CH2 CH2 O C C O O C C O O C C O

H3C H H H H3C CH3

P Q R

Fehling’s solution can be used to help identify these compounds.

Which compounds form a red-brown precipitate on warming with Fehling’s solution?

A P, Q and R B P and Q only C P only D Q only

28 Which reaction would not give ethanoic acid?

A heating ethanenitrile under reflux with dilute sodium hydroxide B heating ethanenitrile under reflux with dilute sulfuric acid

C heating ethanal under reflux with acidified sodium dichromate(VI)

D heating ethanol under reflux with acidified sodium dichromate(VI)

29 Which formula represents an ester that will form propanoic acid on hydrolysis with dilute sulfuric acid?

ABC D

O O O O

O O O O

© UCLES 2016 9701/11/O/N/16 13

30 A solvent, X, used in printing inks has a molecular formula C6H10O4. It may be made by reacting ethane-1,2-diol with ethanoic acid in the presence of an acid catalyst.

H H

HOC C OH

H H

ethane-1,2-diol

What is the structure of solvent X?

AB

H H O O H H O

HOC C O C C OOC C C

H H CH3 H3C H H CH3

C D

O H H O O H H O

C C C C C C C O C

H C O O CH H C O CH 3 H H 3 3 H H 3

© UCLES 2016 9701/11/O/N/16 [Turn over 14

Section B

For each of the questions in this section, one or more of the three numbered statements 1 to 3 may be correct.

Decide whether each of the statements is or is not correct (you may find it helpful to put a tick against the statements that you consider to be correct).

The responses A to D should be selected on the basis of

A B C D

1, 2 and 3 1 and 2 2 and 3 1 only are only are only are is correct correct correct correct

No other combination of statements is used as a correct response.

31 A sample of boron contains aluminium as the only impurity. A mass spectrum of the mixture shows three lines corresponding to three ions, X+, Y+ and Z+.

ion X+ Y+ Z+

m / e 10 11 27 percentage 15.52 74.48 10.00 abundance

Which statements are correct?

1 There are more electrons in Z+ than in X+.

2 The Ar of boron in the sample is 10.83 to four significant figures. 3 There are more protons in Y+ than in X+.

32 Which elements can form π bonds in their compounds?

1 carbon 2 oxygen 3 nitrogen

33 For which enthalpy changes is the value of ∆H always negative?

1 combustion 2 hydration 3 solution

© UCLES 2016 9701/11/O/N/16 15

34 In the manufacture of sulfuric acid, the following exothermic reaction occurs.

2SO2(g) + O2(g) 2SO3(g)

Which changes will move the position of the equilibrium to the right?

1 increasing the pressure 2 increasing the temperature 3 using twice as much catalyst

–3 35 Water is added to anhydrous aluminium chloride to make a 0.1 mol dm solution.

Which observations are correct?

1 The reaction is endothermic. 2 The solution is acidic.

3+ 3 The solution contains the ion [Al (H2O)6] .

36 Catalytic converters are used to modify exhaust emissions from motor vehicles.

Which reactions occur in catalytic converters?

1 1 C8H18 + 12 2 O2 → 8CO2 + 9H2O

2 2NO + 2CO → N2 + 2CO2

3 NO + SO3 → NO2 + SO2

37 Which compounds would produce a carboxylic acid and a ketone when treated with hot, concentrated, acidified KMnO4?

1 23

38 Bromoethane reacts with NaOH in different ways depending on the solvent used.

Which statements about these reactions are correct?

solvent used main organic product

1 water ethane-1,2-diol 2 ethanol ethene 3 water ethanol

© UCLES 2016 9701/11/O/N/16 [Turn over 16

The responses A to D should be selected on the basis of

A B C D

1, 2 and 3 1 and 2 2 and 3 1 only are only are only are is correct correct correct correct

No other combination of statements is used as a correct response.

39 Several structural isomers of C3H6O3 are listed below.

HOCH2COCH2OH

HOCH2CH(OH)CHO

HOCH2CH2CO2H

CH3CH(OH)CO2H

Which statements about these structural isomers are correct?

1 One mole of each reacts with two moles of sodium. 2 Only one of the isomers contains a tertiary alcohol group. 3 They all contain a primary alcohol group.

40 When onions are peeled in air, the reaction shown is thought to occur.

H → 2CH3CH2C + 2H2O + 2[O] H2SO4 + H2S + 2CH3CH2CHO

SO

Which tests would give a positive reaction with the organic product?

1 warming with Tollens’ reagent

2 warming with acidified potassium manganate(VII) 3 warming with alkaline aqueous iodine

Permission to reproduce items where third-party owned material protected by copyright is included has been sought and cleared where possible. Every reasonable effort has been made by the publisher (UCLES) to trace copyright holders, but if any items requiring clearance have unwittingly been included, the publisher will be pleased to make amends at the earliest possible opportunity.

To avoid the issue of disclosure of answer-related information to candidates, all copyright acknowledgements are reproduced online in the Cambridge International Examinations Copyright Acknowledgements Booklet. This is produced for each series of examinations and is freely available to download at www.cie.org.uk after the live examination series.

Cambridge International Examinations is part of the Cambridge Assessment Group. Cambridge Assessment is the brand name of University of Cambridge Local Examinations Syndicate (UCLES), which is itself a department of the University of Cambridge.

© UCLES 2016 9701/11/O/N/16 Cambridge International Examinations Cambridge International Advanced Subsidiary and Advanced Level *2293807595*

CHEMISTRY 9701/21

Paper 2 AS Level Structured Questions October/November 2016

1 hour 15 minutes Candidates answer on the Question Paper. Additional Materials: Data Booklet

READ THESE INSTRUCTIONS FIRST

Write your Centre number, candidate number and name on all the work you hand in. Write in dark blue or black pen. You may use an HB pencil for any diagrams or graphs. Do not use staples, paper clips, glue or correction fluid. DO NOT WRITE IN ANY BARCODES.

Answer all questions. Electronic calculators may be used. You may lose marks if you do not show your working or if you do not use appropriate units. A Data Booklet is provided.

At the end of the examination, fasten all your work securely together. The number of marks is given in brackets [ ] at the end of each question or part question.

This document consists of 10 printed pages and 2 blank pages.

IB16 11_9701_21/6RP R © UCLES 2016 [Turn over 2

Answer all the questions in the spaces provided.

1 A 0.17 g sample of a Group 14 chloride, XCl 4, reacted with water to produce an oxide, XO2, and HCl.

equation 1 XCl 4(s) + 2H2O(l) → XO2(s) + 4HCl (aq)

The HCl produced was absorbed in 100 cm3 of 0.10 mol dm–3 sodium hydroxide solution (an excess).

In a titration, the unreacted sodium hydroxide solution required 30.0 cm3 of 0.20 mol dm–3 hydrochloric acid for complete neutralisation.

(a) Calculate the amount, in moles, of hydrochloric acid used in the titration to neutralise the unreacted sodium hydroxide solution.

amount = ...... mol [1]

(b) Write the equation for the reaction between hydrochloric acid and sodium hydroxide.

...... [1]

(c) Calculate the amount, in moles, of sodium hydroxide neutralised in the titration.

amount = ...... mol [1]

(d) Calculate the amount, in moles, of sodium hydroxide that reacted with the HCl produced by the reaction in equation 1.

amount = ...... mol [1]

(e) Calculate the amount, in moles, of HCl produced by the reaction in equation 1.

amount = ...... mol [1]

© UCLES 2016 9701/21/O/N/16 3

(f) Calculate the amount, in moles, of XCl 4 in the original 0.17 g sample.

amount = ...... mol [1]

(g) Calculate the molecular mass, Mr, of XCl 4.

Mr = ...... [1]

(h) Calculate the relative atomic mass, Ar, of X and suggest its identity.

Ar of X = ......

identity of X ...... [2]

[Total: 9]

© UCLES 2016 9701/21/O/N/16 [Turn over 4

2 For many compounds the enthalpy change of formation cannot be calculated directly. An indirect method based on enthalpy changes of combustion can be used.

The enthalpy change of combustion can be found by a calorimetry experiment in which the heat energy given off during combustion is used to heat a known mass of water and the temperature change recorded.

(a) (i) Explain the meaning of the term standard enthalpy change of combustion.

......

......

...... [3]

(ii) Write the equation for the complete combustion of ethanol, C2H5OH.

...... [1]

(b) In an experiment to determine the enthalpy change of combustion of ethanol, 0.23 g of ethanol was burned and the heat given off raised the temperature of 100 g of water by 16.3 °C.

(i) Calculate the heat energy change, q, during the combustion of 0.23 g of ethanol.

q = ...... J [1]

(ii) Calculate the enthalpy change on burning 1 mole of ethanol. Include a sign in your answer.

∆H = ...... kJ mol–1 [1]

(iii) Suggest two reasons why the value for the enthalpy change of combustion of ethanol determined by a simple laboratory calorimetry experiment is likely to be lower than the true value.

......

......

......

...... [2]

© UCLES 2016 9701/21/O/N/16 5

(c) The table gives some enthalpy change of combustion values.

enthalpy change of substance combustion / kJ mol–1 C(s) –393.5

H2(g) –285.8

C3H7OH(l) –2021.0

(i) Construct a labelled energy cycle to show how these values could be used to calculate the

enthalpy change of formation of C3H7OH(l), ∆Hf.

1 ∆Hf 3C(s) + 4H2(g) + 2 O2(g) C3H7OH(l)

[3]

(ii) Calculate the enthalpy change of formation, ∆Hf, of C3H7OH(l).

–1 ∆Hf = ...... kJ mol [2]

[Total: 13]

© UCLES 2016 9701/21/O/N/16 [Turn over 6

3 The elements in Group 2 and their compounds show various trends in their physical and chemical properties.

(a) The graph below shows the radius values of the atoms and 2+ ions of the elements in Group 2.

0.25

0.20

0.15 radius / nm 0.10

0.05

0 Be Mg Ca Sr Ba Ra element

(i) Explain why both lines show a steady increase in the values of the radii down the group.

......

......

...... [2]

(ii) State and explain which line represents the atomic radii and which represents the ionic radii.

......

......

...... [2]

(b) L is a salt of a Group 2 element M. When L is heated strongly a brown gas is observed and a white solid remains.

The white solid dissolves in water to form a colourless solution of the metal hydroxide M(OH)2.

Addition of dilute sulfuric acid to this colourless solution produces a dense white precipitate.

(i) Identify the anion in salt L.

...... [1]

© UCLES 2016 9701/21/O/N/16 7

(ii) Identify the element M and write an ionic equation for the formation of the white precipitate with sulfuric acid.

M = ......

equation ...... [1]

(iii) Give the formula of salt L and use it to write an equation for the thermal decomposition of salt L.

formula of salt L ......

equation ...... [2]

(c) Calcium carbonate and calcium hydroxide can both be used in agriculture to neutralise acidic soils.

(i) Write ionic equations for the neutralisation of acid by each of calcium hydroxide and calcium carbonate.

calcium hydroxide ......

calcium carbonate ...... [2]

(ii) Suggest and explain why calcium carbonate is a better choice than calcium hydroxide for this purpose in areas of high rainfall.

......

......

...... [2]

(d) Magnesium reacts with both cold water and steam.

Give the formula of the magnesium-containing product of each of these reactions.

with cold water ......

with steam ...... [2]

[Total: 14]

© UCLES 2016 9701/21/O/N/16 [Turn over 8

4 In each section of this question an organic compound is shown. For each compound give its name and answer the questions about it.

(a) CH3CH2CH(CH3)CH=CHCH3

(i) name ...... [1]

(ii) This compound shows stereoisomerism.

Define stereoisomerism.

......

......

...... [1]

(iii) State and explain how many stereoisomers of this structure there are.

......

......

......

......

......

...... [4]

(b) (CH3)2C=C(CH3)2

(i) name ...... [1]

(ii) Draw the skeletal formula of the organic product of the reaction of this compound with cold, dilute, acidified manganate(VII) ions.

[1]

(iii) Name the organic product of the reaction of this compound with hot, concentrated, acidified manganate(VII) ions.

...... [1]

(iv) Draw the structure of part of a molecule of the addition polymer formed from this compound, showing exactly three repeat units.

[1]

© UCLES 2016 9701/21/O/N/16 9

(c) (CH3)2C=CH2

(i) name ...... [1]

(ii) Complete the mechanism for the reaction of this compound with hydrogen bromide. Include all necessary curly arrows, lone pairs, charges and partial charges.

CH3 CH3

H3CCC H2 H3CCC H3

Br H

– 2-bromomethylpropane Br Br [4]

(iii) Explain fully why 2-bromomethylpropane is the major product of this reaction while only relatively small amounts of 1-bromomethylpropane are produced.

......

......

......

......

......

......

...... [3]

[Total: 18]

© UCLES 2016 9701/21/O/N/16 [Turn over 10

5 A sequence of reactions is shown starting with an alcohol, C3H7OH.

reaction 1 reaction 2 CH3CH2CO2HC3H7OH CH3CH2CHO

reaction 3

P

(a) Draw the skeletal formula of the alcohol C3H7OH.

[1]

(b) State the reagents and conditions needed for reaction 1.

......

...... [2]

(c) State the reagents and conditions needed for reaction 2.

......

...... [2]

(d) Name P, the organic product of reaction 3.

...... [1]

[Total: 6]

© UCLES 2016 9701/21/O/N/16

Cambridge International Examinations Cambridge International Advanced Subsidiary and Advanced Level

CHEMISTRY 9701/11

Paper 1 Multiple Choice October/November 2017

1 hour

Additional Materials: Multiple Choice Answer Sheet *3286301793* Soft clean eraser Soft pencil (type B or HB is recommended) Data Booklet

READ THESE INSTRUCTIONS FIRST

Write in soft pencil. Do not use staples, paper clips, glue or correction fluid. Write your name, Centre number and candidate number on the Answer Sheet in the spaces provided unless this has been done for you. DO NOT WRITE IN ANY BARCODES.

There are forty questions on this paper. Answer all questions. For each question there are four possible answers A, B, C and D. Choose the one you consider correct and record your choice in soft pencil on the separate Answer Sheet.

Read the instructions on the Answer Sheet very carefully.

Each correct answer will score one mark. A mark will not be deducted for a wrong answer. Any rough working should be done in this booklet. Electronic calculators may be used.

This document consists of 15 printed pages and 1 blank page.

IB17 11_9701_11/FP © UCLES 2017 [Turn over

2

Section A

For each question there are four possible answers, A, B, C and D. Choose the one you consider to be correct.

Use of the Data Booklet may be appropriate for some questions.

1 Which formula represents the empirical formula of a compound?

A C2H4O B C2H4O2 C C6H12 D H2O2

2 The relative first ionisation energies of four elements with consecutive atomic numbers below 20 are shown on the graph.

One of the elements reacts with hydrogen to form a covalent compound with formula HX.

Which element could be X?

B

A

first ionisation energy / kJ mol–1

D C

atomic number

3 In which structure are three atoms bonded together in a straight line?

A poly(ethene), ] ()CH2CH2n

B propane, C3H8

C silicon tetrachloride, SiCl4

D sulfur hexafluoride, SF6

© UCLES 2017 9701/11/O/N/17 3

4 In the sodium chloride lattice the number of chloride ions that surround each sodium ion is called the co-ordination number of the sodium ions.

What are the co-ordination numbers of the sodium ions and the chloride ions in the sodium chloride lattice?

sodium ions chloride ions

A 4 6 B 6 4 C 6 6 D 8 6

3 5 A fluorescent light tube has an internal volume of 400 cm and an internal pressure of 200 kPa.

It is filled with 0.03 moles of an ideal gas.

What is the temperature of the gas inside the fluorescent light tube?

–1 A 3.21 × 10 K

2 B 3.21 × 10 K

5 C 3.21 × 10 K

8 D 3.21 × 10 K

6 One of the reactions in a lead / acid cell is shown.

+ 2– Pb(s) + PbO2(s) + 4H (aq) + 2SO4 (aq) → 2PbSO4(s) + 2H2O(l)

Which statement about this reaction is correct?

A Lead is both oxidised and reduced. B Lead is neither oxidised nor reduced. C Lead is oxidised only. D Lead is reduced only.

© UCLES 2017 9701/11/O/N/17 [Turn over 4

7 Iodine and propanone react according to the following equation.

I2(aq) + CH3COCH3(aq) → CH3COCH2I(aq) + HI(aq)

If the concentration of propanone is increased, keeping the total reaction volume constant, the rate of the reaction also increases.

What could be the reason for this?

A A greater proportion of collisions is successful at the higher concentration. B The particles are further apart at the higher concentration. C The particles have more energy at the higher concentration. D There are more collisions between reactant particles per second at the higher concentration.

8 Sulfur can be oxidised in two ways.

o –1 S(s) + O2(g) → SO2(g) ∆H = –296.5 kJ mol

o –1 2S(s) + 3O2(g) → 2SO3(g) ∆H = –791.4 kJ mol

Sulfur trioxide can be made from sulfur dioxide and oxygen.

2SO2(g) + O2(g) → 2SO3(g)

What is the standard enthalpy change for this reaction?

–1 A –1384.4 kJ mol

–1 B –989.8 kJ mol

–1 C –494.9 kJ mol

–1 D –198.4 kJ mol

9 Hydrogen iodide dissociates into hydrogen and iodine.

2HI(g) H2(g) + I2(g)

In an experiment, b mol of hydrogen iodide were put into a sealed vessel at pressure p. At equilibrium, x mol of the hydrogen iodide had dissociated.

Which expression for Kp is correct?

2 2 2 2 2 2 A x B x p C x p D x (b − x)2 (b − x)2 4b(b − x) 4(b − x)2

© UCLES 2017 9701/11/O/N/17 5

10 The diagram shows the distribution of molecular energies in a sample of gas at a temperature T1. The activation energy for an uncatalysed reaction of this gas, Ea(uncat), is shown.

T 1 E proportion a(uncat) of molecules with a given energy

0 0 E molecular energy

Which diagram correctly shows the new distribution and new activation energy, Ea(cat), when the temperature is increased to T2, and a catalyst is used that increases the rate of the reaction?

A B T T E 1 1 a(cat) E E proportion T a(uncat) proportion T a(uncat) 2 2 of molecules E of molecules with a given a(cat) with a given energy energy

0 0 0 molecular energy E 0 molecular energy E

C D T E T 1 a(cat) 1 E E proportion a(uncat) proportion a(uncat) of molecules of molecules E with a given with a given a(cat) energy T energy T 2 2 0 0 0 E 0 E molecular energy molecular energy

11 200 g of water are at 25 °C.

The water is heated to 75 °C by burning 2 g of ethanol.

What is the amount of energy transferred to the water?

A 0.418 kJ B 10.4 kJ C 41.8 kJ D 62.7 kJ

© UCLES 2017 9701/11/O/N/17 [Turn over 6

12 The elements Cl , Mg, Si and S are all in Period 3.

What is the correct sequence of the melting points of these elements, from lowest to highest?

lowest highest

melting point melting point

A Cl S Mg Si B Cl S Si Mg C Mg Si S Cl D Si Mg S Cl

13 An element Y reacts according to the following sequence.

burns an excess of l in O2 HC (aq) NaOH(aq) NaOH(aq) Y white solid solutionwhite precipitate solution

What could be element Y?

A Al B Ca C Mg D P

14 Which compound would most usually be added to soil to reduce its acidity?

A aluminium hydroxide B calcium hydroxide C magnesium hydroxide D sodium hydroxide

15 The mineral dolomite is a mixture of magnesium carbonate and calcium carbonate.

An aqueous reagent, X, was added to a small sample of dolomite. Effervescence was seen and a white solid, Y, was formed.

What could be the correct identity of reagent X and solid Y?

reagent X solid Y

A hydrochloric acid calcium chloride B hydrochloric acid magnesium chloride C sulfuric acid calcium sulfate D sulfuric acid magnesium sulfate

© UCLES 2017 9701/11/O/N/17 7

16 Which fertiliser contains the greatest percentage of nitrogen by mass?

A ammonium nitrate, NH4NO3

B ammonium sulfate, (NH4)2SO4

C diammonium hydrogen phosphate, (NH4)2HPO4

D urea, CO(NH2)2

17 71.0 g of chlorine, Cl 2, react with an excess of sodium hydroxide solution at a particular temperature. The reaction produces exactly 35.5 g of product X.

What is product X?

A H2O B NaCl C NaCl O D NaCl O3

18 Compound Q is a white crystalline solid which dissolves easily in water. When concentrated sulfuric acid is added to a dry sample of Q steamy white fumes are formed which, when passed through aqueous silver nitrate solution, form a white precipitate. This precipitate is soluble in dilute ammonia solution.

What could be the identity of compound Q?

A AgCl B NaBr C NaCl D PbBr2

19 The strengths of the covalent bonds within halogen molecules, and the van der Waals’ forces between halogen molecules, vary going down Group 17 from chlorine to bromine to iodine.

Which row shows these correctly?

strength of strength of

covalent bonds van der Waals’ forces

A decreases decreases B decreases increases C increases decreases D increases increases

© UCLES 2017 9701/11/O/N/17 [Turn over 8

20 The structural formula of compound Q is shown.

O

Q

How many stereoisomers exist with this structural formula?

A 1 B 2 C 4 D 8

21 What is the name of compound X?

OH

X compound

A trans-2-hydroxyhex-3-ene B trans-2-hydroxyhexene C trans-5-hydroxyhex-3-ene D trans-5-hydroxyhexene

22 Many, but not all, organic reactions need to be heated before a reaction occurs.

Which reaction occurs quickly at room temperature, 20 °C?

A C2H4 + Br2 → C2H4Br2

B C2H4 + H2O → CH3CH2OH

C CH3CH2OH → C2H4 + H2O

D CH3CH2OH + HBr → CH3CH2Br + H2O

© UCLES 2017 9701/11/O/N/17 9

23 A section of an addition polymer chain is shown.

CH CH2 CH2 CH l l C C

Which monomer could be used to make this polymer?

A CH2CHCH2Cl

B CH2CHCl

C CH3CHCHCl

D CHCl CHCH2CH2Cl

24 Which organic reaction is an example of nucleophilic substitution?

A CH3CH2Br + NaOH → CH2CH2 + H2O + NaBr

B CH3CH2Br + NaOH → CH3CH2OH + NaBr

C CH2CH2 + HCl → C2H5Cl

D C2H6 + Cl 2 → C2H5Cl + HCl

25 Citric acid can be converted into tricarballylic acid in two stages. An intermediate, Q, is formed.

CH2 CO2H CH2 CO2H stage 1 stage 2 HO CQCO2H CH CO2H

CH2 CO2H CH2 CO2H

citric acid tricarballylic acid

Which reagents are needed for each stage?

stage 1 stage 2

A concentrated H2SO4 H2(g) and Ni

B concentrated H2SO4 LiAl H4

C LiAl H4 H2SO4(aq)

D NaOH(aq) H2(g) and Ni

© UCLES 2017 9701/11/O/N/17 [Turn over 10

26 Glucose can be used to prepare sorbitol, a compound used as a sugar substitute.

H OH OH OH OH OH OH O HO OH OH OH OH

glucose sorbitol

Which reagent may be used for this conversion?

A acidified potassium dichromate(VI) B sodium borohydride C sodium hydroxide D Tollens’ reagent

27 3-methylbutanone is treated with alkaline aqueous iodine. The mixture of products is then acidified.

Which compound is present in the final mixture of products?

A 3-methylbutanoic acid B butanoic acid C methylpropanoic acid D propanoic acid

28 At room temperature, propanoic acid was reacted to produce sodium propanoate. No gas was produced during the reaction.

What could the propanoic acid have reacted with?

A NaHCO3(aq) B NaOH(aq) C Na2CO3(aq) D Na2SO4(aq)

© UCLES 2017 9701/11/O/N/17 11

29 Ethene is reacted with steam in the presence of concentrated H3PO4. The product of this reaction is added to acidified potassium dichromate(VI) and heated under reflux for one hour. The final organic product is collected and labelled X.

But-2-ene is treated with hot, concentrated, acidified potassium manganate(VII). The final organic product is collected and labelled Y.

Which statement is correct?

A One molecule of X has more carbon atoms than one molecule of Y. B One molecule of Y has more carbon atoms than one molecule of X. C X and Y have different functional groups. D X is the same compound as Y.

30 A sample of the ester CH3CH2CH2CO2CH2CH3 is hydrolysed. The product mixture is then treated with hot, acidified KMnO4.

What are the final carbon-containing products?

A CH3CH2CO2H only

B CH3CO2H + CH3CH2CO2H

C CH3CO2H + CH3CH2CH2CO2H

D CH3CH2OH + CH3CH2CH2CO2H

© UCLES 2017 9701/11/O/N/17 [Turn over 12

Section B

For each of the questions in this section, one or more of the three numbered statements 1 to 3 may be correct.

Decide whether each of the statements is or is not correct (you may find it helpful to put a tick against the statements that you consider to be correct).

The responses A to D should be selected on the basis of

A B C D

1, 2 and 3 1 and 2 2 and 3 1 only are only are only are is correct correct correct correct

No other combination of statements is used as a correct response.

Use of the Data Booklet may be appropriate for some questions.

31 The definitions of many chemical terms can be illustrated by chemical equations.

Which terms can be illustrated by an equation that includes the formation of a positive ion?

1 first ionisation energy 2 heterolytic fission of a covalent bond 3 enthalpy change of atomisation

32 A student makes sodium chloride by reacting together 0.025 mol of sodium carbonate with an –3 excess of 0.2 mol dm hydrochloric acid.

Na2CO3 + 2HCl → 2NaCl + H2O + CO2

Which statements about the quantities of substance are correct?

3 1 600 cm of carbon dioxide are produced at room temperature and pressure.

3 2 250 cm of the hydrochloric acid are needed to exactly neutralise the sodium carbonate.

3 1.46 g of sodium chloride are produced.

© UCLES 2017 9701/11/O/N/17 13

33 One way of recovering tin from old printed circuit boards is to dissolve it in a mixture of concentrated hydrochloric acid and concentrated nitric acid. The tin dissolves because it reacts with the mixture of these concentrated acids.

Sn + 4HCl + 2HNO3 → SnCl 4 + NO2 + NO + 3H2O

Which statements about this reaction are correct?

1 Nitrogen is present in three different oxidation states in the reactants and products. 2 The oxidation state of tin increases from 0 to +4. 3 The oxidation state of chlorine remains the same.

34 The following reaction takes place in a suitable solvent.

+ – + – + – Na NH2 + NH4 Cl → Na Cl + 2NH3

Which statements explain why this reaction should be classified as a Brønsted-Lowry acid-base reaction?

1 The ammonium ion acts as a proton donor.

+ – 2 Na Cl is a salt. 3 Ammonia is a nucleophile.

35 Which statements about the elements barium and calcium and their compounds are correct?

1 Barium nitrate decomposes at a higher temperature than calcium nitrate. 2 Barium hydroxide is more soluble in water than is calcium hydroxide. 3 Calcium is more reactive with water than is barium.

36 Which statements explain why nitrogen gas is unreactive?

1 Nitrogen atoms are highly electronegative. 2 Nitrogen molecules are non-polar. 3 The triple bond between nitrogen atoms is very strong.

37 In which molecules do all the carbon atoms lie in the same plane?

1 2,3-dimethylbut-2-ene 2 propane 3 cyclohexane

© UCLES 2017 9701/11/O/N/17 [Turn over 14

The responses A to D should be selected on the basis of

A B C D

1, 2 and 3 1 and 2 2 and 3 1 only are only are only are is correct correct correct correct

No other combination of statements is used as a correct response.

38 A reaction pathway diagram is shown.

energy

progress of reaction

Which reactions would have this reaction pathway diagram?

1 (CH3)3CBr + NaOH → (CH3)3COH + NaBr

2 CH3CH2CH2Br + NaOH → CH3CH2CH2OH + NaBr

3 (CH3)3CCH2CH2Cl + 2NH3 → (CH3)3CCH2CH2NH2 + NH4Cl

39 The compounds below are used to make perfumes.

Which compounds will produce a yellow precipitate with alkaline aqueous iodine?

132

OH OH HO O

OH

OH

© UCLES 2017 9701/11/O/N/17 15

40 The reaction of ethanal, CH3CHO, with HCN to form 2-hydroxypropanenitrile is catalysed by NaCN.

What are features of the intermediate of this reaction?

1 It is chiral. 2 It has a single negative charge on one of its atoms. 3 It is a nucleophile.

© UCLES 2017 9701/11/O/N/17 Cambridge International Examinations Cambridge International Advanced Subsidiary and Advanced Level *9349272649*

CHEMISTRY 9701/21

Paper 2 AS Level Structured Questions October/November 2017

1 hour 15 minutes Candidates answer on the Question Paper. Additional Materials: Data Booklet

READ THESE INSTRUCTIONS FIRST

Write your Centre number, candidate number and name on all the work you hand in. Write in dark blue or black pen. You may use an HB pencil for any diagrams or graphs. Do not use staples, paper clips, glue or correction fluid. DO NOT WRITE IN ANY BARCODES.

Answer all questions. Electronic calculators may be used. You may lose marks if you do not show your working or if you do not use appropriate units. A Data Booklet is provided.

At the end of the examination, fasten all your work securely together. The number of marks is given in brackets [ ] at the end of each question or part question.

This document consists of 9 printed pages and 3 blank pages.

IB17 11_9701_21/FP © UCLES 2017 [Turn over 2

Answer all the questions in the spaces provided.

1 Ammonia, NH3, is manufactured from nitrogen and hydrogen by the Haber process.

–1 N2(g) + 3H2(g) 2NH3(g) ΔH = –92 kJ mol

(a) Some bond energies are given.

N≡N = 944 kJ mol–1 H–H = 436 kJ mol–1

(i) Explain the meaning of the term bond energy.

......

...... [2]

(ii) Use the data to calculate a value for the N–H bond energy. You must show your working.

N–H bond energy = ...... kJ mol–1 [2]

(b) The Haber process is usually carried out at a temperature of approximately 400 °C in the presence of a catalyst. Changing the temperature affects both the rate of production of ammonia and the yield of ammonia.

The Boltzmann distribution for a mixture of nitrogen and hydrogen at 400 °C is shown. Ea represents the activation energy for the reaction.

proportion of molecules with a given energy

Ea molecular energy

(i) Using the same axes, sketch a second curve to indicate the Boltzmann distribution at a higher temperature. [2]

© UCLES 2017 9701/21/O/N/17 3

(ii) With reference to the Boltzmann distribution, state and explain the effect of increasing temperature on the rate of production of ammonia.

......

......

......

...... [3]

(iii) State and explain the effect of increasing temperature on the yield of ammonia. Use Le Chatelier’s principle to explain your answer.

......

......

......

...... [3]

7 (c) At a pressure of 2.00 × 10 Pa, 1.00 mol of nitrogen, N2(g), was mixed with 3.00 mol of hydrogen,

H2(g). The final equilibrium mixture formed contained 0.300 mol of ammonia, NH3(g).

(i) Calculate the amounts, in mol, of N2(g) and H2(g) in the equilibrium mixture.

N2(g) = ...... mol

H2(g) = ...... mol [2]

(ii) Calculate the partial pressure of ammonia, pNH3, in the equilibrium mixture.

Give your answer to three significant figures.

pNH3 = ...... Pa [3]

© UCLES 2017 9701/21/O/N/17 [Turn over 4

(d) In another equilibrium mixture the partial pressures are as shown.

substance partial pressure / Pa

6 N2(g) 2.20 × 10

5 H2(g) 9.62 × 10

4 NH3(g) 1.40 × 10

(i) Write the expression for the equilibrium constant, Kp, for the production of ammonia from nitrogen and hydrogen.

Kp =

[1]

(ii) Calculate the value of Kp for this reaction.

State the units.

Kp = ......

units = ...... [2]

(iii) This reaction is repeated with the same starting amounts of nitrogen and hydrogen. The same temperature is used but the container has a smaller volume.

State the effects, if any, of this change on the yield of ammonia and on the value of Kp.

effect on yield of ammonia ......

effect on value of Kp ...... [2]

[Total: 22]

© UCLES 2017 9701/21/O/N/17 5

2 The elements in the third period, and their compounds, show trends in their physical and chemical properties.

(a) A sketch graph of the first ionisation energies of five successive elements in the third period is shown.

ionisation P S energy Mg Si Al

atomic number

(i) Explain why there is a general increase in the first ionisation energy across the third period.

......

......

...... [2]

(ii) Sketch, on the graph, the position of the ionisation energies of the two elements that come before Mg in this sequence. [2]

(iii) Explain, with reference to electron arrangements, the decreases in first ionisation energy between Mg and Al and between P and S.

Mg and Al ......

......

......

P and S ......

......

...... [4]

© UCLES 2017 9701/21/O/N/17 [Turn over 6

(b) The chlorides of the elements in the third period behave in different ways when added to water, depending on their structure and bonding.

L and M are each a chloride of an element in Period 3. A student investigated L and M and their results are given.

L is a white crystalline solid with a melting point of 987 K. L dissolves in water to form an approximately neutral solution. Addition of NaOH(aq) to an aqueous solution of L produces a white precipitate.

M is a liquid with a boiling point of 331 K. M is hydrolysed rapidly by cold water to form a strongly acidic solution, a white solid and white fumes.

Identify L and M. Explain any properties and observations described. Give equations where appropriate.

(i) L is ......

......

......

......

...... [3]

(ii) M is ......

......

......

......

...... [3]

[Total: 14]

© UCLES 2017 9701/21/O/N/17 7

3 Some reactions based on 1-bromobutane, CH3(CH2)3Br, are shown.

reaction 1 reaction 2 CH3(CH2)3OH CH3(CH2)3Br CH3CH2CH=CH2

reaction 5 reaction 3

CH3(CH2)2CHO CH3(CH2)3C≡N

reaction 6 reaction 4

CH3(CH2)2COOH CH3(CH2)3COOH

(a) For each of the reactions state the reagent(s), the particular conditions required, if any, and the type of reaction.

For the type of reaction choose from the list. Each type may be used once, more than once or not at all. Each reaction may be described by more than one type.

elimination hydrolysis substitution

oxidation addition condensation

reaction reagent(s) and conditions type(s) of reaction

1

2

3

4

5

6

[6]

© UCLES 2017 9701/21/O/N/17 [Turn over 8

(b) Complete the diagram to show the SN2 mechanism of reaction 1. R represents the CH3(CH2)2 group. Include all necessary charges, dipoles, lone pairs and curly arrows.

H H

R CBr R COH

H H

[2]

(c) 2-bromo-2-methylpropane is a tertiary halogenoalkane that is a structural isomer of 1-bromobutane.

(i) Define the term structural isomer and name the three different types of structural isomerism.

definition ......

......

......

......

types of structural isomerism

1 ......

2 ......

3 ...... [4]

(ii) 2-bromo-2-methylpropane is treated with the same reagents as in reaction 1. Methylpropan-2-ol is formed.

Identify the mechanism for this reaction. Explain why this reaction proceeds via a different mechanism from that of reaction 1.

mechanism ......

explanation ......

......

......

......

...... [3]

© UCLES 2017 9701/21/O/N/17 9

(d) The product of reaction 2, but-1-ene, does not show stereoisomerism. However, but-1-ene reacts with HCl to form a mixture of structural isomers X and Y.

X (exists as a pair of stereoisomers and is produced in higher yield than Y) but-1-ene + HCl Y (does not show stereoisomerism)

(i) Explain the meaning of the term stereoisomers.

......

......

...... [2]

(ii) Give two reasons why but-1-ene does not show stereoisomerism.

......

......

...... [2]

(iii) Name X and Y.

X ......

Y ...... [2]

(iv) Name the type of stereoisomerism shown by X.

...... [1]

(v) Use the conventional representation to draw the two stereoisomers of X.

[2]

[Total: 24]

© UCLES 2017 9701/21/O/N/17 [Turn over